[obm-l] Exclusão da Lista

2007-04-03 Por tôpico Frederico Reis Marques de Brito

Solicito minha exclusão da lista OBM.

Obrigado.

Att,

Frederico Reis M. Brito.

_
O Windows Live Spaces é seu espaço na internet com fotos (500 por mês), blog 
e agora com rede social http://spaces.live.com/


=
Instruções para entrar na lista, sair da lista e usar a lista em
http://www.mat.puc-rio.br/~nicolau/olimp/obm-l.html
=


Re: [obm-l] Problemas Primos

2006-04-23 Por tôpico Frederico Reis Marques de Brito
Os exercícios 1 e 2 devem estar enunciados erroneamente. O 1o por ser falso 
e o 2o pore ser absolutamente óbvio. Para o 3o, sugiro olhar a prova de que 
um número natural que não tem divisor primo = a sua raiz quadrada é primo( 
que se encontra em qq livro de teoria dos num. ) . A prova do exercício é 
análoga.


Quanto ao 4). Pelo pequeno teorema de FermaT, p^{q-1}==1 ( mod q)   e 
q^{p-1} ==1 ( mod p ). Segue que:

p^{q-1} + q^{p-1} ==p^{q-1}(mod q) ==1 ( mod p )  e, analogamente,
p^{q-1} + q^{p-1}== 1 ( mod q).
, ou seja, p| (p^{q-1} + q^{p-1} ) e  q | (p^{q-1} + q^{p-1})
Como p, q são primos distintos ( portanto coprimos ), decorre que se p| x e 
q| x então p.q | x. Assim:


p^{q-1} + q^{p-1}==1 ( mod p.q ).



From: Bruno França dos Reis [EMAIL PROTECTED]
Reply-To: obm-l@mat.puc-rio.br
To: obm-l@mat.puc-rio.br
Subject: Re: [obm-l] Problemas Primos
Date: Sun, 23 Apr 2006 20:16:30 -0300

1) Tem certeza desse enunciado?
O número 97^2 = 9409  1000 é inteiro e composto, e NÃO tem um fator primo
menor que 37; seu menor fator primo é 97.

On 4/22/06, Ricardo Khawge [EMAIL PROTECTED] wrote:

 Se alguém puder me ajudar nestas questões eu agradeço:

 1) Mostrar que todo inteiro composto maior que 1000 tem um fator primo
 menor
 que 37.

 2) Mostrar que um inteiro da forma 4^(2n+1) nunca é primo.

 3) Mostrar que, se p não divide n, para todos  os  primos p menores ou
 iguais a raiz cúbica de n, então n é primo ou é o produto de dois 
primos.


 4) Sejam p e q primos distintos. Demonstrar: p^(q-1) + q^(p-1) ==1(mod 
pq)


 Obrigado

 _
 COPA 2006: Enfeite o seu MSN Messenger de verde e amarelo!
 http://copa.br.msn.com/extra/emoticons/

 
=

 Instruções para entrar na lista, sair da lista e usar a lista em
 http://www.mat.puc-rio.br/~nicolau/olimp/obm-l.html
 
=





--
Bruno França dos Reis
email: bfreis - gmail.com
gpg-key: http://planeta.terra.com.br/informatica/brunoreis/brunoreis.key
icq: 12626000

e^(pi*i)+1=0



=
Instruções para entrar na lista, sair da lista e usar a lista em
http://www.mat.puc-rio.br/~nicolau/olimp/obm-l.html
=


[obm-l] RE: [obm-l] dúvida conceitual

2005-07-17 Por tôpico Frederico Reis Marques de Brito
Normalmente convenciona-se uma inclusão natural de R^2 em R^3 considerando 
R^2={(x,y,0); x,y e R }.


Dessa forma, seria necess[ário que os dois vetores LI estivessem nesse R^2. 
Entretanto, dois vetores LI em R^3 geram um plano, plano esse isomorfo ao 
R^2.


Frederico.


From: nilton rr [EMAIL PROTECTED]
Reply-To: obm-l@mat.puc-rio.br
To: obm-l@mat.puc-rio.br
Subject: [obm-l] dúvida conceitual
Date: Sun, 17 Jul 2005 10:41:41 -0300 (ART)

Bom dia aos amigos da lista!
Posso dizer que dois vetores do R³ que sejam L.I. constituem uma base do 
R²? Grato.



-
Yahoo! Acesso Grátis: Internet rápida e grátis. Instale o discador agora!


_
MSN Messenger: converse online com seus amigos .  
http://messenger.msn.com.br


=
Instruções para entrar na lista, sair da lista e usar a lista em
http://www.mat.puc-rio.br/~nicolau/olimp/obm-l.html
=


[obm-l] RE: [obm-l] Dúvida

2005-07-17 Por tôpico Frederico Reis Marques de Brito
Um quadrado perfeito é um número a da forma a=n^2, com n inteiro. Assim, 0, 
1, 4, 9, 16, 25, 36, são os primeiros quadrados perfeitos.


Quanto a maneiras de se efetuar ciontas mais rapidamente existem várias que 
se aplicam a casos especiais e, eu particularmente abomino este tipo de 
truque. Você pode encontrar algumas coisas nas RPM.


Saudações,
Frederico.



From: admath [EMAIL PROTECTED]
Reply-To: obm-l@mat.puc-rio.br
To: obm-l obm-l@mat.puc-rio.br
Subject: [obm-l] Dúvida
Date: Sun, 17 Jul 2005 22:11:16 -0300

O que é um quadrado perfeito?
Alguém conhece algumas maneiras de efetuar contas seja de soma, subtração, 
divisão ou multiplicação de uma maneira mais rápida?


Obrigado.


_
MSN Messenger: converse online com seus amigos .  
http://messenger.msn.com.br


=
Instruções para entrar na lista, sair da lista e usar a lista em
http://www.mat.puc-rio.br/~nicolau/olimp/obm-l.html
=


[obm-l] RE: [obm-l] Professor de matemática...

2005-07-01 Por tôpico Frederico Reis Marques de Brito
Vc encontra a solução na própria RPM a de num. 47. Esse exemplar vc encontra 
disponível online na pagina da SBM, www.sbm.org.br, em material online.


Um abraço,
Frederico.


From: [EMAIL PROTECTED]
Reply-To: obm-l@mat.puc-rio.br
To: obm-l@mat.puc-rio.br
Subject: [obm-l] Professor de matemática...
Date: Fri, 1 Jul 2005 13:11:14 EDT

Esse problema, parece-me, foi proposto na revista professor de matemática.
   Qualquer um que desembarque nessa ilha verá imediatamente dois grandes
carvalhos, que chamarei de A e B , e também uma palmeira que chamarei de C.
Enterrei o tesouro em um ponto X que pode ser encontrado da seguinte forma.
Caminhe de C para A, contando seus passos. Chegando em A, vire para a 
esquerda e dê
exatamente o mesmo número de passos para chegar ao ponto M. Volte no ponto 
C.
Caminhe de C para B, contando seus passos. Chegando em B, vire para a 
direita

e dê exatamente o mesmo número de passos, para chegar ao ponto N. O ponto
Xestá na reta que liga M a N, à mesma distância desses dois pontos.
  Voltando a Ilha os carvalhos A e B ainda estavam lá, mas a palmeira C 
tinha

desaparecido. O tesouro está perdido??
   Não sei se este problema foi discutido aqui. se foi peço desculpas e 
uma

indicação de onde encontro a resposta...
 Agradeço desde já a quem puder me ajudar...
  Korshinoi


_
Chegou o que faltava: MSN Acesso Grátis. Instale Já! 
http://www.msn.com.br/discador


=
Instruções para entrar na lista, sair da lista e usar a lista em
http://www.mat.puc-rio.br/~nicolau/olimp/obm-l.html
=


Re: [obm-l] Oswald de Souza (off)

2005-06-22 Por tôpico Frederico Reis Marques de Brito


Ao que consta esse sr, sequer é Matemático e apenas faz parte da realidade 
distorcida produzida pela TV Globo. ainda que o fosse, apareceria fazendo as 
vezes de um estatístico. Assim, a tv propaga e difunde a falsa idéia de que 
Matemática é apenas matemática financeira e probabilidade básica.


Frederico.


From: [EMAIL PROTECTED]
Reply-To: obm-l@mat.puc-rio.br
To: obm-l@mat.puc-rio.br
Subject: Re: [obm-l] Oswald de Souza (off)
Date: Tue, 21 Jun 2005 19:09:40 EDT

Eu ia perguntar isso, mas acabei esquecendo, pois tenho a mesma 
curiosidade.

Outra pergunta:

Se houvesse uma seleção dos 10 brasileiros mais hábeis em resolver 
problemas

de matemática, quantos, vocês acham, que sairiam dessa lista ?

Vou começar opinando:

10 ;-) LOLOL



Em uma mensagem de 21/06/05 19:27:14 Hora padrão leste da Am. Sul,
[EMAIL PROTECTED] escreveu:


 Assunto:[obm-l] Oswald de Souza (off)
 Data:21/06/05 19:27:14 Hora padrão leste da Am. Sul
 De:[EMAIL PROTECTED]
 Responder-para:obm-l@mat.puc-rio.br
 Para:obm-l@mat.puc-rio.br
 Enviado pela Internet



 Sei que isso é totalmente  off-topic, mas qdo se fala
 em matematica todo leigo pensa em Oswald de Souza
 (aquele que fica falando de loteria, futebol), alguem
 ai sabe onde esse cara se formou, se pesquisa
 matematica...





 ___
 Yahoo! Acesso Grátis - Internet rápida e grátis.
 Instale o discador agora! http://br.acesso.yahoo.com/
 
=

 Instruções para entrar na lista, sair da lista e usar a lista em
 http://www.mat.puc-rio.br/~nicolau/olimp/obm-l.html
 
=







[]`s
Rafael


_
MSN Messenger: converse online com seus amigos .  
http://messenger.msn.com.br


=
Instruções para entrar na lista, sair da lista e usar a lista em
http://www.mat.puc-rio.br/~nicolau/olimp/obm-l.html
=


RE: [obm-l] ax + by = c

2005-06-15 Por tôpico Frederico Reis Marques de Brito

ESsa é uma Equação Diofantina.

Como vc mesmo notou o mdc (23, 10 ) =1. Assim, existe uma combinação linear 
inteira de 10 e 23 dando 1, isto é, existem x* e y* em Z tq  23x* + 10y* = 
1. Multiplique x* e y* por 5 e vc obterá uma solução particular da eq. 
diofantina.


É fácil ver q todas as soluçlões da eq. serão da forma: x = x* - 10k  e y = 
y* + 23k, com k inteiro.


Qq dúvida, escreva novamente ou consulte um çlivro de Teor dos Num q tenha 
um capítulo sobre eq. diofantinas.


Fred.


From: Maurício [EMAIL PROTECTED]
Reply-To: obm-l@mat.puc-rio.br
To: obm-l@mat.puc-rio.br
Subject: [obm-l] ax + by = c
Date: Tue, 14 Jun 2005 21:35:25 -0700 (PDT)


  Oi, pessoal,

  Estou lendo um livro de teoria dos números que me
pede como exercício que resolva a equação:

  ax + by = c

para x e y, com a,x,b,y,c inteiros. O livro não diz
como fazer. Como c tem que ser múltiplo do máximo
divisor comum o que eu fiz foi adaptar o algoritmo do
Euclides para calcular o mdc, ou seja, eu calculo o
resto de a/b, depois o resto de b dividido por esse
resto etc., só que a cada passo eu anoto o x e o y que
fornecem cada resto. Por exemplo:

  23x + 10y = 5

  Monto essa tabela de (x,y,c):

1 , 0 , 23
0 , 1 , 10
1 , -2 , 3
-3 , 7 , 1

  Aí é só multiplicar por 5: (x,y) = (-3*5,7*5).
  Esse tipo de equação aparece bastante nos exercícios
que estou fazendo. Existe alguma outra maneira de
resolver, mais simples? Também: é possivel resolver
algo do tipo ax=b(mod m) sem resolver completamente ax
+ km = b?

  Obrigado,
  Maurício




__
Discover Yahoo!
Have fun online with music videos, cool games, IM and more. Check it out!
http://discover.yahoo.com/online.html
=
Instruções para entrar na lista, sair da lista e usar a lista em
http://www.mat.puc-rio.br/~nicolau/olimp/obm-l.html
=


_
MSN Messenger: converse online com seus amigos .  
http://messenger.msn.com.br


=
Instruções para entrar na lista, sair da lista e usar a lista em
http://www.mat.puc-rio.br/~nicolau/olimp/obm-l.html
=


[obm-l] RE: [obm-l] Re:[obm-l] DEmonstração Mais elementar.

2005-04-03 Por tôpico Frederico Reis Marques de Brito
Olá Cláudio. está aí o nó da questão. Não conheço demonstração de que 1/p 
seja dízima periódica simples que não use o Peq. teorema...

Um abraço,
Frederico.
From: claudio.buffara [EMAIL PROTECTED]
Reply-To: obm-l@mat.puc-rio.br
To: obm-l obm-l@mat.puc-rio.br
Subject: [obm-l] Re:[obm-l] DEmonstração Mais elementar.
Date: Sat,  2 Apr 2005 16:36:05 -0300
Se p = 3, então p divide 111, 11, 1, e qualquer número formado 
por 3k algarismos 1 (k inteiro positivo).

Suponhamos, portanto, que p  2, 3 e 5.
Nesse caso, 1/p é uma dízima periódica simples (não sei se isso é mais 
fácil de demonstrar do que o pequeno teorema de Fermat ou o teorema de 
Euler)

Escrevendo 1/p = 0,a_1a_2...a_na_1a_2...a_na_1a_2...,
teremos 10^n/p = a_1a_2...a_n,a_1a_2a_na_1a_2...
de forma que (10^n - 1)/p = a_1a_2...a_n, ou seja,
p divide 10^n - 1 = 9*11...1
Como p não divide 9, p divide N = 11...1 (n algarismos 1).
Além disso, os números (10^n+1)*N, (10^(2n)+10^n+1)*N, ... são todos 
formados apenas por algarismos 1 e são obviamente divisíveis por p.

[]s,
Claudio.

De:[EMAIL PROTECTED]
Para:obm-l@mat.puc-rio.br
Cópia:
Data:Sat, 02 Apr 2005 13:06:30 -0300
Assunto:[obm-l] DEmonstração Mais elementar.


 Olá a todos.

 è bem conhecido o fato de que se p é primo diferente de 2 e 5 então p
 divide infinitos dos
 números R_n:=(10^n-)/9. Entretanto, a demonstração mais direta usa o 
Peq.
 Teorema de Fermat, que
 não é um resultado elementar. O fato está relacionado com a 
periodicidade da
 expansão decimal de 1/p. Gostaria de obter uma demonstração alternativa,
 que usasse fatos mais elementares. Alguém conhece alguma?

 Agradeço desde já a todas as sugestões.
 Um abraço a todos,
 Frederico.

 _
 Chegou o que faltava: MSN Acesso Grátis. Instale Já!
 http://www.msn.com.br/discador

 
=
 Instruções para entrar na lista, sair da lista e usar a lista em
 http://www.mat.puc-rio.br/~nicolau/olimp/obm-l.html
 
=

_
Chegou o que faltava: MSN Acesso Grátis. Instale Já! 
http://www.msn.com.br/discador

=
Instruções para entrar na lista, sair da lista e usar a lista em
http://www.mat.puc-rio.br/~nicolau/olimp/obm-l.html
=


[obm-l] DEmonstração Mais elementar.

2005-04-02 Por tôpico Frederico Reis Marques de Brito

Olá a todos.
è bem conhecido o fato de que se  p é primo diferente de 2 e 5 então   p 
divide infinitos dos
números R_n:=(10^n-)/9.  Entretanto, a demonstração mais direta usa o Peq. 
Teorema de Fermat, que
não é um resultado elementar. O fato está relacionado com a periodicidade da 
expansão decimal de 1/p.  Gostaria de obter uma demonstração alternativa, 
que usasse fatos mais elementares. Alguém conhece alguma?

Agradeço desde já a todas as sugestões.
Um abraço a todos,
Frederico.
_
Chegou o que faltava: MSN Acesso Grátis. Instale Já! 
http://www.msn.com.br/discador

=
Instruções para entrar na lista, sair da lista e usar a lista em
http://www.mat.puc-rio.br/~nicolau/olimp/obm-l.html
=


RE: [obm-l] Definicao de Equacoes.

2005-03-03 Por tôpico Frederico Reis Marques de Brito
Uma equação algébrica nada mais é q algo da forma P(x_1,...,x_n)=0 em que P 
é um poliNômio em n variáveis.

Por outro lado, uma eq. diferencial, é algo como F(x_1,x_2,...,x_n, y, 
y_1,..y_m ) = 0, em que F é uma função ( em geral consideramos F contínua ou 
contínua qtp ), y=y(x_1,...,x_n),  e  os y_i irepresentam derivadas parciais 
de y com relação às variáveis independentes x_1,..,x_n.

Acho que essas definições estão razoavelmente formais.
Abraços,
Frederico.

From: Fabio Niski [EMAIL PROTECTED]
Reply-To: obm-l@mat.puc-rio.br
To: obm-l@mat.puc-rio.br
Subject: [obm-l] Definicao de Equacoes.
Date: Thu, 03 Mar 2005 16:50:56 -0300
Pessoal, acredito nunca ter lido a definicao formal de equação algebrica e 
equação diferencial. Alguem poderia postar aqui? Por se tratar de algo 
facil intuitivamente de entender, acredito que as definicoes sejam 
engenhosas.

um abraco
Niski
=
Instruções para entrar na lista, sair da lista e usar a lista em
http://www.mat.puc-rio.br/~nicolau/olimp/obm-l.html
=
_
MSN Messenger: converse online com seus amigos .  
http://messenger.msn.com.br

=
Instruções para entrar na lista, sair da lista e usar a lista em
http://www.mat.puc-rio.br/~nicolau/olimp/obm-l.html
=


[obm-l] Re: [obm-l] Matemática Hoje como anda???

2005-01-31 Por tôpico Frederico Reis Marques de Brito
Caro Cláudio e demais colegas da lista. Sobre os problemas do Instituto 
Clay, conhecidos como Problemas do Milênio, o principal , creio eu, é a 
Hipótese de Riemann, que aliás já constava da lista dos problemas de 
Hilbert, no início do século passado. A editora Record lançou em 2004 o 
livro Os Problemas do Milênio - Sete Grandes Enigmas Matemáticos do Nosso 
Tempo, de Keith Devlin. O livro
não é escrito para matemáticos, mas para curiosos no assunto. Tem linguagem 
fácil e tenta tornar os conceitos envolvidos nos problemas o mais acessível. 
O único porém é que alguns verbetes foram mal traduzidos, mas nada que 
comprometa a leitura. Recomendo.

Abraços a todos,
Frederico.
From: Claudio Buffara [EMAIL PROTECTED]
Reply-To: obm-l@mat.puc-rio.br
To: obm-l@mat.puc-rio.br
Subject: Re: [obm-l] Matemática Hoje como anda???
Date: Sun, 30 Jan 2005 23:47:21 -0200
Segundo os matematicos (pelo menos aqueles consultados pelo instituto 
Clay),
os problemas mais relevantes estao descritos aqui:
http://www.claymath.org/millennium/

[]s,
Claudio.
on 30.01.05 23:03, André Barreto at [EMAIL PROTECTED]
wrote:
Oi amigos.
Caramba creio que SBM está bem melhor que a sociedade brasileira de física 
e
a de química... Tenho a sensação de que a de matemática trabalha mais...
essas coisas como a lista a eureka os livros e tudo mas que acho tenham
ligação também com o IMPA são bem legais... Elas deviam fazer isso 
também...

Bem vamos ao que interessa eu gostaria de levantar um questionamento.
Quais são hoje as fronteiras da matemática... O que hoje mais intrigam os
matemáticos. Ou seja, a matemática moderna está a tratar de que
atualmente??? Já percebi que muitos tópicos atuais da matemática estão a
trabalhar conjuntamente com a física, problemas na engenharia, computação e
etc... mas há algo hoje sendo pesquisado da matemática para a matemática 
com
extrema importancia.

Atenciosamente
André Sento Sé Barreto

_
MSN Messenger: converse online com seus amigos .  
http://messenger.msn.com.br

=
Instruções para entrar na lista, sair da lista e usar a lista em
http://www.mat.puc-rio.br/~nicolau/olimp/obm-l.html
=


Re: [obm-l] Livros ( OFF - TOPIC)

2005-01-19 Por tôpico Frederico Reis Marques de Brito
Para Geometria Riemanniana, sugiro começar pelo clássico do Manfredo, 
editado pelo IMPA. O livro
é, de fato, uma obra-prima.

Abraço,
Frederico.
From: Alan Pellejero [EMAIL PROTECTED]
Reply-To: obm-l@mat.puc-rio.br
To: obm-l@mat.puc-rio.br
Subject: Re: [obm-l] Livros ( OFF - TOPIC)
Date: Wed, 19 Jan 2005 09:40:37 -0300 (ART)
Olá amigos da lista...
Por falar em livros, gostaria de receber algumas
sugestões de livros para estudar geometria riemmaniana
e cálculo tensorial.
Grato!
 --- Felipe Nardes [EMAIL PROTECTED]
escreveu:
 vou fazer a turma ITA do poliedro esse ano e
 gostaria que vocês me
 indicassem bom livros de matemática, física e
 química para complementar as
 apostilas que o curso utiliza. Já tenho os livros do
 2º grau, por isso
 queria pegar alguns livros melhores, com exercícios
 mais difíceis e uma
 teoria mais completa.

 valeu


_
 MSN Messenger: converse online com seus amigos .
 http://messenger.msn.com.br


=
 Instruções para entrar na lista, sair da lista e
 usar a lista em
 http://www.mat.puc-rio.br/~nicolau/olimp/obm-l.html

=



___
Yahoo! Acesso Grátis - Instale o discador do Yahoo! agora. 
http://br.acesso.yahoo.com/ - Internet rápida e grátis
=
Instruções para entrar na lista, sair da lista e usar a lista em
http://www.mat.puc-rio.br/~nicolau/olimp/obm-l.html
=
_
MSN Hotmail, o maior webmail do Brasil.  http://www.hotmail.com
=
Instruções para entrar na lista, sair da lista e usar a lista em
http://www.mat.puc-rio.br/~nicolau/olimp/obm-l.html
=


Re: [obm-l] Numeros naturais

2005-01-13 Por tôpico Frederico Reis Marques de Brito
Seja k o mínimo de X ( existe pelo princípio da boa Ordenação ). Então X = 
k. Z. De fato, obviamente kZ está contido em X. Qto a recíproca, suponha que 
exista x em X  que não
seja múltiplo de k . Pela minimalidade de k,  x  k e podemos escrever
x=k.q + r   com 0 r  k  , usando o Lema da Divisão. Agora, x e k.q  
pertencem
a X = r pertence a X, o que contraria a minimalidade de k ...

Sdações,
Fred.

From: Kellem :-) 100% SeJ [EMAIL PROTECTED]
Reply-To: obm-l@mat.puc-rio.br
To: obm-l@mat.puc-rio.br
Subject: Re: [obm-l] Numeros naturais
Date: Thu, 13 Jan 2005 21:02:07 -0200
Oi Tertuliano
é m e n pertencem sse m e m+n pertencem ou m e n pertencem sse M.N  e m+n
(...)?
Valeu
kellem
- Original Message -
From: Tertuliano Carneiro [EMAIL PROTECTED]
To: obm-l@mat.puc-rio.br
Sent: Thursday, January 13, 2005 6:32 PM
Subject: [obm-l] Numeros naturais
 Olá para todos!
 Gostaria q alguem me ajudasse nesta questao:

 Seja X um subconjunto nao vazio dos naturais e tq m e
 n pertencem a x sse m e m+n pertencem a X. Prove q
 existe k natural tq x é o conjunto dos multiplos de k.

 Um abraco!





 ___
 Yahoo! Acesso Grátis - Instale o discador do Yahoo! agora.
http://br.acesso.yahoo.com/ - Internet rápida e grátis
 
=
 Instruções para entrar na lista, sair da lista e usar a lista em
 http://www.mat.puc-rio.br/~nicolau/olimp/obm-l.html
 
=


=
Instruções para entrar na lista, sair da lista e usar a lista em
http://www.mat.puc-rio.br/~nicolau/olimp/obm-l.html
=
_
MSN Messenger: converse online com seus amigos .  
http://messenger.msn.com.br

=
Instruções para entrar na lista, sair da lista e usar a lista em
http://www.mat.puc-rio.br/~nicolau/olimp/obm-l.html
=


[obm-l] Re: [obm-l] RE: [obm-l] Pra que serve a matemática?

2004-12-29 Por tôpico Frederico Reis Marques de Brito
Recomendo fortemente a leitura de EM DEFESA DE UM MATEMÁTICO, do G. H. 
Hardy. ( È um livro
barato da Martins Fontes... ). Durante a leitura vc será forçado a refletir 
em 1o lugar sobre o que é a matemática e somente
após essa etapa fará sentido arguir sobre utilidade... Até porque diga-se de 
passagem que bombas nucleares tem utilidade...

Abraços a todos, e desejo de um feliz 2005.
Frederico.
From: [EMAIL PROTECTED]
Reply-To: obm-l@mat.puc-rio.br
To: obm-l@mat.puc-rio.br
Subject: Re: [obm-l] RE: [obm-l] Pra que serve a matemática?
Date: Tue, 28 Dec 2004 21:58:32 +
Fora que ela é um prazeroso entretenimento... além de produzir belas obras
de arte! :)
[]s,
Daniel
Leandro Lacorte Recova ([EMAIL PROTECTED]) escreveu:

Carissimo Bruno,

Essa pergunta seria o mesmo que Pra que serve tomar agua ? . Agua e 
algo
essencial para a sobrevivencia de todos seres humanos, assim como a
matematica e essencial para o desenvolvimento da humanidade. Energia que
chega na sua casa, Telefones Celulares, Robos em fabricas, maquinas de
tomografia computadorizada, e outras coisas.

Seria impossivel listar toda a utilidade dessa ciencia num simples 
e-mail.

Faca voce mesmo uma pesquisa no Google.

Regards,

Leandro
Los Angeles, CA.

-Original Message-
From: [EMAIL PROTECTED] [mailto:[EMAIL PROTECTED] On
Behalf Of Bruno Soares
Sent: Tuesday, December 28, 2004 10:44 AM
To: obm-l@mat.puc-rio.br
Subject: [obm-l] Pra que serve a matemática?

Boa tarde

Pra que serve a matemática?

Pergunta um tanto óbvia, mas quando pensamos que algo é muito óbvio, é
quando não estamos pensando.

=
Instruções para entrar na lista, sair da lista e usar a lista em
http://www.mat.puc-rio.br/~nicolau/olimp/obm-l.html
=

=
Instruções para entrar na lista, sair da lista e usar a lista em
http://www.mat.puc-rio.br/~nicolau/olimp/obm-l.html
=


=
Instruções para entrar na lista, sair da lista e usar a lista em
http://www.mat.puc-rio.br/~nicolau/olimp/obm-l.html
=
_
MSN Messenger: converse online com seus amigos .  
http://messenger.msn.com.br

=
Instruções para entrar na lista, sair da lista e usar a lista em
http://www.mat.puc-rio.br/~nicolau/olimp/obm-l.html
=


[obm-l] Re: [obm-l] Curiosidades Geométricas

2004-10-15 Por tôpico Frederico Reis Marques de Brito
Só tem um PEQUENO problema, a garrafa não está mergulhada em R^3, 
diferentemente como ocorre com a faixa de Möbius...

From: Douglas Drumond [EMAIL PROTECTED]
Reply-To: [EMAIL PROTECTED]
To: [EMAIL PROTECTED]
Subject: Re: [obm-l] Curiosidades Geométricas
Date: Thu, 14 Oct 2004 17:54:59 -0300
Garrafa de Klein.
On Thu, 14 Oct 2004 09:43:17 -0300 (ART), Valdery Sousa
[EMAIL PROTECTED] wrote:

 Olá pessoal!

 Já pensaram em uma figura espacial com uma única superfície?
 É simples de obte-la:  pegue uma fita circular  fechada ,em forma de
 anel, e façam um corte de modo a transformar-se em um retângulo.
 Inverta os lados, e cole de modo a q a parte q estava para dentro fique
 para fora.

 Vcs obterão a chamada fita de Mobius, com uma única superfície!
 (Normalmente, pensando em termos espaciais,  há sempre uma superfície
 interna e uma externa .)
 Agora digam-me: pode haver uma garrafa sem fundo?
 Divirtam-se!

 Cordialmente,
 Valdery.


 
 Yahoo! Acesso Grátis - Internet rápida e grátis. Instale o discador 
agora!



=
Instruções para entrar na lista, sair da lista e usar a lista em
http://www.mat.puc-rio.br/~nicolau/olimp/obm-l.html
=
_
MSN Hotmail, o maior webmail do Brasil.  http://www.hotmail.com
=
Instruções para entrar na lista, sair da lista e usar a lista em
http://www.mat.puc-rio.br/~nicolau/olimp/obm-l.html
=


[obm-l] RE: [obm-l] Um clássico da matemática

2004-07-12 Por tôpico Frederico Reis Marques de Brito
BOm, não creioo que alguém tenha autoridade para criticar o artigo do prof. 
Manfredo do Carmo. Cabe
esclarecer , Manfredo Perdigão do Carmo é o maior geômetra brasileiro, um 
dos maiores do mundo...



From: Raphael Marx [EMAIL PROTECTED]
Reply-To: [EMAIL PROTECTED]
To: [EMAIL PROTECTED]
Subject: [obm-l] Um clássico da matemática Date: Mon, 12 Jul 2004 05:09:14 
-0700 (PDT)

http://www2.uol.com.br/cienciahoje/ch/ch205/memoria.htm
Vai aí um artigo interessante, podendo ser sujeito a crítcas pela lista em 
sua historicidade ou mesmo na sua matemática.
Gostaria de saber o que houve com aquele site disponibilizado para provas 
do IME. Se alguém puder envie-me as provas para 
[EMAIL PROTECTED]
Agradecidamente, Raphael Marques Franco


-
Do you Yahoo!?
New and Improved Yahoo! Mail - 100MB free storage!
_
MSN Hotmail, o maior webmail do Brasil.  http://www.hotmail.com
=
Instruções para entrar na lista, sair da lista e usar a lista em
http://www.mat.puc-rio.br/~nicolau/olimp/obm-l.html
=


[obm-l] RE: [obm-l] Re: [obm-l] Hipótese de Riemann

2004-06-14 Por tôpico Frederico Reis Marques de Brito
Digamos que, se um número fosse primo qdo fosse divisível por 1 e por ele 
mesmo, então todos os nos, execto o zero, seriam primos...

Ainda que ácrescentássemos; no primo é um inteiro que é divisível APENAS por 
1 e por ele mesmo, estaríamos errados.

A definição correta é então: UM número inteiro POSITIVO é primo qdo tem 
EXATAMENTE dois divisores positivos distintos. Assim, -3 não é primo e 1 tb 
não o é. ( Aqui cabe dizer que, em algumas situações é conveniente
aceitar -3 e -5 como primos, mas nunca o 1 ). Essa definição é feita a bem 
do Teorema Fundamental da aritmética, qwue garante a unicidade da fatoração 
em primos ( a menos da ordem dos fatores). Se não, vejamos: 6 = (-2)(-3)=2 . 
3 = 1 . 2 . 3  seriam três decomposições em primos, distintas.


From: Fabiano Sant'Ana [EMAIL PROTECTED]
Reply-To: [EMAIL PROTECTED]
To: [EMAIL PROTECTED]
Subject: [obm-l] Re: [obm-l] Hipótese de Riemann Date: Mon, 14 Jun 2004 
11:27:08 -0300

o que são primos então?
Abraços
Fabiano Sant'Ana
(desculpa ficar insistindo no mesmo assunto, é que fiquei curioso) :)
- Original Message -
From: [EMAIL PROTECTED]
To: [EMAIL PROTECTED]
Sent: Sunday, June 06, 2004 12:43 PM
Subject: [obm-l] Hipótese de Riemann
 Saudadade do tempo em aprendi que núemro primo é aquele que se divide 
por
1 e por ele mesmo.
 Abraço para a lista.


=
Instruções para entrar na lista, sair da lista e usar a lista em
http://www.mat.puc-rio.br/~nicolau/olimp/obm-l.html
=
_
MSN Hotmail, o maior webmail do Brasil.  http://www.hotmail.com
=
Instruções para entrar na lista, sair da lista e usar a lista em
http://www.mat.puc-rio.br/~nicolau/olimp/obm-l.html
=


RE: [obm-l] Modulos

2004-05-31 Por tôpico Frederico Reis Marques de Brito
Imagino que 1/2 ( mod 23 ) represente o inverso de 2 módulo 23, a saber 12. 
Portanto:
-1/2 mod 23 = -12 ( mod 23 ) = 11 ( mod 23 ).

E o inverso de 4 ( mod 23 ) é 6 =  x = 6 (  mod 23 ) , no exercício (2).
Frederico. .

From: paulobarclay [EMAIL PROTECTED]
Reply-To: [EMAIL PROTECTED]
To: obm-l [EMAIL PROTECTED]
Subject: [obm-l] Modulos
Date: Mon, 31 May 2004 17:17:53 -0300
Gostaria de uma ajuda para resolver as seguintes
equações:
1)-1/2mod23=x
2)1/4mod23=x
desde já agradeço.
paulo
__
Acabe com aquelas janelinhas que pulam na sua tela.
AntiPop-up UOL - É grátis!
http://antipopup.uol.com.br/

=
Instruções para entrar na lista, sair da lista e usar a lista em
http://www.mat.puc-rio.br/~nicolau/olimp/obm-l.html
=
_
MSN Hotmail, o maior webmail do Brasil.  http://www.hotmail.com
=
Instruções para entrar na lista, sair da lista e usar a lista em
http://www.mat.puc-rio.br/~nicolau/olimp/obm-l.html
=


Re: [obm-l] Novamente as gavetas

2004-05-11 Por tôpico Frederico Reis Marques de Brito
Na minha solução também bastam 53 números, já que foram formados 52 
conjuntos...
Um abraço,
Fred.


From: Claudio Buffara [EMAIL PROTECTED]
Reply-To: [EMAIL PROTECTED]
To: [EMAIL PROTECTED]
Subject: Re: [obm-l] Novamente as gavetas
Date: Tue, 11 May 2004 15:24:30 -0300
Gostei! Por alguma razao, eu nunca me lembro de particionar o conjunto-base
em pares. A minha solucao foi mais complicada, mas acho que consegui
melhorar o resultado para 53 elementos (ao inves de 55).
[]s,
Claudio.
on 11.05.04 14:49, Frederico Reis Marques de Brito at 
[EMAIL PROTECTED]
wrote:

 Pois bem, então onde está o erro do seguinte raciocínio: separe os 
números
 de 1 a 100 em conjuntos como os seguintes:

 {1,13} , {2,14} ,{3,15}, ..., {12, 24}
 {25,37} , {26, 38 }, ..., {36, 48}
 {49, 61} , {50, 62} ,..., {60, 72}
 {73, 84}, {74, 85} , ..., {84, 96}

 e  {97}, {98}, {99} , {100}. TEmos ao todo 4* 12 + 4 = 52 conjuntos
 disjuntos cuja união dá o conjunto dos naturais de 1 a 100, inclusive. 
Dados
 55  desses números, 2 terão que estar num mesmo subconjunto. Isso não 
pode
 ocorrer nos últimos 4 subconjuntos , que são unitários. Logo,
 há dois números entre 1 e 100 que estão num dos primeiros 48 
subconjuntos,
 que são todos da forma {a , a+12} = a diferença entre esses dois 
números é
 precisamente 12!??!?

 Um abraço,
 FRed.


 From: Claudio Buffara [EMAIL PROTECTED]
 Reply-To: [EMAIL PROTECTED]
 To: [EMAIL PROTECTED]
 Subject: Re: [obm-l] Novamente as gavetas
 Date: Tue, 11 May 2004 13:59:05 -0300

 on 11.05.04 12:48, Frederico Reis Marques de Brito at
 [EMAIL PROTECTED]
 wrote:

 Bom, dessa vez o resultado é verdadeiro.
 Provar que dados  55 números inteiros entre 1 e 100, incluindo estes,
 existem dois cuja diferença é exatamente 12.
 Um abraço a todos,
 Fred.


=
Instruções para entrar na lista, sair da lista e usar a lista em
http://www.mat.puc-rio.br/~nicolau/olimp/obm-l.html
=
_
MSN Messenger: converse com os seus amigos online.  
http://messenger.msn.com.br

=
Instruções para entrar na lista, sair da lista e usar a lista em
http://www.mat.puc-rio.br/~nicolau/olimp/obm-l.html
=


[obm-l] Novamente as gavetas

2004-05-11 Por tôpico Frederico Reis Marques de Brito
Bom, dessa vez o resultado é verdadeiro.
Provar que dados  55 números inteiros entre 1 e 100, incluindo estes, 
existem dois cuja diferença é exatamente 12.
Um abraço a todos,
Fred.


From: Ricardo Bittencourt [EMAIL PROTECTED]
Reply-To: [EMAIL PROTECTED]
To: [EMAIL PROTECTED]
Subject: Re: [obm-l] Princípio de Dirichlet - variacao
Date: Tue, 11 May 2004 01:14:24 -0300
Frederico Reis Marques de Brito wrote:

Ricardo, não sei o que quiz dizer com a 1a parte, mas a segunda está 
correta e, portanto, a afirmação é FALSA!
É que eu por um instante achei que a afirmação fosse
verdadeira; mas como triângulos equiláteros eu já sabia que
iam dar problema, resolvi ver se tinha outro jeito de dispor
três pontos equidistantes no plano sem ser em triângulo
equilátero. Mas no final não tinha, o triângulo equilátero
é o único jeito mesmo.

Ricardo Bittencourt   http://www.mundobizarro.tk
[EMAIL PROTECTED]   tenki ga ii kara sanpo shimashou
-- União contra o forward - crie suas proprias piadas --
=
Instruções para entrar na lista, sair da lista e usar a lista em
http://www.mat.puc-rio.br/~nicolau/olimp/obm-l.html
=
_
MSN Messenger: converse com os seus amigos online.  
http://messenger.msn.com.br

=
Instruções para entrar na lista, sair da lista e usar a lista em
http://www.mat.puc-rio.br/~nicolau/olimp/obm-l.html
=


Re: [obm-l] Novamente as gavetas

2004-05-11 Por tôpico Frederico Reis Marques de Brito
Pois bem, então onde está o erro do seguinte raciocínio: separe os números 
de 1 a 100 em conjuntos como os seguintes:

{1,13} , {2,14} ,{3,15}, ..., {12, 24}
{25,37} , {26, 38 }, ..., {36, 48}
{49, 61} , {50, 62} ,..., {60, 72}
{73, 84}, {74, 85} , ..., {84, 96}
e  {97}, {98}, {99} , {100}. TEmos ao todo 4* 12 + 4 = 52 conjuntos 
disjuntos cuja união dá o conjunto dos naturais de 1 a 100, inclusive. Dados 
55  desses números, 2 terão que estar num mesmo subconjunto. Isso não pode 
ocorrer nos últimos 4 subconjuntos , que são unitários. Logo,
há dois números entre 1 e 100 que estão num dos primeiros 48 subconjuntos, 
que são todos da forma {a , a+12} = a diferença entre esses dois números é 
precisamente 12!??!?

Um abraço,
FRed.

From: Claudio Buffara [EMAIL PROTECTED]
Reply-To: [EMAIL PROTECTED]
To: [EMAIL PROTECTED]
Subject: Re: [obm-l] Novamente as gavetas
Date: Tue, 11 May 2004 13:59:05 -0300
on 11.05.04 12:48, Frederico Reis Marques de Brito at 
[EMAIL PROTECTED]
wrote:

 Bom, dessa vez o resultado é verdadeiro.
 Provar que dados  55 números inteiros entre 1 e 100, incluindo estes,
 existem dois cuja diferença é exatamente 12.
 Um abraço a todos,
 Fred.


Oi, Fred:
E quanto aos 60 numeros abaixo?

1,2,3,4,5,6,7,8,9,10,11,12,
25,26,27,28,29,30,31,32,33,34,35,36,
49,50,51,52,53,54,55,56,57,58,59,60,
63,64,64,66,67,68,69,70,71,72,73,74,
87,88,89,90,91,92,93,94,95,96,97,98.
[]s,
Claudio.
=
Instruções para entrar na lista, sair da lista e usar a lista em
http://www.mat.puc-rio.br/~nicolau/olimp/obm-l.html
=
_
MSN Messenger: converse com os seus amigos online.  
http://messenger.msn.com.br

=
Instruções para entrar na lista, sair da lista e usar a lista em
http://www.mat.puc-rio.br/~nicolau/olimp/obm-l.html
=


[obm-l] Re: [obm-l] RE: [obm-l] Princípio de Dirichlet - variacao

2004-05-10 Por tôpico Frederico Reis Marques de Brito
Ricardo, não sei o que quiz dizer com a 1a parte, mas a segunda está correta 
e, portanto, a afirmação é FALSA!

Um abraço,
fred.

From: Ricardo Bittencourt [EMAIL PROTECTED]
Reply-To: [EMAIL PROTECTED]
To: [EMAIL PROTECTED]
Subject: Re: [obm-l] RE: [obm-l] Princípio de Dirichlet - variacao
Date: Sun, 09 May 2004 22:59:54 -0300
Frederico Reis Marques de Brito wrote:
Considere o conjunto S dos pontos do R^2 que distam, na métrica 
euclidiana, 1 unidade da origem do R^2. Se a cada ponto de S  associarmos 
um elemento do conjunto  T={A,B} então existirão sempre três pontos de S 
equidistantes ( na métrica euclidiana ) associados a um mesmo elemento de 
T.
Mas é verdade isso mesmo? Sejam dois pontos A,B;
então o lugar geométrico dos pontos distantes do ponto A
um comprimento d(AB) é um círculo de raio d(AB) centrado
em A, o mesmo vale pra B. Os dois círculos se encontram
em dois pontos, que determinam as duas únicas possíveis
posições para um ponto C tal que os três sejam equidistantes,
e nessas condições ABC formam um triângulo equilátero.
Agora, se pra resolver o problema você precisa
inscrever um triângulo equilátero no seu conjunto S,
então vai dar zica. Quebre o conjunto S em três intervalos
semi-abertos R=[0,120[ , S=[120,240[ , T=[240,360[
(ângulos em graus). Para um triângulo equilátero estar
inscrito no conjunto S, precisa ter um ponto em cada
um desses intervalos. Mas agora eu pinto de azul os
conjuntos R e S, e de vermelho o conjunto T, e garanto
que não há triângulos equiláteros com vértices de mesma cor.

Ricardo Bittencourt   http://www.mundobizarro.tk
[EMAIL PROTECTED]   tenki ga ii kara sanpo shimashou
-- União contra o forward - crie suas proprias piadas --
=
Instruções para entrar na lista, sair da lista e usar a lista em
http://www.mat.puc-rio.br/~nicolau/olimp/obm-l.html
=
_
MSN Hotmail, o maior webmail do Brasil.  http://www.hotmail.com
=
Instruções para entrar na lista, sair da lista e usar a lista em
http://www.mat.puc-rio.br/~nicolau/olimp/obm-l.html
=


[obm-l] Re: [obm-l] Re:_[obm-l]_Princípio_de_Dirichlet

2004-05-09 Por tôpico Frederico Reis Marques de Brito
Lamentável perceber, novamente, que alguns particípantes da lista usam-na 
com fins diversos
dos quais ela foi criada. Concordo que o enunciado, tal como proposto, não é 
nenhum modelo de precisão, mas também acho que seu real significado é por 
demais evidente e que o objetivo era , tão somente, exemplificar o uso do 
Princípio das Gavetas num caso não muito trivial...
Servaõ-lhe de lição os possíveis erros do passado... 

Frederico.

From: Johann Peter Gustav Lejeune Dirichlet 
[EMAIL PROTECTED]
Reply-To: [EMAIL PROTECTED]
To: [EMAIL PROTECTED]
Subject: Re: [obm-l] Re:_[obm-l]_Princípio_de_Dirichlet
Date: Sun, 9 May 2004 16:21:11 -0300 (ART)

Basta parti-lo ao meio!
--- Fabiano Sant'Ana [EMAIL PROTECTED]
escreveu:  como um simples ponto poderá possuir
Duas
 cores?
 - Original Message -
 From: Frederico Reis Marques de Brito
 [EMAIL PROTECTED]
 To: [EMAIL PROTECTED]
 Sent: Saturday, May 08, 2004 10:54 PM
 Subject: [obm-l] Princípio de Dirichlet


  Mais um probleminha de contagem:
 
  Se pintarmos cada ponto de um círculo com
 duas cores, de forma aleatória,
  então existirão
  três pontos equidistantes pintados com a
 mesma cor.
 
 
  Fred.
 
 

_
  MSN Hotmail, o maior webmail do Brasil.
 http://www.hotmail.com
 
 

=
  Instruções para entrar na lista, sair da
 lista e usar a lista em
 

http://www.mat.puc-rio.br/~nicolau/olimp/obm-l.html
 

=
 


 ---
 Outgoing mail is certified Virus Free.
 Checked by AVG anti-virus system
 (http://www.grisoft.com).
 Version: 6.0.676 / Virus Database: 438 -
 Release Date: 04/05/04


=
 Instruções para entrar na lista, sair da lista
 e usar a lista em

http://www.mat.puc-rio.br/~nicolau/olimp/obm-l.html

=
=
TRANSIRE SVVM PECTVS MVNDOQVE POTIRI
CONGREGATI EX TOTO ORBE MATHEMATICI OB SCRIPTA INSIGNIA TRIBVERE
Fields Medal(John Charles Fields)
N.F.C. (Ne Fronti Crede)

__
Yahoo! Messenger - Fale com seus amigos online. Instale agora!
http://br.download.yahoo.com/messenger/
=
Instruções para entrar na lista, sair da lista e usar a lista em
http://www.mat.puc-rio.br/~nicolau/olimp/obm-l.html
=
_
MSN Messenger: converse com os seus amigos online.  
http://messenger.msn.com.br

=
Instruções para entrar na lista, sair da lista e usar a lista em
http://www.mat.puc-rio.br/~nicolau/olimp/obm-l.html
=


[obm-l] RE: [obm-l] Princípio de Dirichlet - variacao

2004-05-09 Por tôpico Frederico Reis Marques de Brito
Convém então esclarecer novamente uma confusão. Embora não haja muito 
consenso a respeito das nomenclaturas entre círculo e circunferência, é mais 
comum referir-se a região delimitada por uma circunferência como disco, até 
porque essa nomenclatura não dá margem a ambiguidade. O problema que propus 
não foi o que o Ricardo resolveu, mas basicamente o que o Cláudio ora 
propõe.  Assim sendo, reformulo o enunciado , deixando-o mais exato e 
formal:

Considere o conjunto S dos pontos do R^2 que distam, na métrica euclidiana, 
1 unidade da origem do R^2. Se a cada ponto de S  associarmos um elemento do 
conjunto  T={A,B} então existirão sempre três pontos de S equidistantes ( na 
métrica euclidiana ) associados a um mesmo elemento de T.

Abraços a todos,
FRED.

From: Claudio Buffara [EMAIL PROTECTED]
Reply-To: [EMAIL PROTECTED]
To: [EMAIL PROTECTED]
Subject: [obm-l] Princípio de Dirichlet - variacao
Date: Sun, 09 May 2004 15:53:12 -0300
Frederico Reis Marques de Brito wrote:

 Se pintarmos cada ponto de um círculo com [uma dentre] duas cores,
 de forma aleatória, então existirão
 três pontos equidistantes pintados com a mesma cor.

E se ao inves de circulo (ou seja, disco) o enunciado falasse em
circunferencia (de modo que nao pudessemos usar o centro)?
Ainda teriamos um triangulo equilatero com os 3 vertices da mesma cor?
[]s,
Claudio.
=
Instruções para entrar na lista, sair da lista e usar a lista em
http://www.mat.puc-rio.br/~nicolau/olimp/obm-l.html
=
_
MSN Hotmail, o maior webmail do Brasil.  http://www.hotmail.com
=
Instruções para entrar na lista, sair da lista e usar a lista em
http://www.mat.puc-rio.br/~nicolau/olimp/obm-l.html
=


[obm-l] Princípio de Dirichlet

2004-05-08 Por tôpico Frederico Reis Marques de Brito
Mais um probleminha de contagem:
Se pintarmos cada ponto de um círculo com duas cores, de forma aleatória, 
então existirão
três pontos equidistantes pintados com a mesma cor.

Fred.
_
MSN Hotmail, o maior webmail do Brasil.  http://www.hotmail.com
=
Instruções para entrar na lista, sair da lista e usar a lista em
http://www.mat.puc-rio.br/~nicolau/olimp/obm-l.html
=


Re: [obm-l] congruencia e aritmetica modular

2004-04-04 Por tôpico Frederico Reis Marques de Brito
Formalmente, a segunda resposta está, tal como proposta, errada. O resto é 
um ( dos infinitos ) representantes da classe modular. assim, saliento,  11 
( mod 4 ) não é um número, taõ pouco o resto de 11 por 4, mas é a classe do  
3 ( mod 4 )...

Frederico.

From: Rafael [EMAIL PROTECTED]
Reply-To: [EMAIL PROTECTED]
To: [EMAIL PROTECTED]
Subject: Re: [obm-l] congruencia e aritmetica modular
Date: Sat, 3 Apr 2004 16:38:55 -0300
Creio que, para ambas as perguntas, a resposta seja sim.

De acordo com a definição:

A = B (mod n) == (A-B)/n é inteiro

- Exemplo:

6 = 2 (mod 4), pois (6-2)/4 = 1 que é inteiro

Para a segunda pergunta:

Seja B = q*n + r  e  0 = r  n,

B mod n = r

- Exemplo:

9 mod 4 = 1, pois 9 / 4 = 2 e resto 1.

Abraços,

Rafael de A. Sampaio



- Original Message -
From: André Zimmermann [EMAIL PROTECTED]
To: [EMAIL PROTECTED]
Sent: Saturday, April 03, 2004 2:52 PM
Subject: [obm-l] congruencia e aritmetica modular


Pessoal,

É satisfatório e suficiente dizer que:

A é congruente a B (módulo n) se n for divisor da diferença entre A e B ?

E que B módulo n é igual ao resto da divisão inteira de B por n ?

Estas são as dúvidas de um cérebro enferrujado

Obrigado pelo desengripante.

André.

=
Instruções para entrar na lista, sair da lista e usar a lista em
http://www.mat.puc-rio.br/~nicolau/olimp/obm-l.html
=
_
MSN Messenger: converse com os seus amigos online.  
http://messenger.msn.com.br

=
Instruções para entrar na lista, sair da lista e usar a lista em
http://www.mat.puc-rio.br/~nicolau/olimp/obm-l.html
=


RE: [obm-l] LIVRO

2004-03-01 Por tôpico Frederico Reis Marques de Brito
Recomendo:
[1] Teoria ingênua dos conjuntos  - Paul R. Halmos, editado em português 
pela Ciência Moderna, 2001.
[2] A Matemática do Ensino Médio - vol. 1 - Elon, Morgado, eduardo Wagner e 
Paulo Cezar P. Carvalho, da SBM.  O 1o capítulo é dedicado aos conjuntos.

Boa sorte!
Frederico.

From: Nelson [EMAIL PROTECTED]
Reply-To: [EMAIL PROTECTED]
To: [EMAIL PROTECTED]
Subject: [obm-l] LIVRO
Date: Mon, 1 Mar 2004 09:14:21 -0300 (ART)
Olá a todos, gostaria de saber se existe algum livro (ou site) que contenha 
um número razoável de demonstrações de propriedades elementares envolvendo 
noções de conjuntos.

Desde já, agradeço.
Nelson


-
Yahoo! Mail - O melhor e-mail do Brasil. Abra sua conta agora!
_
MSN Hotmail, o maior webmail do Brasil.  http://www.hotmail.com
=
Instruções para entrar na lista, sair da lista e usar a lista em
http://www.mat.puc-rio.br/~nicolau/olimp/obm-l.html
=


Re: [obm-l] Equacao polinomial

2004-02-08 Por tôpico Frederico Reis Marques de Brito
Olá Rafael. O fato de nada se dizer sobre a multiplicidade da raiz significa 
que ela NÂO é prezumível, ou seja, você não pode assumir que a 
multiplicidade seja 10. Também não é nada claro( até porque é falso ) que 
apenas o coeficiente -10 determine os demais.
Quanto às médias, ó que posso dizer é que chamou-me a atenção o fato de ter 
sido nos dado o produto e a soma das raízes, no mais usei o que se chama de 
experiência matemática...

Um abraço,
frederico.

From: Rafael [EMAIL PROTECTED]
Reply-To: [EMAIL PROTECTED]
To: [EMAIL PROTECTED]
Subject: Re: [obm-l] Equacao polinomial
Date: Sat, 7 Feb 2004 15:47:39 -0200
Frederico,

A conclusão sobre a multiplicidade não ser simples foi tida a partir do
momento que nada se disse sobre as raízes serem distintas. Ora, a regra de
sinais de Descartes, conforme já comentei, assegura que os coeficientes
alternam-se se há dez raízes reais positivas, e sabemos que se trata de uma
equação de coeficientes 1, -10, ..., 1 cujas raízes são reais e positivas.
Nada mais se diz sobre os coeficientes, logo nada os restringe. E, ao meu
ver, o segundo coeficiente, -10, já nos garante que a solução para essa
equação é única, pois, se considerarmos o desenvolvimento binomial, somente
temos -10 para (x-1)^10.
Se ainda não for convincente, gostaria de maiores detalhes sobre o meu
erro, assim como de onde você partiu para calcular MA e MG das raízes,
conhecendo-se, até então, somente a soma e o produto delas.
Abraços,

Rafael de A. Sampaio



- Original Message -
From: Frederico Reis Marques de Brito [EMAIL PROTECTED]
To: [EMAIL PROTECTED]
Sent: Saturday, February 07, 2004 10:24 AM
Subject: Re: [obm-l] Equacao polinomial
 Bom Rafael, embora a resposta que vc obteve esteja correta, seu 
argumento
 não me parece convincente. Afinal, você não teve subsídios para concluir
que
 a raiz tinha multiplicidade 10...

 Observe que, pelas relações entre coef. e raízes, a soma das raízes vale
10
 e o produto vale 1. Por hipótese e pelo Teor . Fund. da àlgebra, temos 
10
 raízes reais e positivas. Decoorre que a média aritmética das raízes (
MA )
 é  1  e a média geométrica ( MG ) tb vale 1. Ora, como sabemos
 MG = MA e vale a igualdade se, e só se, todas as raízes são iguais.
 Portanto ~x=1 é uma raiz de multiplicidade 10.

 Um abraço,
 Frederico.

=
Instruções para entrar na lista, sair da lista e usar a lista em
http://www.mat.puc-rio.br/~nicolau/olimp/obm-l.html
=
_
MSN Hotmail, o maior webmail do Brasil.  http://www.hotmail.com
=
Instruções para entrar na lista, sair da lista e usar a lista em
http://www.mat.puc-rio.br/~nicolau/olimp/obm-l.html
=


Re: [obm-l] Equacao polinomial

2004-02-07 Por tôpico Frederico Reis Marques de Brito
Bom Rafael, embora a resposta que vc obteve esteja correta, seu argumento 
não me parece convincente. Afinal, você não teve subsídios para concluir que 
a raiz tinha multiplicidade 10...

Observe que, pelas relações entre coef. e raízes, a soma das raízes vale 10 
e o produto vale 1. Por hipótese e pelo Teor . Fund. da àlgebra, temos 10 
raízes reais e positivas. Decoorre que a média aritmética das raízes ( MA )  
é  1  e a média geométrica ( MG ) tb vale 1. Ora, como sabemos
MG = MA e vale a igualdade se, e só se, todas as raízes são iguais.  
Portanto ~x=1 é uma raiz de multiplicidade 10.

Um abraço,
Frederico.

From: Rafael [EMAIL PROTECTED]
Reply-To: [EMAIL PROTECTED]
To: [EMAIL PROTECTED]
Subject: Re: [obm-l] Equacao polinomial
Date: Sat, 7 Feb 2004 05:08:22 -0200
Cláudio,

A equação proposta por você é interessantíssima.

Pela regra de sinais de Descartes e do enunciado, sabemos que, se há dez
raízes reais e positivas, todos os coeficientes de índice par são positivos
e todos os de índice ímpar, negativos, admitindo-se que haja termos em x
cujo exponte varia de 2 a 6. Assim:
a_10 = 1  0, a_8  0, a_6  0, a_4  0, a_2  0, a_0 = 1  0

a_9 = -10  0, a_7  0, a_5  0, a_3  0, a_1  0

Como nada se diz quanto a serem raízes distintas, temos que (x-1)^10 =
x^10 - 10x^9 + 45x^8 - 120x^7 + 210x^6 - 252x^5 + 210x^4 - 120x^3 + 45x^2 -
10x + 1. Logo, x = 1 é solução única, cuja multiplicidade é 10.
Espero que esteja correto.

Abraços,

Rafael de A. Sampaio



- Original Message -
From: Claudio Buffara [EMAIL PROTECTED]
To: Lista OBM [EMAIL PROTECTED]
Sent: Saturday, February 07, 2004 1:43 AM
Subject: [obm-l] Equacao polinomial
 Jah que problemas envolvendo raizes de polinomios estao entre os mais
 populares da lista, aqui vai um:

 Determine as raizes de:
 x^10 - 10*x^9 + a_8*x^8 + a_7*x^7 + ... + a_1*x + 1 = 0, sabendo que 
todas
 elas sao reais e positivas.

 Um abraco,
 Claudio.

=
Instruções para entrar na lista, sair da lista e usar a lista em
http://www.mat.puc-rio.br/~nicolau/olimp/obm-l.html
=
_
MSN Messenger: converse com os seus amigos online.  
http://messenger.msn.com.br

=
Instruções para entrar na lista, sair da lista e usar a lista em
http://www.mat.puc-rio.br/~nicolau/olimp/obm-l.html
=


RE: [obm-l] Phi de Euler

2004-01-30 Por tôpico Frederico Reis Marques de Brito
Muito interessante essa demonstração combinatória!

Quanto a sua reformulação, ainda restringindo o contradomínio aos números 
pares a função phi é altamente não sobrejetiva...

Frederico.


From: Claudio Buffara [EMAIL PROTECTED]
Reply-To: [EMAIL PROTECTED]
To: [EMAIL PROTECTED]
Subject: [obm-l] Phi de Euler
Date: Fri, 30 Jan 2004 09:44:19 -0200
Oi, Platao e Duda:

Dentro do espirito de se buscar sempre a solucao mais bonita pra cada
problema, aqui vai a minha candidata pra este ai:
Se mdc(n,k) = 1 entao mdc(n,n-k) = 1. Logo, se n  2, podemos arranjar os
inteiros positivos menores que n e primos com n em pares disjuntos da forma
{k,n-k}. Isso quer dizer que Phi(n) eh par para n  2.
Eh claro que se k = n - k entao n = 2k e mdc(n,k) = k  1 (a menos que k = 
1
== n = 2, mas esse caso jah foi descartado).



E se mudarmos a pergunta original para: A imagem da funcao Phi contem todos
os inteiros positivos pares? (por exemplo, 14?)
Um abraco,
Claudio.
on 30.01.04 01:04, Eduardo Casagrande Stabel at [EMAIL PROTECTED]
wrote:
 Oi Platão e demais.

 Não querendo corrigir, mas já enriquecendo a mensagem do Platão. Se n é
 primo (com exceção a n=2) então Phi(n) = n-1 é par. Se n é potência de 
primo
 n = p^i (com i=2) então Phi(n) = p^i - p^(i-1) também é par. Já que a
 função Phi é multiplicatica, isto é, se mdc(m,n)=1 então Phi(mn) = 
Phi(m)
 Phi(n), então segue a conclusão de que, a menos para n = 2, Phi(n) é um
 número par.

 Para quem não conhece (a maioria), o Platão é amigo meu, de Novo 
Hamburgo, e
 portanto também gaúcho. Saudações ao mais novo membro da lista, todos
 esperamos boas contribuições como essa! Seja bem-vindo!

 Abração,
 Duda.


 From: Platão Gonçalves Terra Neto [EMAIL PROTECTED]
 Basta ver que se p é primo, ímpar, então phi(p)=p-1, par.
 Para n=b^c, b primo, phi(b^c)=b^c-b^(c-1), que é par, ou seja,  se
 n=a1^p2*a2^p2*...an^pn, sendo ai, todos primos , distintos , n2 e pi
 expoentes, então phi(n) é par.
 Se n=2^k, phi(n)=2^k-2^(k-1), que é par, exceção, para phi(2)=1.
 phi(1)=1.
 Logo, phi(n) é par , para todo n2, donde ,N* não é imagem de phi(n)
 - Original Message -
 From: André Martin Timpanaro [EMAIL PROTECTED]
 To: [EMAIL PROTECTED]
 Sent: Thursday, January 29, 2004 8:38 PM
 Subject: [obm-l] Dúvida


 A afirmação abaixo é verdadeira?

 Dado um número natural n não nulo existe algum natural m tal que
 phi(m)=n.
 Onde phi(x) é a função phi de Euler.
 Em outras palavras, a imagem de phi(x) é N* ?

 André T.

 _
 MSN Messenger: converse com os seus amigos online.
 http://messenger.msn.com.br


 
=
 Instruções para entrar na lista, sair da lista e usar a lista em
 http://www.mat.puc-rio.br/~nicolau/olimp/obm-l.html

 
=



=
Instruções para entrar na lista, sair da lista e usar a lista em
http://www.mat.puc-rio.br/~nicolau/olimp/obm-l.html
=
_
MSN Messenger: converse com os seus amigos online.  
http://messenger.msn.com.br

=
Instruções para entrar na lista, sair da lista e usar a lista em
http://www.mat.puc-rio.br/~nicolau/olimp/obm-l.html
=


Re: [obm-l] Phi de Euler

2004-01-30 Por tôpico Frederico Reis Marques de Brito
Dividindo em alguns casos é de fato possível demonstrar sua última 
afirmação. è fácil ver que   x teria que ser da forma   2^a .  q^b   , com  
a=0 ou 1 e   q   primo. Daí é só testar as possibilidades...

Imagino que exista alguma demonstração mais direta, mas essa é bem 
construtiva.

Frederico.


From: Claudio Buffara [EMAIL PROTECTED]
Reply-To: [EMAIL PROTECTED]
To: [EMAIL PROTECTED]
Subject: Re: [obm-l] Phi de Euler
Date: Fri, 30 Jan 2004 12:13:32 -0200
Com relacao a beleza matematica, uma regra que eu acho que falha pouco eh a
seguinte: se um resultado tem uma demonstracao combinatoria, entao essa
demonstracao eh a mais bonita. O unico contra-exemplo que me ocorre eh o
caso do uso de algebra linear pra se demonstrar alguns resultados de
combinatoria, mas isso eh a minha opiniao pessoal...
Outro resultado parecido que tem uma demonstracao combinatoria identica eh:
se n eh um inteiro positivo, entao d(n) eh impar se e somente se n eh
quadrado perfeito, onde d(n) = no. de divisores positivos de n.
Acho que dah pra provar que se p eh um primo tal que 2p+1 eh composto, 
entao
a equacao Phi(x) = 2p nao tem solucao.

Um abraco,
Claudio.
on 30.01.04 11:00, Frederico Reis Marques de Brito at 
[EMAIL PROTECTED]
wrote:

 Muito interessante essa demonstração combinatória!

 Quanto a sua reformulação, ainda restringindo o contradomínio aos 
números
 pares a função phi é altamente não sobrejetiva...

 Frederico.


 From: Claudio Buffara [EMAIL PROTECTED]
 Reply-To: [EMAIL PROTECTED]
 To: [EMAIL PROTECTED]
 Subject: [obm-l] Phi de Euler
 Date: Fri, 30 Jan 2004 09:44:19 -0200

 Oi, Platao e Duda:

 Dentro do espirito de se buscar sempre a solucao mais bonita pra cada
 problema, aqui vai a minha candidata pra este ai:
 Se mdc(n,k) = 1 entao mdc(n,n-k) = 1. Logo, se n  2, podemos arranjar 
os
 inteiros positivos menores que n e primos com n em pares disjuntos da 
forma
 {k,n-k}. Isso quer dizer que Phi(n) eh par para n  2.

 Eh claro que se k = n - k entao n = 2k e mdc(n,k) = k  1 (a menos que 
k =
 1
 == n = 2, mas esse caso jah foi descartado).

 

 E se mudarmos a pergunta original para: A imagem da funcao Phi contem 
todos
 os inteiros positivos pares? (por exemplo, 14?)

 Um abraco,
 Claudio.

 on 30.01.04 01:04, Eduardo Casagrande Stabel at [EMAIL PROTECTED]
 wrote:

 Oi Platão e demais.

 Não querendo corrigir, mas já enriquecendo a mensagem do Platão. Se n 
é
 primo (com exceção a n=2) então Phi(n) = n-1 é par. Se n é potência de
 primo
 n = p^i (com i=2) então Phi(n) = p^i - p^(i-1) também é par. Já que a
 função Phi é multiplicatica, isto é, se mdc(m,n)=1 então Phi(mn) =
 Phi(m)
 Phi(n), então segue a conclusão de que, a menos para n = 2, Phi(n) é 
um
 número par.

 Para quem não conhece (a maioria), o Platão é amigo meu, de Novo
 Hamburgo, e
 portanto também gaúcho. Saudações ao mais novo membro da lista, todos
 esperamos boas contribuições como essa! Seja bem-vindo!

 Abração,
 Duda.


 From: Platão Gonçalves Terra Neto [EMAIL PROTECTED]
 Basta ver que se p é primo, ímpar, então phi(p)=p-1, par.
 Para n=b^c, b primo, phi(b^c)=b^c-b^(c-1), que é par, ou seja,  se
 n=a1^p2*a2^p2*...an^pn, sendo ai, todos primos , distintos , n2 e pi
 expoentes, então phi(n) é par.
 Se n=2^k, phi(n)=2^k-2^(k-1), que é par, exceção, para phi(2)=1.
 phi(1)=1.
 Logo, phi(n) é par , para todo n2, donde ,N* não é imagem de phi(n)
 - Original Message -
 From: André Martin Timpanaro [EMAIL PROTECTED]
 To: [EMAIL PROTECTED]
 Sent: Thursday, January 29, 2004 8:38 PM
 Subject: [obm-l] Dúvida


 A afirmação abaixo é verdadeira?

 Dado um número natural n não nulo existe algum natural m tal que
 phi(m)=n.
 Onde phi(x) é a função phi de Euler.
 Em outras palavras, a imagem de phi(x) é N* ?

 André T.


=
Instruções para entrar na lista, sair da lista e usar a lista em
http://www.mat.puc-rio.br/~nicolau/olimp/obm-l.html
=
_
MSN Messenger: converse com os seus amigos online.  
http://messenger.msn.com.br

=
Instruções para entrar na lista, sair da lista e usar a lista em
http://www.mat.puc-rio.br/~nicolau/olimp/obm-l.html
=


Re: [obm-l] Impossibilidade do movimento

2004-01-24 Por tôpico Frederico Reis Marques de Brito
Isto é absolutamente falso. Observe que 1/(10^n)   tende a  0quando   n  
tender a infinito, de forma estritamente decrescente, isto é , se  n  m =  
1/(10^n)  1/(10^m), mas 0 não é um termo dessa sequência. Posto isto , é 
fácil ver que não existe um menor número e que as demais parcelas são 
múltiplas desta...

Frederico.


From: Marcelo Augusto Pereira [EMAIL PROTECTED]
Reply-To: [EMAIL PROTECTED]
To: [EMAIL PROTECTED]
Subject: Re: [obm-l] Impossibilidade do movimento
Date: Fri, 23 Jan 2004 22:10:01 -0200
O fato de essa soma ser calculável(1/9)  não indica que existe um número de
valor muito pequeno e que esse número seria o valor mínimo que possa
existir? Assim todos os outros números seriam múltiplos desse menor valor
possível, ou seja, esse número seria algo como um valor quântico. Dessa
forma, também existiria uma unidade quântica de deslocamento linear, o que
faria com que a quantidade de pontos em um segmento de reta não fosse
infinita e o movimento fosse possível. Se para cada número existisse um
menor, a soma teria que ser infinita, e o resultado infinito.
- Original Message -
From: Frederico Reis Marques de Brito [EMAIL PROTECTED]
To: [EMAIL PROTECTED]
Sent: Friday, January 23, 2004 9:27 PM
Subject: RE: [obm-l] Impossibilidade do movimento

 Essencialmente esse problema é ujm dos paradoxos de Zenão, um grego 
antigo
 que usava a idéia de infinito para chegar a conclusões aparentemente
 absurdas, tais como a impossibilidade do movimento, por exemplo. Agora 
vou
 dar uma de Dirichlet, o da lista é claro: Pense no seguinte, uma soma de
 infinitas parcelas positivas é sempre infinito, ou não necessariamente?
Para
 ajudar nessa resposta, pense em calcular, por exemplo: 1/10 + 1/100 +
1/1000
 + ...   . Bom e agora, o que tudo isto tem a ver com sua pergunta?

 Espero ter ajudado, apesar dessa resposta meio enigmática, mas acho que
 assim auxilio mais!

 Frederico.

 From: Marcelo Augusto Pereira [EMAIL PROTECTED]
 Reply-To: [EMAIL PROTECTED]
 To: [EMAIL PROTECTED]
 Subject: [obm-l] Impossibilidade do movimento
 Date: Fri, 23 Jan 2004 19:05:25 -0200
 
 Entre dois números reais há infinitos outros. Considere um segmento de
reta
 com o número 0 assinalado em uma ponta e o número 1 marcado na outra.
 Considere também que esse segmento de reta foi representado no chão com
um
 risco de um metro de comprimento. Para cada número entre 0 e 1 há um
ponto
 correspondente no segmento de reta e, conseqüentemente, no risco 
marcado
no
 chão. Como eu consigo caminhar do ponto 0 até o ponto 1, se para chegar
de
 0
 até 1 eu tenho que passar por infinitos pontos?
 
 
=
 Instruções para entrar na lista, sair da lista e usar a lista em
 http://www.mat.puc-rio.br/~nicolau/olimp/obm-l.html
 
=

 _
 MSN Hotmail, o maior webmail do Brasil.  http://www.hotmail.com

 
=
 Instruções para entrar na lista, sair da lista e usar a lista em
 http://www.mat.puc-rio.br/~nicolau/olimp/obm-l.html
 
=


=
Instruções para entrar na lista, sair da lista e usar a lista em
http://www.mat.puc-rio.br/~nicolau/olimp/obm-l.html
=
_
MSN Messenger: converse com os seus amigos online.  
http://messenger.msn.com.br

=
Instruções para entrar na lista, sair da lista e usar a lista em
http://www.mat.puc-rio.br/~nicolau/olimp/obm-l.html
=


Re: [obm-l] Impossibilidade do movimento

2004-01-24 Por tôpico Frederico Reis Marques de Brito
Vou entremear minha resposta na sua.
FRederico.
From: Marcelo Augusto Pereira [EMAIL PROTECTED]
Reply-To: [EMAIL PROTECTED]
To: [EMAIL PROTECTED]
Subject: Re: [obm-l] Impossibilidade do movimento
Date: Sat, 24 Jan 2004 13:02:41 -0200
1)Partindo desse princípio, pode-se dizer que a cada termo adicionado 
naquela
soma, o valor total aumenta. Por exemplo, se eu utilizar 10 termos eu tenho
um valor; se eu utilizar 100 termos eu tenho outro maior, e assim
sucessivamente.
(até aqui está certo.)

2)Desse modo, como a soma é infinita e possui estritamente
termos positivos, seu resultado deveria ser infinito.
(Isto é falso. Embora concorde em ter-se uma certa estranheza inicial, mas o 
fato é que qdo somamos termos que tendem a zero, talvez a soma ainda possa 
ser finita. Tal como ocorre com 1/10^n. Entretanto, é necessário dizer que 
apenas em algumas sequências a soma converge, precisamente, qdo as séries 
são convergentes. Imagine o seguinte:
2=1,99..., o que essa igualdade significa? Significa que se de 1 somamos 
0,9, 0,09, 0,009, etc..., somando assim cada vez uma quantidade menor, 
completamos 2 inteiros se efetuarmos a soma das infinitas parcelas. Se 
pararmos em qq etapa teremos um pouco menos que 2...)

3) No entanto, pelos
conhecimentos atuais de matemática, isso não ocorre. Muito estranho!
(bom, esses conceitos aparentemente simples envolvem em realidade coisas 
profundas tais como a idéia de ínfimo e a própria construção dos números 
reais, portanto entendo perfeitamente suas dúvidas. Não sei qual a sua 
formação, mas de qq forma, tente ver o livro Análise1 - Do Elon Lages Lima, 
Projeto Euclides-SBM, os capítulos III e IV, talvez ajude um pouco... )]]


- Original Message -
From: Frederico Reis Marques de Brito [EMAIL PROTECTED]
To: [EMAIL PROTECTED]
Sent: Saturday, January 24, 2004 9:47 AM
Subject: Re: [obm-l] Impossibilidade do movimento
 Isto é absolutamente falso. Observe que 1/(10^n)   tende a  0quando
n
 tender a infinito, de forma estritamente decrescente, isto é , se  n  m
=
 1/(10^n)  1/(10^m), mas 0 não é um termo dessa sequência. Posto isto , 
é
 fácil ver que não existe um menor número e que as demais parcelas são
 múltiplas desta...

 Frederico.


 From: Marcelo Augusto Pereira [EMAIL PROTECTED]
 Reply-To: [EMAIL PROTECTED]
 To: [EMAIL PROTECTED]
 Subject: Re: [obm-l] Impossibilidade do movimento
 Date: Fri, 23 Jan 2004 22:10:01 -0200
 
 O fato de essa soma ser calculável(1/9)  não indica que existe um 
número
de
 valor muito pequeno e que esse número seria o valor mínimo que possa
 existir? Assim todos os outros números seriam múltiplos desse menor 
valor
 possível, ou seja, esse número seria algo como um valor quântico. Dessa
 forma, também existiria uma unidade quântica de deslocamento linear, o
que
 faria com que a quantidade de pontos em um segmento de reta não fosse
 infinita e o movimento fosse possível. Se para cada número existisse um
 menor, a soma teria que ser infinita, e o resultado infinito.
 
 - Original Message -
 From: Frederico Reis Marques de Brito [EMAIL PROTECTED]
 To: [EMAIL PROTECTED]
 Sent: Friday, January 23, 2004 9:27 PM
 Subject: RE: [obm-l] Impossibilidade do movimento
 
 
  
   Essencialmente esse problema é ujm dos paradoxos de Zenão, um grego
 antigo
   que usava a idéia de infinito para chegar a conclusões aparentemente
   absurdas, tais como a impossibilidade do movimento, por exemplo. 
Agora
 vou
   dar uma de Dirichlet, o da lista é claro: Pense no seguinte, uma 
soma
de
   infinitas parcelas positivas é sempre infinito, ou não
necessariamente?
 Para
   ajudar nessa resposta, pense em calcular, por exemplo: 1/10 + 1/100 
+
 1/1000
   + ...   . Bom e agora, o que tudo isto tem a ver com sua pergunta?
  
   Espero ter ajudado, apesar dessa resposta meio enigmática, mas acho
que
   assim auxilio mais!
  
   Frederico.
  
   From: Marcelo Augusto Pereira [EMAIL PROTECTED]
   Reply-To: [EMAIL PROTECTED]
   To: [EMAIL PROTECTED]
   Subject: [obm-l] Impossibilidade do movimento
   Date: Fri, 23 Jan 2004 19:05:25 -0200
   
   Entre dois números reais há infinitos outros. Considere um segmento
de
 reta
   com o número 0 assinalado em uma ponta e o número 1 marcado na 
outra.
   Considere também que esse segmento de reta foi representado no chão
com
 um
   risco de um metro de comprimento. Para cada número entre 0 e 1 há 
um
 ponto
   correspondente no segmento de reta e, conseqüentemente, no risco
 marcado
 no
   chão. Como eu consigo caminhar do ponto 0 até o ponto 1, se para
chegar
 de
   0
   até 1 eu tenho que passar por infinitos pontos?
   
  
 
=
   Instruções para entrar na lista, sair da lista e usar a lista em
   http://www.mat.puc-rio.br/~nicolau/olimp/obm-l.html
  
 
=
  
   _
   MSN Hotmail, o maior webmail do Brasil

RE: [obm-l] Paradoxos (era: Impossibilidade do movimento)

2004-01-24 Por tôpico Frederico Reis Marques de Brito
Existem algumas variantes desse paradoxo, tal como : Todo número natural é 
interessante. Todos baseados no Princípio da Boa Ordenação, que é bastante 
intuitivo, e em definições imperfeitas, tais como: dia surpresa e  número 
interessante.
Outro paradoxo  semelhante é: Todos os homens são miseráveis, obtido 
usando-se indução : um homem com R$1,00  apenas é miserável. Se, por outro 
lado, um homem com k reais é miserável, um com (k+1) também o é, posto que 
tem apenas um real a mais que um miserável...

De qq forma, o que é concenso é que os paradoxos são fontes infindáveis de 
discussões, muitas produtivas e  interessantes.

Frederico.

From: Nicolau C. Saldanha [EMAIL PROTECTED]
Reply-To: [EMAIL PROTECTED]
To: [EMAIL PROTECTED]
Subject: [obm-l] Paradoxos (era: Impossibilidade do movimento)
Date: Sat, 24 Jan 2004 16:10:08 -0200
Outro paradoxo conhecido é o da prova surpresa. O professor chega na
primeira aula de um curso de 20 aulas e diz que durante o curso haverá
uma prova surpresa. Um aluno raciocina que a prova não pode ser no último
dia senão não seria surpresa. Mas já que não pode ser no último dia,
se fosse no penúltimo dia também não seria surpresa. Assim a prova não pode
ser em nenhum dos dois últimos dias. Repetindo o raciocínio, a prova não
pode ser um nunhum dos últimos três dias. E repetindo mais vezes, não
pode ser em *nenhum* dia. Ora, no oitavo dia de aula o professor dá uma
prova, e todos são tomados de surpresa.
Há várias explicações diferentes para este paradoxo. Muitas vezes não se 
pode
dizer que uma explicação está certa e outra errada, deve-se apenas 
dizer
que uma explicação é mais interessante, ou mais esclarecedora.

[]s, N.

[]s, N.
=
Instruções para entrar na lista, sair da lista e usar a lista em
http://www.mat.puc-rio.br/~nicolau/olimp/obm-l.html
=
_
MSN Messenger: converse com os seus amigos online.  
http://messenger.msn.com.br

=
Instruções para entrar na lista, sair da lista e usar a lista em
http://www.mat.puc-rio.br/~nicolau/olimp/obm-l.html
=


RE: [obm-l] Impossibilidade do movimento

2004-01-23 Por tôpico Frederico Reis Marques de Brito
Essencialmente esse problema é ujm dos paradoxos de Zenão, um grego antigo 
que usava a idéia de infinito para chegar a conclusões aparentemente 
absurdas, tais como a impossibilidade do movimento, por exemplo. Agora vou 
dar uma de Dirichlet, o da lista é claro: Pense no seguinte, uma soma de 
infinitas parcelas positivas é sempre infinito, ou não necessariamente? Para 
ajudar nessa resposta, pense em calcular, por exemplo: 1/10 + 1/100 + 1/1000 
+ ...   . Bom e agora, o que tudo isto tem a ver com sua pergunta?

Espero ter ajudado, apesar dessa resposta meio enigmática, mas acho que 
assim auxilio mais!

Frederico.

From: Marcelo Augusto Pereira [EMAIL PROTECTED]
Reply-To: [EMAIL PROTECTED]
To: [EMAIL PROTECTED]
Subject: [obm-l] Impossibilidade do movimento
Date: Fri, 23 Jan 2004 19:05:25 -0200
Entre dois números reais há infinitos outros. Considere um segmento de reta
com o número 0 assinalado em uma ponta e o número 1 marcado na outra.
Considere também que esse segmento de reta foi representado no chão com um
risco de um metro de comprimento. Para cada número entre 0 e 1 há um ponto
correspondente no segmento de reta e, conseqüentemente, no risco marcado no
chão. Como eu consigo caminhar do ponto 0 até o ponto 1, se para chegar de 
0
até 1 eu tenho que passar por infinitos pontos?

=
Instruções para entrar na lista, sair da lista e usar a lista em
http://www.mat.puc-rio.br/~nicolau/olimp/obm-l.html
=
_
MSN Hotmail, o maior webmail do Brasil.  http://www.hotmail.com
=
Instruções para entrar na lista, sair da lista e usar a lista em
http://www.mat.puc-rio.br/~nicolau/olimp/obm-l.html
=


[obm-l] RE: [obm-l] Relação de ordem em C

2004-01-22 Por tôpico Frederico Reis Marques de Brito
Bom Rafael. A afirmação , tal como feita, não é verdadeira. È possível 
ordenar os complexos, por exemplo pela ordem do dicionário. correto é 
afirmar que não existe ordem possível em C que preste, ou formalmente 
dizendo, que seja compatível com as operações de soma e produto de 
complexos. ESta demonstração é bem simples e baseia-se, fundamentalmente, no 
fato de que num corpo ordenado  os quadrados devem ser positivos e como bem 
sabemos i^2=-1 em C. A demonstração completa vc pode encontrar em Meu 
Professor de Matemática e Outras Histórias, do Elon. Ed. SBM.

Se vc não encontrar , escreva novamente que lhe mando os detalhes.

Frederico.


From: Rafael [EMAIL PROTECTED]
Reply-To: [EMAIL PROTECTED]
To: [EMAIL PROTECTED]
Subject: [obm-l] Relação de ordem em C
Date: Thu, 22 Jan 2004 04:26:10 -0200
Caros colegas da lista,

Há muito tempo procuro, sem êxito, uma justificativa para algo que se 
aprende logo nas primeiras aulas sobre números complexos: a demonstração 
formal da não existência de ordem no conjunto C.
Por exemplo, sejam z = 2+3i e w = 5+7i, não se pode afirmar que z  w 
ou z  w. No máximo, que z é diferente de w ou alguma comparação específica 
quanto à parte real de um e de outro, tal como em relação à parte 
imaginária.
Parece-me que, do ponto de vista geométrico, é bastante óbvio, visto 
que cada número complexo representa um ponto no plano de Argand-Gauss, não 
se podendo configurar como maior ou menor em relação a outro, mas tão 
somente a sua posição.

Ainda assim, alguém conheceria a demonstração ou algo a respeito?

Fico muito grato desde já por qualquer comentário.

Abraços,

Rafael de A. Sampaio

_
MSN Messenger: converse com os seus amigos online.  
http://messenger.msn.com.br

=
Instruções para entrar na lista, sair da lista e usar a lista em
http://www.mat.puc-rio.br/~nicolau/olimp/obm-l.html
=


Re: [obm-l] AJUDA?????

2003-11-30 Por tôpico Frederico Reis Marques de Brito
Duas retas no espaço são reversas quando não existe um plano que as contenha 
(simultaneamente, é claro). Agora quanto a quantas retas reversas há no 
cubo, sugiro que você de posse da definição, repense sua pergunta...

Frederico.


From: [EMAIL PROTECTED]
Reply-To: [EMAIL PROTECTED]
To: [EMAIL PROTECTED]
Subject: [obm-l] AJUDA?
Date: Sun, 30 Nov 2003 01:06:26 -0200
PODERIA DEFINIR OQUE É RETAS REVERSAS.
QUANTAS RETAS REVERSAS TEM UM CUBO ?
_
Voce quer um iGMail protegido contra vírus e spams?
Clique aqui: http://www.igmailseguro.ig.com.br
Ofertas imperdíveis! Link: http://www.americanas.com.br/ig/
=
Instruções para entrar na lista, sair da lista e usar a lista em
http://www.mat.puc-rio.br/~nicolau/olimp/obm-l.html
=
_
MSN Hotmail, o maior webmail do Brasil.  http://www.hotmail.com
=
Instruções para entrar na lista, sair da lista e usar a lista em
http://www.mat.puc-rio.br/~nicolau/olimp/obm-l.html
=


[obm-l] Re: [obm-l] dúvida 2

2003-11-30 Por tôpico Frederico Reis Marques de Brito
Resposta:  sen(18o) =( raiz(5) -1)/4 .

Para efetuar este cálculo faça o seguinte:  Construa um triângulo isósceles 
com laterais medindo 1 e ângulo do vértice ( aquele oposto a base )  de 36o. 
 Chamemos o vértice oposto a base de A e os vértices da base de B e C. 
Assim,  AB= AC=1  e  BC é a base.  A seguir trace a bissetriz interna  BD. 
Então o triângulo  ABD  também é isósceles ( já que tem dois ângulos 
internos congruentes, de 36o cada ) . Portanto, AD =BD =BC. Use agora que   
os triângulos BAC   e CBD   são semelhantes e que:

BC/CD =  AB/BC=(resolvendo a equalção para achar  BC ) BC =  
(raiz(5)-1)/2  . Baixe a mediatriz  AE da base do triângulo isósceles  ABC . 
Como sabemos ela é também bissetriz do ângulo do vértice A. Assim  o ângulo 
BÂE = 18o.   Tomando o triângulo retângulo  ABE temos que:
sen(18o) =  (BC/2) / AB  =  BC/2  = ( raiz(5) - 1)/4 .   
   CQD.

Frederico.


From: [EMAIL PROTECTED]
Reply-To: [EMAIL PROTECTED]
To: [EMAIL PROTECTED]
Subject: [obm-l] dúvida 2 Date: Sun, 30 Nov 2003 01:00:32 -0200
quanto vale o sen18º como calculo 

_
Voce quer um iGMail protegido contra vírus e spams?
Clique aqui: http://www.igmailseguro.ig.com.br
Ofertas imperdíveis! Link: http://www.americanas.com.br/ig/
=
Instruções para entrar na lista, sair da lista e usar a lista em
http://www.mat.puc-rio.br/~nicolau/olimp/obm-l.html
=
_
MSN Hotmail, o maior webmail do Brasil.  http://www.hotmail.com
=
Instruções para entrar na lista, sair da lista e usar a lista em
http://www.mat.puc-rio.br/~nicolau/olimp/obm-l.html
=


Re: [obm-l] Mais problemas Sobre Grupos

2003-10-20 Por tôpico Frederico Reis Marques de Brito
b) Se   mdc(a,m)=1  =  a  é uma unidade   em Z_m, isto é, existe  b   tal 
que  ab =1( de fato, pelo Teorema de Bezout:  existe b e y inteiros tq  ab 
+mx=1. ).Decorre que  1  pertence a a   =1. Z_m está contido em  a  
. Decorre que a = Z_m.

Depois tento os demais...
Abraços,
Fred.

From: Carlos Maçaranduba [EMAIL PROTECTED]
Reply-To: [EMAIL PROTECTED]
To: [EMAIL PROTECTED]
Subject: [obm-l] Mais problemas Sobre Grupos
Date: Sun, 19 Oct 2003 20:32:10 -0300 (ART)
Eu consegui provar a letra a o resto nao) ai vão:

Seja Z conjunto dos inteiros e x o subgrupo gerado
por x e Zm um grupo qualquer mod m. Mostre que a,b e m
inteiros( m= 2):
a)sendo B = b (mod m) e A = a (mod m) se a divide b
entao, como subgrupos de Zm,
B esta contido em A.(Esse eu consegui provar o
resto nao)
b)sendo A = a (mod m) se mdc(a,m) = 1 , entao A =
Zm.
c)sendo A = a (mod m) e D = d (mod m) se mdc(a,m) = d
, entao A = D.
d) De posse das informacoes acima, determine todos os
subgrupos de (Z36 , +).
e)Mostre  que se (G , *) é um grupo multiplicativo de
ordem 2 entao G é ciclico.
f)Mostre  que se (G , *) é um grupo multiplicativo de
ordem 3 entao G é ciclico.(Sugestao: Sendo G =
{x,a,b}, x o elemento neutrode G, pense sobre o que
poderia ser o elemento ab)
g)Mostre  que se (G , *) é um grupo multiplicativo de
elemento neutro x , mostre que se y^2 =x, para cada y
em G, entao G é abeliano.(Sugestao:Note que y^2 = x
implica que y^-1 = x .Tome 2 elementos quaisquer a e b
em G e comece escrevendo ab = (ab)^-1 = ...)
Yahoo! Mail - o melhor webmail do Brasil
http://mail.yahoo.com.br
=
Instruções para entrar na lista, sair da lista e usar a lista em
http://www.mat.puc-rio.br/~nicolau/olimp/obm-l.html
=
_
MSN Hotmail, o maior webmail do Brasil.  http://www.hotmail.com
=
Instruções para entrar na lista, sair da lista e usar a lista em
http://www.mat.puc-rio.br/~nicolau/olimp/obm-l.html
=


Re: [obm-l] aritmetica

2003-09-11 Por tôpico Frederico Reis Marques de Brito
Chame de  X  o número de passos dados pela 1a pessoa  e de Y o de passos da 
2a. Então:

X = Y + 12 e

0,77X =  0,80Y  , já que ambas percorreram toda a extensão da chácara. Esta 
última eq. é equivalente a
77X= 80Y . Agora resolva o sistema:X=Y+12e   77X= 80Y .  Lembre-se 
de passara resposta para  para Km ...

Abraço,
Frederico.

From: Marcelo [EMAIL PROTECTED]
Reply-To: [EMAIL PROTECTED]
To: [EMAIL PROTECTED]
Subject: [obm-l] aritmetica
Date: Thu, 11 Sep 2003 10:13:12 -0300
A lista OBM,
Tentei de varias formas e nao consegui resolver (montar a solucao),
agradeco antecipadamente a atencao.
Problema: Para medir o comprimento de uma chacara, duas pessoas percorreram
a pe, contando o número de passos que dao. A primeira pessoa da 12 passos
mais do que a segunda. Calcular em quilometros o comprimento da chacara,
sabendo-se que o passo da primeira pessoa mede 0,77 metros e o da segunda
mede 0,80 metros.
A resposta do livro: 0,2464 km

=
Instruções para entrar na lista, sair da lista e usar a lista em
http://www.mat.puc-rio.br/~nicolau/olimp/obm-l.html
=
_
MSN Messenger: converse com os seus amigos online.  
http://messenger.msn.com.br

=
Instruções para entrar na lista, sair da lista e usar a lista em
http://www.mat.puc-rio.br/~nicolau/olimp/obm-l.html
=


Re: [obm-l] Logaritmo Irracional

2003-09-06 Por tôpico Frederico Reis Marques de Brito
Aproveito a oportunidade para acrescentar:
(a)  Mostre que   cos (5 graus ) , cos(10 graus )  e cos (20 graus )  são  
irracionais.
(b)  Podemos generalizar este fato de alguma forma?

Abraços a todos.
( Ah  Cláudio, meu computador teve uma pane geral nesses últimos dias e 
creio não ter recebido a tal correção da enquete, proposta pelo mOrgado, 
você chegou a enviá-la? )

Frederico.


From: Claudio Buffara [EMAIL PROTECTED]
Reply-To: [EMAIL PROTECTED]
To: Lista OBM [EMAIL PROTECTED]
Subject: [obm-l] Logaritmo Irracional
Date: Sat, 06 Sep 2003 08:54:23 -0300
Oi, pessoal:

Eu me lembro de jah ter visto mais de 10 mensagens aqui na lista sobre a
irracionalide de raiz(2), raiz(p), p^(1/n), etc. mas nunca sobre a
irracionalidade de um logaritmo. Assim, aqui vai um problema:
Prove que se N eh um inteiro positivo que nao eh uma potencia de 10, entao
log(N) (logaritmo na base 10) eh irracional.
Dica: a demonstracao eh ateh mais curta do que o caso de raiz(2) e usa
apenas o teorema da unicidade da fatoracao dos numeros inteiros.
Um abraco,
Claudio.
=
Instruções para entrar na lista, sair da lista e usar a lista em
http://www.mat.puc-rio.br/~nicolau/olimp/obm-l.html
=
_
MSN Messenger: converse com os seus amigos online.  
http://messenger.msn.com.br

=
Instruções para entrar na lista, sair da lista e usar a lista em
http://www.mat.puc-rio.br/~nicolau/olimp/obm-l.html
=


[obm-l] Geometria Espacial

2003-08-28 Por tôpico Frederico Reis Marques de Brito
OLá pessoal.

Confesso que nunca tive interesse por geometria espacial. Mas outro dia 
parei a perguntar-me se, similarmente ao que ocorre na geom. plana, há 
alguma fórmula para o angulo interno formado pelas faces de um poliedro 
regular e, neste caso, uma fonte para a demonstracao.

Desde ja agradeco.

Um abraco a todos.

Frederico.

_
MSN Messenger: converse com os seus amigos online.  
http://messenger.msn.com.br

=
Instruções para entrar na lista, sair da lista e usar a lista em
http://www.mat.puc-rio.br/~nicolau/olimp/obm-l.html
=


[obm-l] Re: [obm-l] Questões Divertidas

2003-08-20 Por tôpico Frederico Reis Marques de Brito
Olá Cláudio ( obrigado por ter dado atenção às minhas questoes) e demais 
COLEGAS da lista ( por colegas entendo aqueles que, de alguma forma, estão 
realmenteinteressados na discussão sobre a Matemática e suas belezas 
contribuindo efetivamente para a manutenção e o desenvolvimento da cultura 
matemática neste país.).

Correto. Concordo com as três soluções. Entretanto para o segundo exercício 
podemos dar uma solução mais rápida:
como   a^2b^2c^2 + ab +ac + bc = wabcpara todo  a, b, c positivos , 
fazendo a=b=c=1, temos:
w=4 .
Resta provar que  w=4 satisfaz a condição imposta no enunciado. Para tanto, 
usamos novamente, a desigualdade entere as médias,  MA = MG:
(a^2b^2c^2+ab+ac+bc)/4 =  (a^2b^2c^2abacbc)^{1/4} = (a^4b^4c^4)^{1/4}=abc 
=
(abc)^2+ab+ac+bc = 4abc.

Um grande abraço,
Frederico.

From: Claudio Buffara [EMAIL PROTECTED]
Reply-To: [EMAIL PROTECTED]
To: [EMAIL PROTECTED]
Subject: Re: [obm-l] Questões Divertidas
Date: Tue, 19 Aug 2003 15:08:27 -0300
Oi, Frederico:

Jah que ninguem mais respondeu, aqui vai...

 (1)Mostre  que  tg(x)  + cotg (x) = 2

Supondo que x (mod 2Pi) esteja em (0,Pi/2) U (Pi,3Pi/2), o resultado eh
consequencia de que (tg(x) - 1)^2 = 0.

 (2)  Encontre o maior número real   w tal que wabc =   (abc)^2 
+ ab
 + ac + bc , para todo a,b,c 0 .

O problema equivale a achar o valor minimo de:
F(a,b,c) = abc + 1/a + 1/b + 1/c, com a,b,c  0.

Esse deu um certo trabalho, mas consegui descobrir uma solucao sem usar
calculo.
Media Geometrica = Media Harmonica ==
(abc)^(1/3) = 3/(1/a + 1/b + 1/c) ==
abc = 27/(1/a + 1/b + 1/c)^3 ==
F(a,b,c) = 27/(1/a +1/b + 1/c)^3 + (1/a + 1/b + 1/c),
com igualdade == a = b = c, ou seja:
F(a,b,c) eh minimo quando a = b = c
Mas, fazendo x = 1/a + 1/b + 1/c, teremos:
F(a,b,c) = 27/x^3 + x = 4*[27/x^3 + x/3 + x/3 + x/3]/4
Media Aritmetica = Media Geometrica ==
[27/x^3 + x/3 + x/3 + x/3]/4 = [(27/x^3)*(x/3)*(x/3)*(x/3)]^(1/4) = 1 ==
27/x^3 + x/3 + x/3 + x/3 = 27/x^3 + x = 4,
com igualdade == 27/x^3 = x/3 == x = 3 == 1/a + 1/b + 1/c = 3, ou
seja:
F(a,b,c) eh minimo quando 1/a + 1/b + 1/c = 3.
Assim, o valor minimo de F(a,b,c) eh atingido quando:
a = b = c e 1/a + 1/b + 1/c = 3 == a = b = c = 1
e nesse caso F(a,b,c) = 4
Conclusao: o maior w eh igual a 4.



 (3)  V ou F:O produto da soma de nos reais positivos pela soma de 
seus
 inversos é =  ao quadrado da quantidade de números.

V - consequencia da desigualdade entre a media harmonica e a media
geometrica de numeros positivos.
Um abraco,
Claudio.
=
Instruções para entrar na lista, sair da lista e usar a lista em
http://www.mat.puc-rio.br/~nicolau/olimp/obm-l.html
=
_
MSN Messenger: converse com os seus amigos online.  
http://messenger.msn.com.br

=
Instruções para entrar na lista, sair da lista e usar a lista em
http://www.mat.puc-rio.br/~nicolau/olimp/obm-l.html
=


Re: [obm-l] Primos da forma 2*3*5*...*p + 1

2003-08-20 Por tôpico Frederico Reis Marques de Brito
Segundo Paulo Ribemboim, são problemas em aberto:
Existência de infinitos primos  p   tais que  p# +1  seja primo  e
seja 
composto.
Até a publicação do livro Mistérios e Recordes ( SBM )  (2001), altamente 
recomendado,  o  maior primo na 1a condição conhecido  era  p= 42209, 
descoberto em 99, e que tem apenas 18.241 algarismos...

Este é mais um indício seja, provavelmente, a área mais surprrendente da 
Matemática.

Vou procurar resultados mais recentes... Talvez de lá pra cá tenha se 
encontrado uma resposta parcial ou mesmo completa para as questões. Achando 
algo interessante envio a lista.

Abraços,
Frederico.

  - Original Message -
  From: Claudio Buffara
  [EMAIL PROTECTED]
  To: [EMAIL PROTECTED]
  Sent: Monday, August 18, 2003 12:13 PM
  Subject: [obm-l] Primos da forma 2*3*5*...*p +
  1
 
 
  E serah que existem infinitos primos da forma
  n! + 1?
 
  Por exemplo, n! + 1 eh primo para n = 1, 2, 3,
  11, 27, ...
 
  O teorema de Wilson implica que se n = p - 1,
  com p primo, n! + 1 eh
  divisivel por p. Logo existem infinitos
  compostos da forma n! + 1...
 
  []'s,
  Claudio.
 
 
 
=
  Instruções para entrar na lista, sair da lista
  e usar a lista em
 
 http://www.mat.puc-rio.br/~nicolau/olimp/obm-l.html
 
 
=

 ___
 Desafio AntiZona - Um emocionante desafio de perguntas e respostas que
 te dá um Renault Clio, kits de eletrônicos, computadores, notebooks e
 mochilas. Cadastre-se, participe e concorra!
 www.cade.com.br/antizona
 
=
 Instruções para entrar na lista, sair da lista e usar a lista em
 http://www.mat.puc-rio.br/~nicolau/olimp/obm-l.html
 
=



=
Instruções para entrar na lista, sair da lista e usar a lista em
http://www.mat.puc-rio.br/~nicolau/olimp/obm-l.html
=
_
MSN Hotmail, o maior webmail do Brasil.  http://www.hotmail.com
=
Instruções para entrar na lista, sair da lista e usar a lista em
http://www.mat.puc-rio.br/~nicolau/olimp/obm-l.html
=


[obm-l] Re: [obm-l] Re: [obm-l] Questões Divertidas

2003-08-19 Por tôpico Frederico Reis Marques de Brito
Olá a todos. Realmente, este fato só é válido nos quadrantes ímpares. Achei 
que já tinha mandado esta errata para a lista mas pelo visto devo ter 
respondido a algum email pessoal. De qq forma, obrigado Morgado.
Abraços,
Frederico.


From: Augusto Cesar de Oliveira Morgado [EMAIL PROTECTED]
Reply-To: [EMAIL PROTECTED]
To: [EMAIL PROTECTED]
Subject: Re: [obm-l] Re: [obm-l] Questões Divertidas
Date: Mon, 18 Aug 2003 18:54:42 -0300 (EST)
Epa, isso so eh verdade no primeiro e no terceiro quadrantes.
Morgado
Em Mon, 18 Aug 2003 15:33:52 -0400, Aleandre Augusto da Rocha 
[EMAIL PROTECTED] disse:


 - Original Message -
 From: Frederico Reis Marques de Brito [EMAIL PROTECTED]
 To: [EMAIL PROTECTED]
 Sent: Saturday, August 16, 2003 9:47 AM
 Subject: [obm-l] Questões Divertidas


 
  Caros colegas. As questões que se seguem são todas simples, desde que
  pensemos na coisa certa. Como gostei delas resolvi partilhá-las com 
vcs:
 
  (1)Mostre  que  tg(x)  + cotg (x) = 2  .
 

 tg(x) + cotg(x) = 2
 sen(x)/cos(x) + cos(x)/sen(x) = 2
 (sen^2(x) + cos^2(x))/sen(x)cos(x) = 2
 1 = 2sen(x)cos(x)
 1=sen(2x)

 
  Abraços,
  Frederico.

 -Auggy


 
=
 Instruções para entrar na lista, sair da lista e usar a lista em
 http://www.mat.puc-rio.br/~nicolau/olimp/obm-l.html
 
=



=
Instruções para entrar na lista, sair da lista e usar a lista em
http://www.mat.puc-rio.br/~nicolau/olimp/obm-l.html
=
_
MSN Messenger: converse com os seus amigos online.  
http://messenger.msn.com.br

=
Instruções para entrar na lista, sair da lista e usar a lista em
http://www.mat.puc-rio.br/~nicolau/olimp/obm-l.html
=


[obm-l] Re: [obm-l] Re: [obm-l] Questões Divertidas

2003-08-19 Por tôpico Frederico Reis Marques de Brito
Olá Alexandre. É bastante perigoso  tentarmos demonstrar uma 
desigualdadeou mesmo uma igualdade mechendo nos dois membros da mesma. Ao 
chegarmos numa conclusão verdadeira, como a que vc chegou, é necessário 
checar se os passos são treversíveis, pois afinal de contas vc partiu da 
hipótese.  Além disso, senx . cos x pode ser negativo , o que ocorre 
precisamente nos quadrantes pares, onde a tese é falsa, mas  vc não se deu 
conta disso.
Uma sugestão, olhe os outros problemas e tente achar a idéia comum aos três;

Um abraço,
Frederico.

From: Aleandre Augusto da Rocha [EMAIL PROTECTED]
Reply-To: [EMAIL PROTECTED]
To: [EMAIL PROTECTED]
Subject: [obm-l] Re: [obm-l] Questões Divertidas
Date: Mon, 18 Aug 2003 15:33:52 -0400
- Original Message -
From: Frederico Reis Marques de Brito [EMAIL PROTECTED]
To: [EMAIL PROTECTED]
Sent: Saturday, August 16, 2003 9:47 AM
Subject: [obm-l] Questões Divertidas

 Caros colegas. As questões que se seguem são todas simples, desde que
 pensemos na coisa certa. Como gostei delas resolvi partilhá-las com 
vcs:

 (1)Mostre  que  tg(x)  + cotg (x) = 2  .


tg(x) + cotg(x) = 2
sen(x)/cos(x) + cos(x)/sen(x) = 2
(sen^2(x) + cos^2(x))/sen(x)cos(x) = 2
1 = 2sen(x)cos(x)
1=sen(2x)

 Abraços,
 Frederico.
-Auggy

=
Instruções para entrar na lista, sair da lista e usar a lista em
http://www.mat.puc-rio.br/~nicolau/olimp/obm-l.html
=
_
MSN Messenger: converse com os seus amigos online.  
http://messenger.msn.com.br

=
Instruções para entrar na lista, sair da lista e usar a lista em
http://www.mat.puc-rio.br/~nicolau/olimp/obm-l.html
=


Re: [obm-l] Infinitos Primos.

2003-08-17 Por tôpico Frederico Reis Marques de Brito
Eu ainda não havia visto a prova basdeada nos ciclotômicos. O Dirichlet ( o 
da lista)  enviou um endereço contendo artigo sobre isto. Lerei-o. A prova 
que conheço é a resposta oficial de uma Olimpíada Russa, mas é muito longa ( 
3 páginas ) e braçal. Talvez tenha a idéia geral da dos ciclotômicos...

Obrigado.
Abraços a todos.
Frederico.
From: Cláudio \(Prática\) [EMAIL PROTECTED]
Reply-To: [EMAIL PROTECTED]
To: [EMAIL PROTECTED]
Subject: Re: [obm-l] Infinitos Primos.
Date: Sat, 16 Aug 2003 13:30:39 -0300
A demonstração clássica do caso 4k + 1 leva em conta que, para todo primo p
e todo inteiro N, se p divide N^2 + 1, então p = 2 ou p é da forma 4k + 1.
Sejam p1, p2, ..., pn todos os primos da forma 4k + 1.

Então, os fatores primos de (2*p1*p2*...*pn)^2 + 1 são todos da forma 4k + 
3
==
contradição.

*

Existe uma demonstração da infinitude dos primos da forma Nk + 1 para
qualquer N que usa polinômios ciclotômicos. É essa que você conhece?
Um abraço,
Claudio.
- Original Message -
From: Frederico Reis Marques de Brito [EMAIL PROTECTED]
To: [EMAIL PROTECTED]
Sent: Saturday, August 16, 2003 10:56 AM
Subject: [obm-l] Infinitos Primos.


 Pessoal, como todos devem saber dada em toda  progressão aritmética
em
 que a razão e o termo inicial são coprimos existe uma quantidade 
infinita
de
 primos. Este é o conhecido Teorema de Dirichlet, cuja demonstração  é
 bastante complexa. Alguns casos especiais são facilmente demonstrados 
como
 4k+3   ou   6k+5 e já foram tratados nesta lista.  Proponho então a
 demonstração  dos seguintes casos:
 10K +1e4k +1 , especialmente o primeiro deles, poias embora
conheça
 as demonstrações gostaria de obter provas mais simples das de que tenho
 conhecimento.
 Se alguém tiver uma idéia, por favor escreva-me.

 Abraços,
 Frederico.

 _
 MSN Messenger: converse com os seus amigos online.
 http://messenger.msn.com.br

 
=
 Instruções para entrar na lista, sair da lista e usar a lista em
 http://www.mat.puc-rio.br/~nicolau/olimp/obm-l.html
 
=

=
Instruções para entrar na lista, sair da lista e usar a lista em
http://www.mat.puc-rio.br/~nicolau/olimp/obm-l.html
=
_
MSN Messenger: converse com os seus amigos online.  
http://messenger.msn.com.br

=
Instruções para entrar na lista, sair da lista e usar a lista em
http://www.mat.puc-rio.br/~nicolau/olimp/obm-l.html
=


[obm-l] Questões Divertidas

2003-08-16 Por tôpico Frederico Reis Marques de Brito
Caros colegas. As questões que se seguem são todas simples, desde que 
pensemos na coisa certa. Como gostei delas resolvi partilhá-las com vcs:

(1)Mostre  que  tg(x)  + cotg (x) = 2  .

(2)  Encontre o maior número real   w tal que wabc =   (abc)^2 + ab 
+ ac + bc , para todo a,b,c 0 .

(3)  V ou F:O produto da soma de nos reais positivos pela soma de seus 
inversos é =  ao quadrado da quantidade de números.

Abraços,
Frederico.
_
MSN Messenger: converse com os seus amigos online.  
http://messenger.msn.com.br

=
Instruções para entrar na lista, sair da lista e usar a lista em
http://www.mat.puc-rio.br/~nicolau/olimp/obm-l.html
=


[obm-l] Infinitos Primos.

2003-08-16 Por tôpico Frederico Reis Marques de Brito


Pessoal, como todos devem saber dada em toda  progressão aritméticaem 
que a razão e o termo inicial são coprimos existe uma quantidade infinita de 
primos. Este é o conhecido Teorema de Dirichlet, cuja demonstração  é 
bastante complexa. Alguns casos especiais são facilmente demonstrados como  
4k+3   ou   6k+5 e já foram tratados nesta lista.  Proponho então a 
demonstração  dos seguintes casos:
10K +1e4k +1 , especialmente o primeiro deles, poias embora conheça 
as demonstrações gostaria de obter provas mais simples das de que tenho 
conhecimento.
Se alguém tiver uma idéia, por favor escreva-me.

Abraços,
Frederico.
_
MSN Messenger: converse com os seus amigos online.  
http://messenger.msn.com.br

=
Instruções para entrar na lista, sair da lista e usar a lista em
http://www.mat.puc-rio.br/~nicolau/olimp/obm-l.html
=


Re: [obm-l] ENQUETE - BELEZA MATEMATICA

2003-08-14 Por tôpico Frederico Reis Marques de Brito
Exatamente Cláudio, o Princípio de Dirichlet tb é conhecido como Princípio 
da Casa dos Pombos ou das gavetas.

O exemplo do monge é muito bom. Coloquei-o certa vez numa prova de cálculo 
I. Os alunos acharam bacana. E quanto ao TNP a prova não é simples 
realmente, mas a tentação de mostrar aos alunos a relação entre ln  e os nos 
primos, destas relações absolutaamente inesperadas, é forte.

Aproveito para colocar mais alguns resultados e dizer que, a medida que leio 
as respostas dos nossos colegas a sua enquete, fico cada vez mais perplexo, 
pois raramente discordo de algum, o que me alegra por demonstrar que, 
convenhamos, a Matemática é linda demais.

(6) Esse é simples e bonitinho demais: Existem desertos de primos tão 
grandes qto se queira, isto é, formalmente: dado N natural, existe uma 
sequência de N inteiros consecutivos compostos.

(7) A demonstração de que os números transcendentes são não-enumeráveis.

(8) A solução da eq: 2^x = x^2  . ( acho que se encontra isto em : Meu 
Professor de Matemática... , do Elon. SBM )

(9) A demonstração de que  C  não é um corpo ordenado, pela simplicidade. ( 
Aqui vale dizer que não é necessário o emprego dos termos técnicos, como 
corpo, por exemplo... )

(10) A relação de Euler para poliedros, que, pecaminosamente havia me 
esquecido.

Abraços,

Frederico.

From: Claudio Buffara [EMAIL PROTECTED]
Reply-To: [EMAIL PROTECTED]
To: [EMAIL PROTECTED]
Subject: Re: [obm-l] ENQUETE - BELEZA MATEMATICA
Date: Sat, 09 Aug 2003 20:34:04 -0300
on 09.08.03 18:39, Frederico Reis Marques de Brito at 
[EMAIL PROTECTED]
wrote:

 1) Acho que esse será praticamente unânime: Teorema de Euclides sobre a
 exist~encia de  infinitos primos.

 2) Teorema de Bezout sobre MDC: O máximo dvisor comum de dois inteiros é 
uma
 comb. linear inteira ( em realidade a menor positiva ) desses números ,
 pelas várias aplicações deste na Teoria dos Números.

 3) O Princípio de Dirichlet, pela potência .

 4) O posto-linha = posto-coluna. Não sei mais sempre achei este 
resultado
 muito inusitado, já que uma matriz nada mais é que um amontoado de
 números...

 5) Teorema do Valor Intermediário ( Acho que podemos abordar este tema 
no
 ensino médio... )

 Como o Morgado, pensarei um pouco mais antes de enviar outros 5. (  A
 propósito é tentador citar o Teorema dos Números Primos, mas acho que 
esse
 tema não seria acessível. Fica um voto de louvor então!)

 Frederico.

Oi, Frederico.

O principio de Dirichlet a que voce se refere eh o das casas de pombos?

O TNP eh um pouco avancado demais (mesmo com uma demonstracao elementar),
mas aquelas desigualdades de Chebichev sao aceitaveis, assim como o
postulado de Bertrand, que penso seriamente em botar na minha lista.
Pra mim, o TVI (e qualquer resultado que dependa do axioma do supremo) foi
um caso mais dificil de decidir, mas como tem aquele probleminha do monge
subindo e descendo a montanha, acho que ele tambem eh aceitavel. E, afinal
de contas, tem um volume da colecao do Iezzi que trata de limites, 
derivadas
e integrais...

Um abraco,
Claudio.
=
Instruções para entrar na lista, sair da lista e usar a lista em
http://www.mat.puc-rio.br/~nicolau/olimp/obm-l.html
=
_
MSN Hotmail, o maior webmail do Brasil.  http://www.hotmail.com
=
Instruções para entrar na lista, sair da lista e usar a lista em
http://www.mat.puc-rio.br/~nicolau/olimp/obm-l.html
=


Re: [obm-l] ENQUETE - BELEZA MATEMATICA

2003-08-14 Por tôpico Frederico Reis Marques de Brito
Cláudio:

 A Matemática é a única ciência em que uma geração não poe abaixo o que a 
anterior construiu. 

As idéias matemáticas belas são permanentes e  a sua enquete em nada remetia 
a que enviássemos apenas teoremas modernosos, ademais a genialidade dos 
Matemáticos da Carochinha  parece não ter mais espaço nos últimos anos ...

Perplexo, com seu comentário,

Frederico.


From: Johann Peter Gustav Lejeune Dirichlet 
[EMAIL PROTECTED]
Reply-To: [EMAIL PROTECTED]
To: [EMAIL PROTECTED]
Subject: Re: [obm-l] ENQUETE - BELEZA MATEMATICA
Date: Tue, 12 Aug 2003 15:15:20 -0300 (ART)

Ah e,e??Veja o primeiro capitulo do
Proofs from THE BOOK.Ai ce vai ver como topologia
e ultrapassado
 Claudio Buffara wrote:
  Estou extremamente decepcionado com as listas
 de problemas supostamente
  bonitos que foram enviadas pra lista ateh o
 presente momento. Imaginem soh -
  teorema do valor intermediario, secoes
 conicas, poliedros regulares,
  conjuntos enumeraveis. Onde voces estao com a
 cabeca? Isso tudo eh
  matematica do tempo da carochinha. Infinitude
 dos primos? Isso eh tao velho
  que ja devia ter sido revogado!

 O Noga Alon conta que fizeram esta pergunta
 para ele uma vez que ele
 começou explicando a prova de Euclides de que
 há infinitos primos
 em um programa de televisão, eu acho:

 And today, are there still infinitely many
 primes?

 E sem sair do clima, deem uma olhada em

http://qsilver.queensu.ca/~phil158d/intro/montyh3.htm

 Eu deveria ter visto isso antes de escrever o
 meu artigo da Eureka!

 []s, N.

=
 Instruções para entrar na lista, sair da lista
 e usar a lista em

http://www.mat.puc-rio.br/~nicolau/olimp/obm-l.html

=
___
Conheça o novo Cadê? - Mais rápido, mais fácil e mais preciso.
Toda a web, 42 milhões de páginas brasileiras e nova busca por imagens!
http://www.cade.com.br
=
Instruções para entrar na lista, sair da lista e usar a lista em
http://www.mat.puc-rio.br/~nicolau/olimp/obm-l.html
=
_
MSN Hotmail, o maior webmail do Brasil.  http://www.hotmail.com
=
Instruções para entrar na lista, sair da lista e usar a lista em
http://www.mat.puc-rio.br/~nicolau/olimp/obm-l.html
=


Re: [obm-l] ENQUETE - BELEZA MATEMATICA

2003-08-14 Por tôpico Frederico Reis Marques de Brito
1) Acho que esse será praticamente unânime: Teorema de Euclides sobre a 
exist~encia de  infinitos primos.

2) Teorema de Bezout sobre MDC: O máximo dvisor comum de dois inteiros é uma 
comb. linear inteira ( em realidade a menor positiva ) desses números , 
pelas várias aplicações deste na Teoria dos Números.

3) O Princípio de Dirichlet, pela potência .

4) O posto-linha = posto-coluna. Não sei mais sempre achei este resultado 
muito inusitado, já que uma matriz nada mais é que um amontoado de 
números...

5) Teorema do Valor Intermediário ( Acho que podemos abordar este tema no 
ensino médio... )

Como o Morgado, pensarei um pouco mais antes de enviar outros 5. (  A 
propósito é tentador citar o Teorema dos Números Primos, mas acho que esse 
tema não seria acessível. Fica um voto de louvor então!)

Frederico.

From: Claudio Buffara [EMAIL PROTECTED]
Reply-To: [EMAIL PROTECTED]
To: Lista OBM [EMAIL PROTECTED]
CC: Claudio Buffara [EMAIL PROTECTED]
Subject: [obm-l] ENQUETE - BELEZA MATEMATICA
Date: Sat, 09 Aug 2003 10:24:26 -0300
Caros colegas da lista:

Gostaria de contar com sua participacao numa enquete sobre beleza
matematica.
O que eu precisao eh que cada um de voces me envie uma lista contendo algo
como 5 a 10 problemas/teoremas que voces consideram os mais bonitos e cujas
solucoes/demonstracoes sao as mais elegantes e/ou inusitadas e/ou
engenhosas. Nao precisa incluir a solucao/demonstracao, apenas o enunciado.
No entanto, se voce tiver em mente uma solucao/demonstracao especifica
(entre varias existentes) nao deixe de mencionar pelo menos o metodo
utilizado.
A unica restricao eh que estes resultados devem ser de um nivel acessivel a
um aluno normal de 2o. grau (ou seja, o Ultimo Teorema de Fermat e o 
Porisma
de Poncelet estao fora, mas o caso n = 4 do UTF e a versao para triangulos
do Porisma poderiam ser incluidos).

Importante: os resultados devem ser acessiveis a um aluno normal de 2o.
grau, mas nao necessariamente fazer parte do curriculo normal do 2o. grau.
Tambem nao precisa responder hoje ou amanha ou mesmo na semana que vem. 
Acho
que vale a pena pensar por um tempo e consultar a literatura - as vezes 
pode
ter um resultado belissimo do qual voce simplesmente se esqueceu por nao
encontra-lo ha muito tempo. As Eurekas sao uma otima referencia. O Proofs
from the Book tambem, apesar de nem tudo lah ter nivel de 2o. grau.

Se houver um numero suficiente de respostas, eu me comprometo a publicar 
uma
compilacao dos problemas e teoremas mais votados.

Desde jah a gradeco o interesse de quem quiser participar.

Um abraco,
Claudio.
=
Instruções para entrar na lista, sair da lista e usar a lista em
http://www.mat.puc-rio.br/~nicolau/olimp/obm-l.html
=
_
MSN Hotmail, o maior webmail do Brasil.  http://www.hotmail.com
=
Instruções para entrar na lista, sair da lista e usar a lista em
http://www.mat.puc-rio.br/~nicolau/olimp/obm-l.html
=


Re: [obm-l] ENQUETE - BELEZA MATEMATICA

2003-08-14 Por tôpico Frederico Reis Marques de Brito
Prezado Cláudio e demais colegas da lista,
Quero desculpar-me por ter interpretado mal sua mensagem. Em realidade 
cometo um erro frequente de ler as mensagens na ordem em que elas estão na 
minha caixa, desta forma leio as mais recentes primeiro. Desta forma li a 
resposta de um membro da lista sobre sua mensagem original, mas que continha 
apenas o primeiro trecho de sua mensagem, o que me levou a uma interpretação 
incorreta e me deixou meio perplexo, minha impressão sobre o Cláudio foi 
sempre bastante positiva. Com este embróglio, prometo ler os e-mails na 
ordem correta. Aproveito para solidariezar-me com o Cláudio e outros que tb 
já não aguentam mais ficar lendo pseudo-paradoxos horripilantes e 
quebra-cabeças supostamente lógicos...

Um abraço a todos.

Frederico.


From: Claudio Buffara [EMAIL PROTECTED]
Reply-To: [EMAIL PROTECTED]
To: Lista OBM [EMAIL PROTECTED]
Subject: [obm-l] ENQUETE - BELEZA MATEMATICA
Date: Wed, 13 Aug 2003 09:44:57 -0300
Caros colegas:

Foi com uma mistura de surpresa e horror que eu constatei que alguns
participantes da lista interpretaram literalmente o que eu escrevi em minha
ultima mensagem sobre o assunto em epigrafe, do qual alias sou 
patrocinador.
O que me entristece eh que um deles ja havia me causado uma impressao muito
positiva por sua disposicao de tirar duvidas e de mostrar um pouco da 
beleza
que existe na matematica.

Sendo assim, o que me resta fazer eh esclarecer que aquela mensagem tinha
dois objetivos:
1. Fazer rir;
2. Protestar contra o excesso de mensagens sem nexo, pegadinhas, 
problemas
irrelevantes e mal-formulados e, mais geralmente, spam, que certos
participantes insistem em enviar para a lista.

Espero ter obtido um modico de sucesso em cada um deles.

E para que nao haja duvidas:
NAO! EU NAO ACHO QUE O TEOREMA SOBRE A INFINITUDE DOS PRIMOS DEVA SER
REVOGADO! E TAMBEM NAO ACHO QUE TEOREMAS PROVADOS NO TEMPO DA CAROCHINHA OU
ANTES DISSO DEVAM SER NEGLIGENCIADOS - MUITO PELO CONTRARIO, ESTES SAO EM
GERAL OS MAIS BONITOS!
No mais, ainda estou pensando no conteudo da minha lista de verdade.

Este triste episodio me fez lembrar de um ex-presidente do nosso pais de
quem se dizia que ria 3 vezes de cada piada: a primeira, quando ouvia; a
segunda, quando explicavam; e a terceira, quando entendia.
Alias, dizem que esse mesmo governante em certa ocasiao envolveu-se em
acalorada discussao com seus ministros sobre o ponto de ebulicao da agua.
Ele insistia que a agua fervia a 90 graus. Por mais de 1 hora, os ministros
tentaram de todas as formas convence-lo que nao, que a agua fervia a 100
graus. Fracassaram. Entretanto, alguns dias depois, em nova reuniao, o
presidente dirigiu-se aos ministros e disse: Senhores: eu lhes devo
desculpas. Os senhores estao corretissimos. A agua de fato ferve a 100
graus. Eu me confundi. O que ferve a 90 graus eh o angulo reto.
Por fim, quero falar um pouco da mensagem do Nicolau. O episodio ocorrido
com o Noga Alon mostra que a vida real pode ser pelo menos tao absurda
quanto a ficcao. Alem disso, noto, com satisfacao, que a discussao gerada 
em
torno do problema das tres portas resultou em pelo menos dois participantes
da lista se convencendo de que a atitude correta eh trocar de porta. Eu
chamo isso de progresso. Um passo a mais para longe do obscurantismo!

Um grande abraco a todos,
Claudio.
=
Instruções para entrar na lista, sair da lista e usar a lista em
http://www.mat.puc-rio.br/~nicolau/olimp/obm-l.html
=
_
MSN Messenger: converse com os seus amigos online.  
http://messenger.msn.com.br

=
Instruções para entrar na lista, sair da lista e usar a lista em
http://www.mat.puc-rio.br/~nicolau/olimp/obm-l.html
=


[obm-l] Não é piada!!!

2003-08-14 Por tôpico Frederico Reis Marques de Brito
Outra dia, quer dizer noite, assistindo o prigramo do tal senhor gordo, em 
que ele lia alguns hilariantes erros dos alunos nas provas de redação, 
comecei a lembrar de algumas pérolas matemáticas dadas pelos alunos das mais 
diversas engenharias e afins nas provas de cálculo. Então só para distrair 
um pouco, listei algumas *( fatos verídicos ocorridos em minhas classes ou 
de amigos professores .) :

(1)   Derivar   sen xusando a  regra do produto , algo do tipo:
cos . x  +  sen . 1 

(2)   Usar que todas as funções matemáticas são lineares. Desta forma:  
sen(x+y) = sen x + sen y ,  e^{x-y} = e^x - e^y ,  1/ (a+b) = 1/a + 1/b  
etc...

(3)  Achar que uma reta não é uma curva...

(4) lim_{  x - +  infinito } (x^2 + 3x + 1 ) =  infinito^2 + 3infinito + 1 
...

(5)  i = \sqrt{ - 0 } ... ( Observe a necessidade do - . )

(6)  Essa é de uma aluna de Matemática para a professora de Geometria Plana: 
Ah professora, eu não sei pra quê tudo isto... Eu vou ser professora de 
Matemática não de Geometria.!

Aproveito para tb puxar a orelha dos autores de livros de Cálculo I, alguns 
perlo menos,  não sei bem com qwual objetivo, talvez a simplificação 
extrema,   eles  consideram que, por exemplo,  não existe
lim_{ x - 0}   \sqrt(x) . ...

Abraços,
Frederico.
_
MSN Hotmail, o maior webmail do Brasil.  http://www.hotmail.com
=
Instruções para entrar na lista, sair da lista e usar a lista em
http://www.mat.puc-rio.br/~nicolau/olimp/obm-l.html
=


Re: [obm-l] ENQUETE - BELEZA MATEMATICA

2003-08-10 Por tôpico Frederico Reis Marques de Brito
Chamem um exorcista, o Nicolau está possuído pelo espírito de Fermat... Só 
espero que ele não demore 350 anos até enviar a sua solução...

Brincadeira... Agora escrevendo seriamente. Tb não sei que problema dos 
pontos é esse e muito menos a solução de Fermat para o mesmo.

Morgado, salve-nos...

Abraços,
Frederico.

From: Nicolau C. Saldanha [EMAIL PROTECTED]
Reply-To: [EMAIL PROTECTED]
To: [EMAIL PROTECTED]
Subject: Re: [obm-l] ENQUETE - BELEZA MATEMATICA
Date: Sat, 9 Aug 2003 18:54:19 -0300
On Sat, Aug 09, 2003 at 11:08:53AM -0300, A. C. Morgado wrote:

 4) O problema dos pontos. Pela beleza da solução de Fermat.

Perdão pela minha ignorância, mas o que é o problema dos pontos?

 5) São apenas 5 os poliedros regulares. (Outro que, em geral, não nos
 damos conta de quão surpreendente ele é.)
Aqui é preciso demonstrar não só que não existem outros poliedros
regulares mas também que os cinco poliedros que nós conhecemos
de fato existem. Uma demonstração é pura e simplesmente dar coordenadas
em R^3 para os vértices mas esta demonstração de certa forma é 
insatisfatória
pois é caso a caso. O que seria interessante é demonstrar de forma geral
que se o ângulo interno de um polígono regular de n lados é menor
do que 2 pi/m então podemos juntar m polígonos regulares de n lados
ao redor de cada vértice e completar um poliedro regular.

Eu tenho uma demonstração notável deste fato mas este e-mail é pequeno
demais para ela. ;-)
[]s, N.
=
Instruções para entrar na lista, sair da lista e usar a lista em
http://www.mat.puc-rio.br/~nicolau/olimp/obm-l.html
=
_
MSN Hotmail, o maior webmail do Brasil.  http://www.hotmail.com
=
Instruções para entrar na lista, sair da lista e usar a lista em
http://www.mat.puc-rio.br/~nicolau/olimp/obm-l.html
=


[obm-l] V ou F Analítico.

2003-08-10 Por tôpico Frederico Reis Marques de Brito
Bom pessoal, é o seguinte.

Seja  a_n , n e IN , uma sequência de  reais  e suponha que   a_n - a . 
Verdadeiro ou Falso:

(a_1 +  a_2  +  ... + a_n ) / n -  a.

Infelizmente não sei como indicar um somatório ...

Abraços,

Frederico.

_
MSN Messenger: converse com os seus amigos online.  
http://messenger.msn.com.br

=
Instruções para entrar na lista, sair da lista e usar a lista em
http://www.mat.puc-rio.br/~nicolau/olimp/obm-l.html
=


Re: [obm-l] primos...

2003-07-31 Por tôpico Frederico Reis Marques de Brito
Exceto 2  todo primo é congruente a 1 ou 3 mod 4. Observe que produto de 
inteiros congruentes a 1 mod 4  tb é congruente a 1 mod 4. Em seguida, 
suponha, por absurdo , que   p1 , p2 , ..., pk  , sejam todos os primos 
congruentes a 3 mod 4 maiores que  3 , e tomeA = 4p1 p2 ... pk  + 3 . A 
não pode serr primo, pois é congruente a 3 mod 4   e maior que todos os 
primos desta forma, por hipótese de absurdo. Mas pelo Teor. Fund. Aritmética 
ele tem algum fator primo, e pelo que dissemos antes, deve ter um fator 
primo congruente a 3 mod 4. Logo este fator deve ser algum dos pi´s, 
digamosd p1. Mas se p1 divide a , decorre que  p1   divide 3. Absurdo.
Um abraço.
Frederico.


From: [EMAIL PROTECTED]
Reply-To: [EMAIL PROTECTED]
To: [EMAIL PROTECTED]
Subject: [obm-l] primos...
Date: Wed, 30 Jul 2003 02:53:21 EDT
Prove que existem infinitos primos congruos a 3 módulo 4..
Um abraço,
  Crom
_
MSN Messenger: converse com os seus amigos online.  
http://messenger.msn.com.br

=
Instruções para entrar na lista, sair da lista e usar a lista em
http://www.mat.puc-rio.br/~nicolau/olimp/obm-l.html
=


Re: [obm-l] Ajuda

2003-07-31 Por tôpico Frederico Reis Marques de Brito
1)   (113k + 7 )/(k+1)= (113k +113  -106)/(k+1) = 113   - 106/(k+1) será 
inteiro se, e só se,   (k+1) for um divisor de 106= 2 x 53 . Como 106 tem  4 
divisores positivos, existem  8  de  k   satisfazendo o enunciado.

2) tenho que pensar um pouco + .

Frederico.

From: Fabio Bernardo [EMAIL PROTECTED]
Reply-To: [EMAIL PROTECTED]
To: obm [EMAIL PROTECTED]
Subject: [obm-l] Ajuda
Date: Thu, 31 Jul 2003 21:50:11 -0300
Caros colegas, uns alunos me pediram essas duas questões e eu ainda não 
consegui resolvê-las.
Se alguém puder, me ajude por favor.

1) Quantos valores de K, inteiro, existem, tais que, (113k+7)/(k+1) é um 
número inteiro?

2) Para quantos valores reais do número a a equação x^2+ax+6a=0 possui 
somente raízes inteiras?

Desde já agradeço.
_
MSN Messenger: converse com os seus amigos online.  
http://messenger.msn.com.br

=
Instruções para entrar na lista, sair da lista e usar a lista em
http://www.mat.puc-rio.br/~nicolau/olimp/obm-l.html
=


Re: [obm-l] Matrizes (ajuda)

2003-07-23 Por tôpico Frederico Reis Marques de Brito
Basta multiplicar os dois membros da eq. AX=Bpor A^{-1}, pela esquerda ( 
lembre-se de que o produto de matrizes, em geral, é não-comutativo!!! ). 
Dessa forma:

X=A^{-1}. B  .

Frederico.

From: [EMAIL PROTECTED]
Reply-To: [EMAIL PROTECTED]
To: [EMAIL PROTECTED]
Subject: [obm-l] Matrizes (ajuda)
Date: Wed, 23 Jul 2003 08:58:34 -0300
Olá pessoal,
Não estou conseguindo resolver essa questão de matrizes:
Sabendo que AX = B, Anxn, e B diferente de zero, tal que A^(-1) existe. 
Calcule X.

Desde já, grato,
Moreira


_
Quer ajudar o Brasil e não sabe como?
AjudaBrasil: http://www.ajudabrasil.org/mail.html.
=
Instruções para entrar na lista, sair da lista e usar a lista em
http://www.mat.puc-rio.br/~nicolau/olimp/obm-l.html
=
_
MSN Hotmail, o maior webmail do Brasil.  http://www.hotmail.com
=
Instruções para entrar na lista, sair da lista e usar a lista em
http://www.mat.puc-rio.br/~nicolau/olimp/obm-l.html
=


[obm-l] Re: [obm-l] Re: [obm-l] Re: [obm-l] Re: [obm-l] Re: [obm-l] Prova por indução finita

2003-07-22 Por tôpico Frederico Reis Marques de Brito
Henrique, você fez exatamente o que eu temia que houvesse feito. No processo 
de indução , nós assumimos que o resultado é válido para um vcerto número 
natural, k, e devemos PROVAR que esse resultado também é válido para  o 
próximo número natural (k+1). Assim, quando  assumimos que
k!  2^k , estamos supondo que isto ocorra para um valor de k , o famoso 
fixo porém arbitrário e não podemos trocar  k por  k+1.  Este foi seu 1o 
erro. O segundo é que não  podemos demonstrar uma igualdade ou uma 
desigualdade , mechendo nos dois membros e chegando ao final numa 
igualdade obviamente verdadeira, tipo 0=0  ou  1 0, por exemplo.  Para se 
convencer disso, veja este exemplo simplório:

Vamos provar que  -1 =1 .  Elevando ao quadrado dos dois lados: 1 = 1 , 
que é uma afirmação obviamente verdadeira, embora nossa tese não seja.

Grande abraço.(  E não se envergonhe, qq dúvida, escreva novamente!!! )

Frederico.


From: Henrique Patrício Sant'Anna Branco [EMAIL PROTECTED]
Reply-To: [EMAIL PROTECTED]
To: [EMAIL PROTECTED]
Subject: [obm-l] Re: [obm-l] Re: [obm-l] Re: [obm-l] Re: [obm-l] Prova por 
indução finita
Date: Tue, 22 Jul 2003 02:40:52 -0300

 Desta vez fui eu que não entendi sua dúvida. De qq forma pela 
experiência
 que tenho em sala de aula imaginoque o seguinte te ajude:

 supomos que  k!  2^k  . Portanto, desde que  k+1 é positivo, podemos
 multiplicar essa desigualdade dos dois lados por (k+1)  =  (k+1). 
k!

 (k+1). 2^k   = (k+1)!  (k+1) . 2^k   2 . 2^k , pois   k+1  2 .
Segue
 que  (k+1)!  2^{k+1} .

 Ajudou? Se não, pode escrever novamente, mas explique-me melhor sua
dúvida.

Ah, certo, você multiplicou os dois lados por (k+1).
O que eu tinha pensado é o seguinte:
Sabemos que k!  2^k nas condições do enunciado. Então, temos que (k+1)! 
2^(k+1), (k+1)! = k!*(k+1) e 2^(k+1) = 2*2^k (de acordo?).
Portanto, a igualdade fica k!(k+1)  2(2^k). Como temos que k!  2^k e 
(k+1)
 2 (pelo enunciado), então, está demonstrado.
Isso estaria certo?

Abraços,
Henrique.
=
Instruções para entrar na lista, sair da lista e usar a lista em
http://www.mat.puc-rio.br/~nicolau/olimp/obm-l.html
=
_
MSN Hotmail, o maior webmail do Brasil.  http://www.hotmail.com
=
Instruções para entrar na lista, sair da lista e usar a lista em
http://www.mat.puc-rio.br/~nicolau/olimp/obm-l.html
=


[obm-l] Re: [obm-l] Re: [obm-l] Prova por indução finita

2003-07-22 Por tôpico Frederico Reis Marques de Brito
Claro!. A idéia central para se demonstrar a desigualdade   k!  2^k   é 
óbvia: Nos dois produtos há k fatores, só que no 1o produto eles são, exceto 
2, maiores que 2, enquantop no 2o... , mas a questão foi enviada com o 
pedido de que fosse demonstrada pelo princípioda Indução.
Frederico.


From: Johann Peter Gustav Lejeune Dirichlet 
[EMAIL PROTECTED]
Reply-To: [EMAIL PROTECTED]
To: [EMAIL PROTECTED]
Subject: Re: [obm-l] Re: [obm-l] Prova por indução finita
Date: Mon, 21 Jul 2003 14:52:31 -0300 (ART)

pode-se demonstrar que k!/2^k pode ser tapo
grande como se queira
 --- Frederico Reis Marques de Brito
[EMAIL PROTECTED] escreveu:  Olá
Denisson. Essa é dauele tipo em que se usa
 um truque sujo utilíssimo.
 Deixo os detalhes por sua conta e vamos direto
 ao ponto:

 Suponha que k! 2^k.Então(k+1)! = (k+1)
 . k!  (k+1). 2^k  , pela
 hipótese de indução. Como  k=4 , claramente
 k+1 2   =   (k+1)! 
 2^{k+1} .


 O outro se resolve com um truque  dessemesmo
  naipe.

 Abraços,

 Frederico.


 From: denisson [EMAIL PROTECTED]
 Reply-To: [EMAIL PROTECTED]
 To: [EMAIL PROTECTED]
 Subject: [obm-l] Prova por indução finita
 Date: Sun, 20 Jul 2003 15:56:13 -0300
 
 Alguem pode resolver essas pra mim?
 
 Prove por indução finita:
 
 n!2^n, para todo n=4
 
 Prove por indução finita:
 n²2n+1, para todo n=3
 
 obrigado
 
 Denisson
 
 

__
 Acabe com aquelas janelinhas que pulam na sua
 tela.
 AntiPop-up UOL - É grátis!
 http://antipopup.uol.com.br/
 
 

=
 Instruções para entrar na lista, sair da lista
 e usar a lista em

http://www.mat.puc-rio.br/~nicolau/olimp/obm-l.html

=


_
 MSN Messenger: converse com os seus amigos
 online.
 http://messenger.msn.com.br


=
 Instruções para entrar na lista, sair da lista
 e usar a lista em

http://www.mat.puc-rio.br/~nicolau/olimp/obm-l.html

=
___
Yahoo! Mail
Mais espaço, mais segurança e gratuito: caixa postal de 6MB, antivírus, 
proteção contra spam.
http://br.mail.yahoo.com/
=
Instruções para entrar na lista, sair da lista e usar a lista em
http://www.mat.puc-rio.br/~nicolau/olimp/obm-l.html
=
_
MSN Messenger: converse com os seus amigos online.  
http://messenger.msn.com.br

=
Instruções para entrar na lista, sair da lista e usar a lista em
http://www.mat.puc-rio.br/~nicolau/olimp/obm-l.html
=


Re: [obm-l] TFA - Teorema Fundamental da Algebra

2003-07-22 Por tôpico Frederico Reis Marques de Brito
Olá Paulo, bom ter reenviado a prova de Cauchy. Acaso o Teorema de Bolzano a 
que se refere é o tb conhecido como Teorema do Valor Intermediário ( ou em 
realidade algo equivalente a ele ) ? Se não, qual o enunciado?

Obrigado,
FRederico.

From: Paulo Santa Rita [EMAIL PROTECTED]
Reply-To: [EMAIL PROTECTED]
To: [EMAIL PROTECTED]
Subject: [obm-l] TFA - Teorema Fundamental da Algebra
Date: Mon, 21 Jul 2003 18:15:03 +
Ola Pessoal,

Revendo a mensagem na qual aprresento a PROVA DE CAUCHY  para o Teorema
Fundamental da Algebra achei-a um tanto confusa, pois eu estava escrevendo 
com
pressa. Como este Teorema e importante, dificilmente encontrado em livros 
do
ensino medio e sendo a prova de Cauchy simples, facilmente acompanhavel por
um estudante dedicado, resolvi re-escrever a prova, colocando detalhes de 
forma que
qualquer pessoa possa entender.

Esse Teorema tem provas mais longas e mais curvas. Usando Analise complexa 
a
prova e trivial e curtissima, mas nao acho que seja adeguado apresentar 
aqui, por
obvias razoes.

A IDEIA FUNDAMENTAL : A ideia subjacente a esta prova e a seguinte. Se 
everdade
que todo polinomio no plano de Argand tem raiz, entao esta raiz minimiza o 
seu modulo
e a suposicao de um minimo positivo deve conduzir a um absurdo. Como fazer 
este
absurdo surgir ? Considerando um circulo em torno do ponto que minimiza o 
modulo do
polinomio e tratando retas passando por este ponto. Em uma destas retas 
evidenciara
o absurdo. O resto e detalhe.

Segue a Prova de Cauchy :

Seja  P(X) = A0*(X^n) + A1*(X^n-1) + ... + An-1*X + An  um polinomio no 
qual os
coeficientes A0, A1, ..., An-1, An sao numeros complexos quaisquer e X e 
uma
variavel complexa. Queremos mostrar que existe Z complexo tal que :

P(Z) = A0*(Z^n) + A1*(Z^n-1) + ... + An-1*Z + An = 0.

Para tanto, seja M = MIN { MODULO( P(X) ), X variando em C }.

Como, por definicao, modulo( P(X) ) = 0. Segue que M = 0. Portanto, M 
pode
ser

PRIMEIRO CASO : M = 0.

Neste caso, existe um complexo Z0 tal que MODULO( P(Z0) ) = 0. Segue que
P(Z0) = 0 e portanto Z0 e uma raiz de P(X) e a demonstracao esta concluida.
SEGUNDO CASO : M  0.

Neste caso, seja Z0 o complexo tal que MODULO( P(Z0) ) = M. IMAGINANDO no
plano complexo um circulo de centro Z0 e raio R, segue que qualquer ponto Z 
na
circunferencia deste circulo pode ser imaginado como a extremidade de um 
vetor,
soma dos vetores :

Z0 : origem em (0,0) e extremidade no ponto Z0
Z1 : origem no ponto Z0, extremidade no ponto Z e modulo R
Assim, para qualquer Z na circunferencia do circulo, teremos :

Z = Z0 + Z1

Calculando agora P(Z), teremos :
P(Z)=P(Z0 + Z1)=A0*((Z0 + Z1)^n ) + A1*((Z0+Z1)^n-1 ) + ... + An-1*(Z0+Z1) 
+ An
Na expressao acima, ao expandirmos (Z0+Z)^p - p = 0,1,2, ..., n - usando o 
Binomio
de Newton, iremos obter as parcelas A0*(Z0^n), A1*(Z0^n-1), ..., An-1*Z0, 
An nas
quais nao aparece Z1 e diversas outras parcelas, nas quais sempre constara 
Z1 :

1) Sozinho, sem que apareca Z0.  Exemplos :
A0*(Z1^n), A1*(Z1^n-1), ..., An-1*Z1
2) Acompanhado de Z1. Exemplos :
BINOM(N,1)*A0*(Z0^n-1)*(Z1), BINOM(N,N-1)*A0*(Z0)*(Z1^n-1), ...
onde BINOM(N,P) = N! / ( P!*(N-P)! )
Esta observacao deixa claro que P(Z0+Z1) tera o seguinte aspecto :
P(Z0+Z1) = P(Z0) + B0*(Z1^n) + B1*(Z1^n-1) + ... + Bn*Z1
onde cada Bi e uma constante ou um  polinomio em Z0.
Claramente que dependendo dos Ai originais, de n e do valor de Z0, alguns 
destes
Bi poderao ser nulos. Se, alem de eliminar os Bi nulos, ordenarmos o 
polinomio em Z1
resultante segundo as potencias crescentes de Z1, renomeando a seguir os Bi 
por C's,
teremos algo semelhante a :

P(Z0+Z1) = P(Z0) + C1*(Z1^a) + C2*(Z1^b) + ... + Cp*(Z1^w).

Nesta ultima expressao acima :

1) Nenhum dos Ci e nulo, por construcao.
2) a  b  c  ...  w, em virtude da ordenacao
3) p = n, obvio.
Colocando C1*(Z1^a) em evidencia :
P(Z0+Z1) = P(Z0) + C1*(Z1^a)*[ 1 + (C2/C1)*(Z1^b-a) + ... + 
(Cp/C1)*(Z1^w-a) ]
Como estamos supondo P(Z0)  0, podemos dividir tudo po P(Z0). Dividindo :
P(Z0+Z1) / P(Z0) = 1 + ( C1/P(Z0) )*(Z1^a)*[ 1 + (C2/C1)*(Z1^b-a) + ... + 
(Cp/C1)*(Z1^w-a) ]

P(Z0+Z1) / P(Z0) = 1 + ( C1/P(Z0) )*[(Z1^a)*( 1 + (C2/C1)*(Z1^b-a) + ... + 
(Cp/C1)*(Z1^w-a) )]
Fazendo C1/P(Z0) = k :
P(Z0+Z1) / P(Z0) = 1 + k*[(Z1^a)*( 1 + (C2/C1)*(Z1^b-a) + ... + 
(Cp/C1)*(Z1^w-a) )]

P(Z0+Z1) / P(Z0) = 1  +  k*[(Z1^a)*( 1 + Z1*F(Z1) )
onde F(Z1) e uma funcao ( um polinomio ) em Z1.
Claramente que sao numeros complexos tanto k quanto Z1, podendo portanto 
serem
colocados na forma trigonometrica, isto e :

k = P*( cosQ + i*senQ )   e   Z1 = R( cosS + i*senS )
Portanto : k*(Z1^a) = P(R^a)*( cos(Q + aS) + i*sen(Q + aS) ). Dai :
P(Z0+Z1) / P(Z0) = 1+ P(R^a)*( cos(Q + aS) + i*sen(Q + aS) )*( 1 + Z1*F(Z1) 
)

PRESTE BASTANTE ATENCAO AQUI : Z0 e fixo. Ele e o complexo que torna 
modulo( P(Z) )
minimo. Segue que P(Z0) e um complexo fixo e que C1/P(Z0) tambem o e, pois 
C1 e uma constante ou um polinomio em Z0. Mas Z1 nao e fixo. Z1 e UM PONTO 
na circunferencia do circulo de centro 

Re: [obm-l] TFA - Teorema Fundamental da Algebra

2003-07-22 Por tôpico Frederico Reis Marques de Brito
Ah, ok! Acabo de encontrar o enunciado do Teorema de Bolzano, que prezumo, 
era o que vc havia se referido:

Sejam  P(x)  um polinômio a coeficientes reais   e   a  b números 
reais.  Se   P(a) . P(b) 0   então  P(x) tem um no par ( podendo ser = 0 )  
de zeros reais  no intervalo  aberto ( a , b )   e se  P(a) . P(b)  0, P(x) 
tem um número ímpar de zeros em  ( a , b ).

Segue a demonstração:

Podemos supor P(x) não constante( pois no caso cte nada temos a provar )  e 
mônico ( por simplicidade ). Pelo Teorema  Fundamental da Álgebra, podemos 
fatorar   P(x) completamente em fatores lineares:

P(x) = ( x-a1) . ( x-a2 ) ... ( x - ak ) . (x - z1 )  ...  (x -  zm)  .

Em que  os ai´s  são reais   e   os   bj´s   complexos não-reais e são as 
raízes de   P(x)=0.

OBS: Se P(x) não tiver raiz real  então P(x)0   para todo x real e tudo 
bem. Dessa forma podemos assumir que P(x) tenha alguma raiz real.

Mas tb sabemos que as raízes complexas não-reais só aparecem aos pares, z e 
conjugado de z , pois P(x) tem coef. reais. Daí, podemos reescrever :

P(x) = (x - a1)...(x - ak). Q(x) ,  com  Q(x) 0   para todo  x   real   =

P(a) . P(b) = ( a - a1) ... ( a - ak ) .( b - a1)  ...  ( b - ak). Q(a). 
Q(b)   .

Se   a  a_i   b  = a  - a_i 0   e   b  -   a_i  
0 .
Se   a_i  a = a - a_i  0   e   b  -   a_i 
 0 .
Se   a_i  b = a -  a_i 0   e   b  -
a_i  0 .

Desta forma, sea_i  é uma raiz de P(x)  no intervalo (a, b ) =  ( 
a-a_i). (b-b_i)0 . Caso contrário,  (a-a_i). (b -a_i) 0  . Decorre que se  
  P(a).P(b)  0  =  o número de raizes reais de P9x) em (a,b) é 
necessariamente par  e se P(a). P(b) 0  = o no de raízes reais de P(x) em 
(a,b) é necessariamente ímpar.

Abraços a todos.

Frederico.



From: Paulo Santa Rita [EMAIL PROTECTED]
Reply-To: [EMAIL PROTECTED]
To: [EMAIL PROTECTED]
Subject: [obm-l] TFA - Teorema Fundamental da Algebra
Date: Mon, 21 Jul 2003 18:15:03 +
Ola Pessoal,

Revendo a mensagem na qual aprresento a PROVA DE CAUCHY  para o Teorema
Fundamental da Algebra achei-a um tanto confusa, pois eu estava escrevendo 
com
pressa. Como este Teorema e importante, dificilmente encontrado em livros 
do
ensino medio e sendo a prova de Cauchy simples, facilmente acompanhavel por
um estudante dedicado, resolvi re-escrever a prova, colocando detalhes de 
forma que
qualquer pessoa possa entender.

Esse Teorema tem provas mais longas e mais curvas. Usando Analise complexa 
a
prova e trivial e curtissima, mas nao acho que seja adeguado apresentar 
aqui, por
obvias razoes.

A IDEIA FUNDAMENTAL : A ideia subjacente a esta prova e a seguinte. Se 
everdade
que todo polinomio no plano de Argand tem raiz, entao esta raiz minimiza o 
seu modulo
e a suposicao de um minimo positivo deve conduzir a um absurdo. Como fazer 
este
absurdo surgir ? Considerando um circulo em torno do ponto que minimiza o 
modulo do
polinomio e tratando retas passando por este ponto. Em uma destas retas 
evidenciara
o absurdo. O resto e detalhe.

Segue a Prova de Cauchy :

Seja  P(X) = A0*(X^n) + A1*(X^n-1) + ... + An-1*X + An  um polinomio no 
qual os
coeficientes A0, A1, ..., An-1, An sao numeros complexos quaisquer e X e 
uma
variavel complexa. Queremos mostrar que existe Z complexo tal que :

P(Z) = A0*(Z^n) + A1*(Z^n-1) + ... + An-1*Z + An = 0.

Para tanto, seja M = MIN { MODULO( P(X) ), X variando em C }.

Como, por definicao, modulo( P(X) ) = 0. Segue que M = 0. Portanto, M 
pode
ser

PRIMEIRO CASO : M = 0.

Neste caso, existe um complexo Z0 tal que MODULO( P(Z0) ) = 0. Segue que
P(Z0) = 0 e portanto Z0 e uma raiz de P(X) e a demonstracao esta concluida.
SEGUNDO CASO : M  0.

Neste caso, seja Z0 o complexo tal que MODULO( P(Z0) ) = M. IMAGINANDO no
plano complexo um circulo de centro Z0 e raio R, segue que qualquer ponto Z 
na
circunferencia deste circulo pode ser imaginado como a extremidade de um 
vetor,
soma dos vetores :

Z0 : origem em (0,0) e extremidade no ponto Z0
Z1 : origem no ponto Z0, extremidade no ponto Z e modulo R
Assim, para qualquer Z na circunferencia do circulo, teremos :

Z = Z0 + Z1

Calculando agora P(Z), teremos :
P(Z)=P(Z0 + Z1)=A0*((Z0 + Z1)^n ) + A1*((Z0+Z1)^n-1 ) + ... + An-1*(Z0+Z1) 
+ An
Na expressao acima, ao expandirmos (Z0+Z)^p - p = 0,1,2, ..., n - usando o 
Binomio
de Newton, iremos obter as parcelas A0*(Z0^n), A1*(Z0^n-1), ..., An-1*Z0, 
An nas
quais nao aparece Z1 e diversas outras parcelas, nas quais sempre constara 
Z1 :

1) Sozinho, sem que apareca Z0.  Exemplos :
A0*(Z1^n), A1*(Z1^n-1), ..., An-1*Z1
2) Acompanhado de Z1. Exemplos :
BINOM(N,1)*A0*(Z0^n-1)*(Z1), BINOM(N,N-1)*A0*(Z0)*(Z1^n-1), ...
onde BINOM(N,P) = N! / ( P!*(N-P)! )
Esta observacao deixa claro que P(Z0+Z1) tera o seguinte aspecto :
P(Z0+Z1) = P(Z0) + B0*(Z1^n) + B1*(Z1^n-1) + ... + Bn*Z1
onde cada Bi e uma constante ou um  polinomio em Z0.
Claramente que dependendo dos Ai originais, de n e do valor de Z0, alguns 
destes
Bi poderao ser 

[obm-l] Re: [obm-l] Re: [obm-l] Re: [obm-l] Prova por indução finita

2003-07-21 Por tôpico Frederico Reis Marques de Brito
Desta vez fui eu que não entendi sua dúvida. De qq forma pela experiência 
que tenho em sala de aula imaginoque o seguinte te ajude:

supomos que  k!  2^k  . Portanto, desde que  k+1 é positivo, podemos 
multiplicar essa desigualdade dos dois lados por (k+1)  =  (k+1). k!   
(k+1). 2^k   = (k+1)!  (k+1) . 2^k   2 . 2^k , pois   k+1  2 . Segue 
que  (k+1)!  2^{k+1} .

Ajudou? Se não, pode escrever novamente, mas explique-me melhor sua dúvida.

Boa Sorte.

Um abraço,
Frederico.

From: Henrique Patrício Sant'Anna Branco [EMAIL PROTECTED]
Reply-To: [EMAIL PROTECTED]
To: [EMAIL PROTECTED]
Subject: [obm-l] Re: [obm-l] Re: [obm-l] Prova por indução finita
Date: Sun, 20 Jul 2003 21:16:59 -0300
 Suponha que k! 2^k.Então(k+1)! = (k+1) . k!  (k+1). 2^k  , 
pela
 hipótese de indução. Como  k=4 , claramente   k+1 2   =   (k+1)! 
 2^{k+1} .

Não entendi a parte (k+1) . k!  (k+1). 2^k... Isso não deveria ser (k+1) .
k!  2 * 2^k.
Daí, sabemos que k!  2^k e, claramente, k + 1  2. Ou não?
Grato,
Henrique.
=
Instruções para entrar na lista, sair da lista e usar a lista em
http://www.mat.puc-rio.br/~nicolau/olimp/obm-l.html
=
_
MSN Hotmail, o maior webmail do Brasil.  http://www.hotmail.com
=
Instruções para entrar na lista, sair da lista e usar a lista em
http://www.mat.puc-rio.br/~nicolau/olimp/obm-l.html
=


[obm-l] Re: [obm-l] Prova por indução finita

2003-07-20 Por tôpico Frederico Reis Marques de Brito
Olá Denisson. Essa é dauele tipo em que se usa um truque sujo utilíssimo. 
Deixo os detalhes por sua conta e vamos direto ao ponto:

Suponha que k! 2^k.Então(k+1)! = (k+1) . k!  (k+1). 2^k  , pela 
hipótese de indução. Como  k=4 , claramente   k+1 2   =   (k+1)!  
2^{k+1} .

O outro se resolve com um truque  dessemesmo  naipe.

Abraços,

Frederico.


From: denisson [EMAIL PROTECTED]
Reply-To: [EMAIL PROTECTED]
To: [EMAIL PROTECTED]
Subject: [obm-l] Prova por indução finita
Date: Sun, 20 Jul 2003 15:56:13 -0300
Alguem pode resolver essas pra mim?

Prove por indução finita:

n!2^n, para todo n=4

Prove por indução finita:
n²2n+1, para todo n=3
obrigado

Denisson

__
Acabe com aquelas janelinhas que pulam na sua tela.
AntiPop-up UOL - É grátis!
http://antipopup.uol.com.br/
=
Instruções para entrar na lista, sair da lista e usar a lista em
http://www.mat.puc-rio.br/~nicolau/olimp/obm-l.html
=
_
MSN Messenger: converse com os seus amigos online.  
http://messenger.msn.com.br

=
Instruções para entrar na lista, sair da lista e usar a lista em
http://www.mat.puc-rio.br/~nicolau/olimp/obm-l.html
=


[obm-l] Probleminha Legal

2003-07-18 Por tôpico Frederico Reis Marques de Brito
Olá pessoal. Outro dia encontrei um probleminha interessante num livro que 
estava folheando. É o seguinte:

Mostre que o menor ângulo de medida inteira ( em graus )  que se pode 
construir ( usando apenas  com regua e compasso ) é o de   3graus.

Um abraço a todos.

Frederico.

=
Instruções para entrar na lista, sair da lista e usar a lista em
http://www.mat.puc-rio.br/~nicolau/olimp/obm-l.html
=
_
MSN Hotmail, o maior webmail do Brasil.  http://www.hotmail.com
=
Instruções para entrar na lista, sair da lista e usar a lista em
http://www.mat.puc-rio.br/~nicolau/olimp/obm-l.html
=


Re: [obm-l] problema curioso

2003-07-18 Por tôpico Frederico Reis Marques de Brito
Estranho. Creio que o enunciado está errado, pois dele conclui-se que  a=b 
ou a=c ou b=c ( ou  não exclusivo ). Assim temos apenas  2 ptos pelos 
quais passa a circunferência. Isto não a determina. Além disso, uma tal 
circunferência poderia ou não interceptar o eixo x.

Frederico.


From: Marcos Reynaldo [EMAIL PROTECTED]
Reply-To: [EMAIL PROTECTED]
To: [EMAIL PROTECTED]
Subject: [obm-l] problema curioso
Date: Fri, 18 Jul 2003 05:52:52 -0300 (ART)
jah ia me esquecendo no mesmo livro do Elon aparece
uma questao estranha, alguem entende ?
4) Em que ponto a circunferencia que passa pelos
pontos A=(a,0), B=(b,0) e C=(c,0) corta o eixo y ?
(Geometria Analitica e Algebra Linear - pagina 74)
Marcos
___
Yahoo! Mail
Mais espaço, mais segurança e gratuito: caixa postal de 6MB, antivírus, 
proteção contra spam.
http://br.mail.yahoo.com/
=
Instruções para entrar na lista, sair da lista e usar a lista em
http://www.mat.puc-rio.br/~nicolau/olimp/obm-l.html
=
_
MSN Messenger: converse com os seus amigos online.  
http://messenger.msn.com.br

=
Instruções para entrar na lista, sair da lista e usar a lista em
http://www.mat.puc-rio.br/~nicolau/olimp/obm-l.html
=


[obm-l] Probleminha Legal!

2003-07-18 Por tôpico Frederico Reis Marques de Brito
Olá´pessoal. Enviei uma mensagem com o problema abaixo há algum tempo, mas 
como ainda não a recebi, creio que houve algum problema. Portanto envio-o 
novamente. Desculpem-me se o receberem repetidas vezes.

Bom, folheando um livrinho de geometria, encontrei essa questão que julguei 
bonitinha:

Mostre que o menor ângulo com medida inteira ( em graus ) que podemos 
construir usando-se apenas régua e compasso é o de 3 graus.

Bom divertimento.
Um abraço a todos.
Frederico.

_
MSN Messenger: converse com os seus amigos online.  
http://messenger.msn.com.br

=
Instruções para entrar na lista, sair da lista e usar a lista em
http://www.mat.puc-rio.br/~nicolau/olimp/obm-l.html
=


Re: [obm-l]

2003-07-15 Por tôpico Frederico Reis Marques de Brito
Aexpressão é igual a 5exp(2003) / 5exp(2001)  = exp(2003)/exp(2001)= exp(2) 
. Desde que  e é aproximadamente igual a 2,7, decorre que a parte inteira de 
 e^2 = 7.  Logo, 7 é o maior inteiro que não supera o número dado pela 
expressão.


From: [EMAIL PROTECTED]
Reply-To: [EMAIL PROTECTED]
To: [EMAIL PROTECTED]
Subject: [obm-l] Date: Tue, 15 Jul 2003 23:55:56 -0300
Alguém me ajude com essa questão:
Qual é o maior valor inteiro que não supera o número:
( 2exp(2003)+3exp(2003)/(2exp(2001)+3exp(2001))
_
Voce quer um iGMail protegido contra vírus e spams?
Clique aqui: http://www.igmailseguro.ig.com.br
Ofertas imperdíveis! Link: http://www.americanas.com.br/ig/
Ofertas imperdíveis!
=
Instruções para entrar na lista, sair da lista e usar a lista em
http://www.mat.puc-rio.br/~nicolau/olimp/obm-l.html
=
_
MSN Hotmail, o maior webmail do Brasil.  http://www.hotmail.com
=
Instruções para entrar na lista, sair da lista e usar a lista em
http://www.mat.puc-rio.br/~nicolau/olimp/obm-l.html
=


[obm-l] Re: [obm-l] Re:_[obm-l]_O_Erro_Comum_da_Generalização_(_Ou_induções_equivocadas_)_

2003-07-14 Por tôpico Frederico Reis Marques de Brito
Para bom entender pingo é i e não há dúvida de que os matemáticos estão 
nesta classe. Posto isto, quero dizer que não admito que NINGUÉM, 
especialmente um ignorante em Matemática e em outros assuntos tb, venha até 
uma lista de discussão sobre Matemática , como alguém  bem disse , nossa 
casa, e nos desrespeite a todos. Por isso, sem palavras sábias, exijo que o 
sr paulo recolha-se a sua insignificância e retire-se dessa lista, deixe-nos 
em nosso mundo bitolado em sua opinião, mas harmônico, produtivo e belo 
para nós.

PARA O QUE NÃO TEM MAIS RAZÃO , A CALMA DO LOUCO ENSINOU A DIZER NADA,
PARA O QUE NÃO TEM MAIS NADA, A CALMA DO LOUCO ENSINOU A DIZER RAZÃO.
(A Sede do Peixe - Milton Nasc./Fernando Brant. )

[EMAIL PROTECTED] escreveu:  Não é
questão de sofisma nem são casos
 isolados.
 Se reclamo,é com sustentação
 argumentativa.Basta verificar a qualidade do
 ensino matemático e a aprendizagem dos alunos.

 Vc fica na defensiva e acredita que por meio de
 palavras sábias consegue esconder a verdade.É
 muito simples,os médicos hipócritas defendem os
 médicos hipócritas,os dentistas hipócritas
 defendem os dentistas hipócritas,e por aí vai.

 João Paulo


 - Original Message -
   From: Frederico Reis Marques de Brito
   To: [EMAIL PROTECTED]
   Sent: Sunday, July 13, 2003 1:16 AM
   Subject: [obm-l] O Erro Comum da
 Generalização ( Ou induções equivocadas )





   Há muito tenho me impressionado com a
 capacidade, creio exacerbada sobretudo
   nos brasileiros, de falarem com certeza sobre
 temas que absolutamente
   desconhecem. É bastante comum um aluno do
 ciclo básico de qq curso de
   engenharia afirmar que os cursos de cálculo
 diferencial e integral são
   desnecessários, posto que existem
 calculadoras científicas altamente
   avançadas.  Ainda pior,  muitos alunos dos
 ensinos fundamental e médio , e
   tb pré-vestibulandos  que , ainda que com
 enorme desconhecimento de
   mérito, apostam que a Matemática é algo
 abominável e absolutamente
   desnecessário a um cidadão comum, mesmo sem
 saber exatamente o que vem a
   ser Matemática. Afirmações do tipo: todos os
 professores (... )  são, à
   luz da lógica matemática, absolutamente
 levianas, pois se tratam da
   mal-fadada indução e´pírica, isto é, a
 generalização de um fato à partir de
   uma certa quantidade de casos particulares.
 Causa-me certa estranheza o
   comportamento desrespeitoso com que uma
 ciência, e evidentemente
   encontram-se aí desrespeitados seus
 pesquisadores, é tratada por aqueles
   que, por qq motivo, não a conhecem. è mais ou
 menos como a atitude da
   criança que nunca comeu uma certa verdura,
 mas não gosta da mesma. Seria
   sadio que nós professores de Matemática,
 principalmente, especialmente
   aqueles que formam os futuros professores ,
 como meu caso, tentássemos cada
   dia mais , reduzir o distanciamento entre o
 mito matemático e a realidade
   matemática. Só assim deixaríamos de ser
 vítimas( ou algozes... ) do sistema
   educacional vigente.

   Saudações,
   Frederico.



_
   MSN Hotmail, o maior webmail do Brasil.
 http://www.hotmail.com



=
   Instruções para entrar na lista, sair da
 lista e usar a lista em


http://www.mat.puc-rio.br/~nicolau/olimp/obm-l.html


=


 --
 Email.it, the professional e-mail, gratis per
 te: http://www.email.it/f

 Sponsor:
 L'interpretazione dei doni di orti, frutteti,
 prati e giardini nel nostro Ristoro Sunflower.
 Clicca qui:
http://adv.email.it/cgi-bin/foclick.cgi?mid=1478d=13-7
___
Yahoo! Mail
Mais espaço, mais segurança e gratuito: caixa postal de 6MB, antivírus, 
proteção contra spam.
http://br.mail.yahoo.com/
=
Instruções para entrar na lista, sair da lista e usar a lista em
http://www.mat.puc-rio.br/~nicolau/olimp/obm-l.html
=
_
MSN Hotmail, o maior webmail do Brasil.  http://www.hotmail.com
=
Instruções para entrar na lista, sair da lista e usar a lista em
http://www.mat.puc-rio.br/~nicolau/olimp/obm-l.html
=


[obm-l] O Erro Comum da Generalização ( Ou induções equivocadas )

2003-07-12 Por tôpico Frederico Reis Marques de Brito


Há muito tenho me impressionado com a capacidade, creio exacerbada sobretudo 
nos brasileiros, de falarem com certeza sobre temas que absolutamente 
desconhecem. É bastante comum um aluno do ciclo básico de qq curso de 
engenharia afirmar que os cursos de cálculo diferencial e integral são 
desnecessários, posto que existem calculadoras científicas altamente 
avançadas.  Ainda pior,  muitos alunos dos ensinos fundamental e médio , e 
tb pré-vestibulandos  que , ainda que com enorme desconhecimento de 
mérito, apostam que a Matemática é algo abominável e absolutamente 
desnecessário a um cidadão comum, mesmo sem saber exatamente o que vem a 
ser Matemática. Afirmações do tipo: todos os professores (... )  são, à 
luz da lógica matemática, absolutamente levianas, pois se tratam da 
mal-fadada indução e´pírica, isto é, a generalização de um fato à partir de 
uma certa quantidade de casos particulares. Causa-me certa estranheza o 
comportamento desrespeitoso com que uma ciência, e evidentemente 
encontram-se aí desrespeitados seus pesquisadores, é tratada por aqueles 
que, por qq motivo, não a conhecem. è mais ou menos como a atitude da 
criança que nunca comeu uma certa verdura, mas não gosta da mesma. Seria 
sadio que nós professores de Matemática, principalmente, especialmente 
aqueles que formam os futuros professores , como meu caso, tentássemos cada 
dia mais , reduzir o distanciamento entre o mito matemático e a realidade 
matemática. Só assim deixaríamos de ser vítimas( ou algozes... ) do sistema 
educacional vigente.

Saudações,
Frederico.
_
MSN Hotmail, o maior webmail do Brasil.  http://www.hotmail.com
=
Instruções para entrar na lista, sair da lista e usar a lista em
http://www.mat.puc-rio.br/~nicolau/olimp/obm-l.html
=


[obm-l] Re: [obm-l] [obm-l] Seu crêisson! Como os Matemáticos Complicam II

2003-07-11 Por tôpico Frederico Reis Marques de Brito
Esqueceu-se de mencionar que  : antes da igualdade (:=)   significa que o 
lado direito da igualdade é definido como o que se encontra à esquerda da 
igualdade.

Abraços a todos ( exceto o chado do Paulo, é claro! )

Frederico.


From: MuriloRFL [EMAIL PROTECTED]
Reply-To: [EMAIL PROTECTED]
To: [EMAIL PROTECTED]
Subject: [obm-l] [obm-l] Seu crêisson! Como os Matemáticos Complicam II
Date: Fri, 11 Jul 2003 00:33:42 -0300
Atenção Seu creisson esta de volta!!!

Vc não consegue se comunicar via e-mail ?!?!
Tem problemas com problemas ?!?!
Seus problemas se acabaram-se 
Curso rapido via e-mail!
Como se comunicar
+ := Soma
- := Subitração
* := Multiplicação
/ := Divisão
sqrt( ) := raiz quadrada
x^n := x elevado a n
= := igual
exemplo 001:
sqrt(a^2+b^2+c^4+d^567) := represeta a raiz da soma de a elevado ao 
quadrado, b elevado ao quadrado, c elevado a 4ª potencia e d elevado a 567ª 
potência!!!

P.s. Quem lê intende! Agora com équio!
  - Original Message -
  From: J.Paulo roxer ´til the end
  To: [EMAIL PROTECTED]
  Sent: Wednesday, July 09, 2003 6:13 PM
  Subject: [obm-l] Re2: [obm-l] Re: Como os Matemáticos Complicam II
  O a está debaixo do V +
  O 2-a está debaixo do próximo V
  =
  a-b está debaixo do próximo V
  e 3
  V 2-a debaixo desse último V
  =2
  V e a-b debaixo desse último V
  para
  0\ a \2 e b \a,então o valor da expressão
  6-a-4 vezes 2-a que está debaixo de V

  Entendendo que desse último 6-a até esse último 2-a,tem um V(são dois 
V,um dentro do outro)

  +2 e o a debaixo de V é igual a:

  Agora,se não der pra entender,vou ficar muito surpreso!
  Se ainda não der,dê o endereço que mando uma cópia impressa via 
correio,porq nunca vi gente que gosta de matemática ter tanta dificuldade.
- Original Message -
From: Eduardo Botelho
To: [EMAIL PROTECTED]
Sent: Wednesday, July 09, 2003 1:47 PM
Subject: [obm-l] Re: Como os Matemáticos Complicam II

J.Paulo roxer ´til the end wrote:

 Sabendo que raiz de a+raiz de 2-a= raiz de a -b e 3 raiz de 2 - 
a=2raiz
 de a -b para 0\ a \ 2 e b \a,então o valor da expressão raiz de 
6-a-4
 vezes raiz de 2-a + 2 raiz de a é igual :

  Do jeito que você propôs a questão, é obvio que ninguém vai entender
mesmo. Seria ótimo de você usasse parênteses ou coisas do tipo.
a + raiz de 2-a significa a + sqrt(2-a) ou a + sqrt(2) -a ??? E por aí
vai do começo até o fim do enunciado.
  Ok.Provavelmente tenho um grande problema no córtex cerebral.
 
  Pode fazer a grande gentileza de perguntar a qualquer grande mestre 
da
  Língua Portuguesa acerca da coerência da seguinte questão?Estou 
ficando
  convencido de que tenho algum problema mesmo.

Professores de Português são bons em Português. Matemáticos são bons 
em
Matemática.
Por favor, este tipo de afronta já está se tornando cansativa.
Agradeceria se você se limitasse apenas aos comentários matemáticos.

Eduardo


=
Instruções para entrar na lista, sair da lista e usar a lista em
http://www.mat.puc-rio.br/~nicolau/olimp/obm-l.html

=
  
  Email.it, the professional e-mail, gratis per te: clicca qui

  Sponsor:
  Trova il tuo artigiano di fiducia su QxService e parteciperai 
all'estrazione di un Nec e606 UMTS e di ricariche telefoniche! Clicca qui!
  Clicca qui

_
MSN Hotmail, o maior webmail do Brasil.  http://www.hotmail.com
=
Instruções para entrar na lista, sair da lista e usar a lista em
http://www.mat.puc-rio.br/~nicolau/olimp/obm-l.html
=


[obm-l] Re: [obm-l] Como os Matemáticos Complicam!

2003-07-10 Por tôpico Frederico Reis Marques de Brito
Provavelmente o seu principal problema com a Matemática é odiá-la 
profundamente, bloqueando seu raciocínio para novos métodos, novas idéias. 
Verdadadeiramente não se aprende nada se não há uma predisposição para 
tanto.
OBS:  Desculpe-me mas já me cansei de suas mensagens e portanto não mais as 
lerei.

Frederico.


From: J.Paulo roxer ´til the end [EMAIL PROTECTED]
Reply-To: [EMAIL PROTECTED]
To: [EMAIL PROTECTED]
Subject: [obm-l] Como os Matemáticos Complicam!
Date: Tue, 8 Jul 2003 17:35:13 -0300
Como sabem,sou semi analfabeto em matem.,em razão do ensino ultrapassado 
dado pelos  professores da referida matéria. De qualquer maneira,posso 
perceber as pegadinhas de mau gosto depois de ver o gabarito.

Estas são algumas das questões do vestibular que fiz.



Num certo grupo de pessoas, metade lê o jornal A NOTÍCIA e um terço lê 
O INFORMATIVO, mas somente um sexto lê ambos os jornais. Do grupo, a 
quantidade de pessoas que não lêem, nem A NOTÍCIA e nem O INFORMATIVO, 
é:

A) metade B) um terço C) um quarto D)um sexto.

Solução:

A Notícia

a+x/6=x/2  a=x/3

O Informativo

x/6+b=x/3  b=x/6

a+x/6+b+c=x

x/3+x/6+x/6+c=x

6=x/3-Resposta item b.

A parte em negrito é a solução pra complicar ,confundir o aluno.

Bem,como já vi o gabarito,noto que a maneira que vai direto ao ponto é 
apenas sabendo que a metade de 6 é 3.(Resposta:1/3)

O número de pares ordenados distintos (p, q) de números inteiros 
positivos, cuja soma é 20 e cujo produto é menor do que 54, é:

A)4

B)5

C)6

D)7

O enunciado já é complicado.Novamente o método mais confuso em negrito:

{p,q} Z+ e:

1)p+q=20

2)p.q54

De 1) em 2),temos:

p.q54

p(20-p)54

p²-20p+540

Valores possíveis para p serão 1,2,3,17,18,19 e q serão 19,18,17,3,2,1

Teremos 6 pares(1,15),(2,18),(3,17),(17,3),(18,2),(19,1)

Resposta:item c





--
Email.it, the professional e-mail, gratis per te: http://www.email.it/f
Sponsor:
Vai su QXservice! Chiama l'artigiano che risolve i tuoi problemi e 
partecipa al concorso Vinci un Videotelefono UMTS e ricariche telefoniche! 
Clicca qui!
Clicca qui: http://adv.email.it/cgi-bin/foclick.cgi?mid=1411d=9-7
_
MSN Hotmail, o maior webmail do Brasil.  http://www.hotmail.com
=
Instruções para entrar na lista, sair da lista e usar a lista em
http://www.mat.puc-rio.br/~nicolau/olimp/obm-l.html
=


Re: [obm-l] Duvida sobre polinomios

2003-07-09 Por tôpico Frederico Reis Marques de Brito
Sim, desde que o polinômio divisor não seja nulo. Existe um resultado, 
análogo ao Lema da Divisão de Euclides para nos inteiros, que garante que 
dados   polinõmios   f(x), g(x) , g(x)  0,  com coeficientes num corpo K ( 
em particular se  K= R = conjunto dos numeros reais )  então existem e são 
únicos  q(x) e  r(x)  com coeficientes em  K  de tal forma que f(x) = 
g(x) X q(x) + r(x) e  r(x) =0   ougrau(r)  gr(g) .  A 
demonstração desse fato é, normalmente, obtida através de indução 
matemática. Você pode obtê-la, por exemplo, em

[1]GONÇALVES, Adilson -  Introdução Á Álgebra  -  Projeto Euclides - 
SBM/IMPA.
[2]   DOMINGUES, Hygino H. , IEZZI, Gelson -  Álgebra Moderna-  Atual 
Editora.

Observe ainda que a hipótese de que os polinômios tenham coeficientes num 
corpo ( anel comutativo com elemento neutro do produto e no qual todo 
elemento não -nulo tenha inverso, ufa!!! ) é absolutamente essencial. SE 
dividirmos  F(x)= x+1   por   G(x)  = 2   , olhando-os como polinômios a 
coef. inteiros, não obteremos um quociente com esta propriedade.
Frederico.

From: leonardo mattos [EMAIL PROTECTED]
Reply-To: [EMAIL PROTECTED]
To: [EMAIL PROTECTED]
Subject: [obm-l] Duvida sobre polinomios
Date: Wed, 09 Jul 2003 12:05:03 +
Gostaria de saber se ao dividirmos um polinomio P(x) com todos os seus 
coeficientes pertencentes aos reais por um polinomio Q(x) tambem com todos 
os seus coeficientes pertencentes aos reais o o resto da divisao tem que 
ser necessariamente um polinomio de coeficientes reais. Se nao gostaria de 
ver um exemplo pelo menos.

Leonardo

_
MSN Hotmail, o maior webmail do Brasil.  http://www.hotmail.com
=
Instruções para entrar na lista, sair da lista e usar a lista em
http://www.mat.puc-rio.br/~nicolau/olimp/obm-l.html
=
_
MSN Messenger: converse com os seus amigos online.  
http://messenger.msn.com.br

=
Instruções para entrar na lista, sair da lista e usar a lista em
http://www.mat.puc-rio.br/~nicolau/olimp/obm-l.html
=


Re: [obm-l] matrizes

2003-07-09 Por tôpico Frederico Reis Marques de Brito
2)   Os autovalores de A são os zeros de seu polinômio característico   
p_A(x) = det ( A - x I ) , em que  I  representa a matriz identidade de 
mesma ordem que  A . Pela Regra de Binnet   det( C . D ) = det (C)  . Det  
(D) .  Suponha então que   B = P^{-1} . A . P , P não-singuilar.  Nesse 
caso:

p_A(x) = det( P^{-1} . ( A -xI) . P ) = det (P^{-1} . A . P - xI ) = p_B(x) 
.

Desde que A e B têm os mesmos polin}ômios caract. terão os mesmos 
autovalores.

OBS: na penúltima igualdade, usamos o fato de que  I  comuta com quaisquer 
outras matrizes, dessa forma: P^{-1} . (xI ). P = x .I  .

Frederico.


From: Marcos Reynaldo [EMAIL PROTECTED]
Reply-To: [EMAIL PROTECTED]
To: [EMAIL PROTECTED]
Subject: [obm-l] matrizes
Date: Wed, 9 Jul 2003 05:44:57 -0300 (ART)
Olá !
Alguém poderia me ajudar nesses problemas ?
Provar que:
i) se uma matriz A é triangular superior (ou
inferior), então a inversa de A é triangular superior
(ou inferior). (usando determinantes)
ii) se A e B são semelhantes* , então A e B possuem os
mesmos autovalores.
* A e B são semelhantes se existir uma matriz
inversível P tal que   (inversa de P).A.P=B
[]'s Marcos

___
Yahoo! Mail
Mais espaço, mais segurança e gratuito: caixa postal de 6MB, antivírus, 
proteção contra spam.
http://br.mail.yahoo.com/
=
Instruções para entrar na lista, sair da lista e usar a lista em
http://www.mat.puc-rio.br/~nicolau/olimp/obm-l.html
=
_
MSN Messenger: converse com os seus amigos online.  
http://messenger.msn.com.br

=
Instruções para entrar na lista, sair da lista e usar a lista em
http://www.mat.puc-rio.br/~nicolau/olimp/obm-l.html
=


[obm-l] QUESTÕES INTERESSANTES

2003-07-09 Por tôpico Frederico Reis Marques de Brito
Olá caros colegas da lista, seguem algumas questões que considero muito 
interessantes, para quem quiser se distrair um pouco nessas férias:

(1)   Seja   A   uma matriz 2x2  com entradas inteiras. Mostre que  A tem 
inversa  com entradas ingteiras   se, e somente se,   det(A) = + - 1 . ( 
Uma das implicações é trivial. )

(2)   Um triângulo equilátero, inscrito numa circunferência de centro na 
origem do sistema cartesiano, tem  o   número complexoz= sqrt(3)  + i   
como um de seus vértices. Determine os outros.

(3)  Uma equação quadrática com coeficientes primos pode ter raiz real 
dupla?

(4) Sejam   a   , bnúmeros naturais.  Mostre  que (3^a  + 3^b + 1 )   
nunca é um quadrado perfeito.

(5) Seja  p   5   um número primo. Neste caso   1/p , o recíproco de  p , 
tem por  representação decimal uma dízima periódica. Indiquemos por  T(p)  o 
número de algarismos que constituem o período.
(a)  Mostre que existe um único primo   p  tal que  T(p)  = 2 .
(b) Mostre que  T(p)   é o menor número natural  n   tal   que:  n | 
(p-1) e  p | R_n  ,  em que  R_n = ...1 ( n  dígitos 1 ) .
Obs:  a | b  quandob   é múltiplo de  a , isto é, existe  q  inteiro tal 
que   b = a . q   .

(6) Mostre que se   a , b, c  são inteiros  ímpares   então a eq. 
ax^2+bx+c=0   não tem raiz racional.
( Proposto por Eduardo Wagner no 1o encontro da RPM. )

Até a próxima.
Frederico.

From: Frederico Reis Marques de Brito [EMAIL PROTECTED]
Reply-To: [EMAIL PROTECTED]
To: [EMAIL PROTECTED]
Subject: Re: [obm-l] Duvida sobre polinomios
Date: Wed, 09 Jul 2003 11:08:23 -0300
Sim, desde que o polinômio divisor não seja nulo. Existe um resultado, 
análogo ao Lema da Divisão de Euclides para nos inteiros, que garante que 
dados   polinõmios   f(x), g(x) , g(x)  0,  com coeficientes num corpo K 
( em particular se  K= R = conjunto dos numeros reais )  então existem e 
são únicos  q(x) e  r(x)  com coeficientes em  K  de tal forma que f(x) 
= g(x) X q(x) + r(x) e  r(x) =0   ougrau(r)  gr(g) .  A 
demonstração desse fato é, normalmente, obtida através de indução 
matemática. Você pode obtê-la, por exemplo, em

[1]GONÇALVES, Adilson -  Introdução Á Álgebra  -  Projeto Euclides - 
SBM/IMPA.
[2]   DOMINGUES, Hygino H. , IEZZI, Gelson -  Álgebra Moderna-  Atual 
Editora.

Observe ainda que a hipótese de que os polinômios tenham coeficientes num 
corpo ( anel comutativo com elemento neutro do produto e no qual todo 
elemento não -nulo tenha inverso, ufa!!! ) é absolutamente essencial. SE 
dividirmos  F(x)= x+1   por   G(x)  = 2   , olhando-os como polinômios a 
coef. inteiros, não obteremos um quociente com esta propriedade.
Frederico.

From: leonardo mattos [EMAIL PROTECTED]
Reply-To: [EMAIL PROTECTED]
To: [EMAIL PROTECTED]
Subject: [obm-l] Duvida sobre polinomios
Date: Wed, 09 Jul 2003 12:05:03 +
Gostaria de saber se ao dividirmos um polinomio P(x) com todos os seus 
coeficientes pertencentes aos reais por um polinomio Q(x) tambem com todos 
os seus coeficientes pertencentes aos reais o o resto da divisao tem que 
ser necessariamente um polinomio de coeficientes reais. Se nao gostaria de 
ver um exemplo pelo menos.

Leonardo

_
MSN Hotmail, o maior webmail do Brasil.  http://www.hotmail.com
=
Instruções para entrar na lista, sair da lista e usar a lista em
http://www.mat.puc-rio.br/~nicolau/olimp/obm-l.html
=
_
MSN Messenger: converse com os seus amigos online.  
http://messenger.msn.com.br

=
Instruções para entrar na lista, sair da lista e usar a lista em
http://www.mat.puc-rio.br/~nicolau/olimp/obm-l.html
=
_
MSN Hotmail, o maior webmail do Brasil.  http://www.hotmail.com
=
Instruções para entrar na lista, sair da lista e usar a lista em
http://www.mat.puc-rio.br/~nicolau/olimp/obm-l.html
=


Re: [obm-l] primos

2003-07-04 Por tôpico Frederico Reis Marques de Brito
Creio que este enunciado está mal formulado. Não há em geral  n  primos  = 
n+1  .
Frederico.



From: Rafael [EMAIL PROTECTED]
Reply-To: [EMAIL PROTECTED]
To: OBM [EMAIL PROTECTED]
Subject: [obm-l] primos
Date: Thu, 3 Jul 2003 20:06:12 -0300 (ART)
Sendo n um número natural maior ou igual a 2,
designemos por p1 , p2 , p3, ...,pn os números primos
não superiores a n+1 e ponhamos  P = p1 . p2 ... pn.
Sabendo que na sequência de n números consecutivos P+2
, P+3 ,..., P+(n+1) não existe nenhum número primo,
considere uma dessas sequências com 10 termos. Seu
primeiro termo é:
resposta: 9242

___
Yahoo! Mail
Mais espaço, mais segurança e gratuito: caixa postal de 6MB, antivírus, 
proteção contra spam.
http://br.mail.yahoo.com/
=
Instruções para entrar na lista, sair da lista e usar a lista em
http://www.mat.puc-rio.br/~nicolau/olimp/obm-l.html
=
_
MSN Messenger: converse com os seus amigos online.  
http://messenger.msn.com.br

=
Instruções para entrar na lista, sair da lista e usar a lista em
http://www.mat.puc-rio.br/~nicolau/olimp/obm-l.html
=


Re: [obm-l] Conjunto convexo?

2003-07-02 Por tôpico Frederico Reis Marques de Brito
Sim, admitindo-se como intervalos os intervalos degeneradosvazio  e 
formados por um único no real  c : [c, c]  .
Frederico.


From: carlos.henr1 [EMAIL PROTECTED]
Reply-To: [EMAIL PROTECTED]
To: [EMAIL PROTECTED]
Subject: [obm-l] Conjunto convexo?
Date: Wed,  2 Jul 2003 11:59:32 -0300
Todo CONJJUNTO CONVEXO na reta dos reais é um INTEVALO???

[]'s

__
Seleção de Softwares UOL.
10 softwares escolhidos pelo UOL para você e sua família.
http://www.uol.com.br/selecao
=
Instruções para entrar na lista, sair da lista e usar a lista em
http://www.mat.puc-rio.br/~nicolau/olimp/obm-l.html
=
_
MSN Messenger: converse com os seus amigos online.  
http://messenger.msn.com.br

=
Instruções para entrar na lista, sair da lista e usar a lista em
http://www.mat.puc-rio.br/~nicolau/olimp/obm-l.html
=


Re: [obm-l] Divisibilidade

2003-06-27 Por tôpico Frederico Reis Marques de Brito
Qto a 2a pergunta, usando qq múltiplo do mmc, em particular, o produto dos 
números...

~qto a primeira não me lembro exatamente qual o critério de divisibilidade 
por 17, mas todos os critérios podem ser demonstrados, normalmente sem gdes 
problemas, olhando-se para as classes residuais nesse caso, devemos olhar 
módulo 17...

Frederico.



From: Denisson [EMAIL PROTECTED]
Reply-To: [EMAIL PROTECTED]
To: [EMAIL PROTECTED]
Subject: [obm-l] Divisibilidade
Date: Fri, 27 Jun 2003 01:32:38 -0300 (ART)
Alguém poderia demonstrar como se chegou aos critérios de divisibilidade? 
Em especial aos mais dificeis como o critério do 17. Não peço uma 
demonstração matemática formal, peço algum argumento lógico.

Foi dito tb na lista há um bom tempo que não é preciso tirar o MMC para se 
realizar uma soma de frações. Eu nunca havia pensado nisso, como posso 
somar duas frações como 2/5+1/8 sem tirar o mmc?

Obrigado

Denisson



-
Yahoo! Mail
O melhor e-mail gratuito da internet: 6MB de espaço, antivírus, acesso 
POP3, filtro contra spam.
_
MSN Messenger: converse com os seus amigos online.  
http://messenger.msn.com.br

=
Instruções para entrar na lista, sair da lista e usar a lista em
http://www.mat.puc-rio.br/~nicolau/olimp/obm-l.html
=


Re: [obm-l] Derivada

2003-06-25 Por tôpico Frederico Reis Marques de Brito
Podemos interpretar a derivada primeira de várias formas: como o coeficiente 
angular( inclinação ) da reta tangente ao gráfico da função, como velocidade 
de um ponto que se move em linha reta tendo a posição dada em função do 
tempo, ou, mais geralmente como taxa de variação da função. A derivada 
segunda informa, portanto, a taxa de variação da derivada primeira. 
Fisicamente, isto é a aceleração: taxa de variação da velocidade em relação 
ao tempo. Geometricamente, a derivada segunda nos informa sobre como as 
retas tangentes ao gráfico de f variam.

Frederico.


From: Patrick Passos [EMAIL PROTECTED]
Reply-To: [EMAIL PROTECTED]
To: [EMAIL PROTECTED]
Subject: [obm-l] Derivada
Date: Wed, 25 Jun 2003 04:59:33 -0300 (ART)
Bom será que alguem poderia me dar uma ajuda com relação a derivada? Eu 
precisava saber em uma explicação rapida qual o significado da primeira e 
da segunda derivada, e também o significado do valor de x quando a segunda 
derivada é igual a zero. Se possivel tambem exemplos de graficos das três 
perguntas.. Bom, obrigado..
Até mais..



-
Yahoo! Mail
Mais espaço, mais segurança e gratuito: caixa postal de 6MB, antivírus, 
proteção contra spam.
_
MSN Messenger: converse com os seus amigos online.  
http://messenger.msn.com.br

=
Instruções para entrar na lista, sair da lista e usar a lista em
http://www.mat.puc-rio.br/~nicolau/olimp/obm-l.html
=


[obm-l] Re: [obm-l] Livro - Teoria dos Números

2003-06-24 Por tôpico Frederico Reis Marques de Brito
Recomendo-lhe dois livros:

SANTOS, José Plínio de Oliveira -  Introdução à Teoria dos Números  - 
Coleção Matemática Universitária - SBM

MILIES, César Polcino  , COELHO, Sônia Pitta - Números: Uma Introdução à 
Matemática - edUSP

O segundo trata de temas mais básicos, o priimeiro faz interessantes 
ligações entre combinatória e teoria dos números e  avança até o estudo de 
raízes primitivas, resíduos quadráticos e fraçoes parciais.

OBS: ambos tem preço acessível, algo em torno de R$25,00, cada.

Frederico.


From: Victor Luiz [EMAIL PROTECTED]
Reply-To: [EMAIL PROTECTED]
To: [EMAIL PROTECTED]
Subject: [obm-l] Livro - Teoria dos Números
Date: Mon, 23 Jun 2003 21:05:43 -0300
-BEGIN PGP SIGNED MESSAGE-
Hash: SHA1
Indo de carona nessas recomendações de livros que têm sido enviados para a
lista eu gostaria de que me recomendassem algum livro básico que trate da
teoria dos números.
Obrigado,

Victor Luiz Salgado de Lima.

- 
Spam sux. www.wecanstopspam.org
-BEGIN PGP SIGNATURE-
Version: GnuPG v1.2.1 (MingW32) - GPGOE 0.4.1
iD8DBQE+95XWpBwZ7xrHmVsRApEpAJ9FFOr2m5dx/QvAYcVTMYFNUmR4+ACeN5a9
yK0v7zUzhowGTGO2kYButko=
=Xqs/
-END PGP SIGNATURE-
=
Instruções para entrar na lista, sair da lista e usar a lista em
http://www.mat.puc-rio.br/~nicolau/olimp/obm-l.html
=
_
MSN Hotmail, o maior webmail do Brasil.  http://www.hotmail.com
=
Instruções para entrar na lista, sair da lista e usar a lista em
http://www.mat.puc-rio.br/~nicolau/olimp/obm-l.html
=


Re: [obm-l] 2 questoes

2003-06-23 Por tôpico Frederico Reis Marques de Brito
1) O primeiro problema me parece mal formulado. Entendendo que o 
enunciado´seria:
 Dado um no racional a/b não-nulo e diferente de 1, existe n=2 inteiro tal 
que raiz n-esima de a/b é irracional., uma demonstração pode ser a 
seguinte:

Indiquemos a raiz n-ésima de   x   por   R_n(x)  .

Podemos tomar   x = a/b , com a e b coprimos ( isto é, sem fator comum ) . 
Podemos supor   a  0, pois  caso a0, tome n=2.  Suponha também  a 1.  
Neste caso,  a   admite uma fatoração única . Seja  pum fator primo 
qualquer  de   a ( note que  p não pode ser fator de b   ! )   e sejam   
 o expoente depna fatoração de   a . Tome n = m+1. Então   
R_n (x ) é irracional. De fato, supondo   R_n(x) = c/d , com  c  e  d  
coprimos, decorre quea/b = c^n/d^n =  ad^n=bc^n . Como  a  e  b são 
coprimos, cada fator primo de a, em particular  p,  deve dividir   c ( logo 
não pode dividir  d  !!! )  e, portanto, p^n  dividebc^n =  p^n  divide 
  ad^n   = p^n divide  a. Absurdo! A maior potência de  p  que divide  a  
é p^m   e   m = n-1

Se  a = 1, repita o procedimento acima para   b .

2)   f(x) = | -8 - 3 sen(x) |   .-1 =sen(x) =1= -11 = -8 
-3sen(x) = -5 , o primeiro valor atingido quando sen(x)=1   e o segundo 
quando sen(x)=-1, portanto quandox=pi/2  e  x= -pi/2, respectivamente.  
Como   g(x)=-8-3sen(x)   é uma função continua, decorre do TVI , que  a 
imagem de   g(x)   é o intervalo fechado  [-11 , -5 ] . Logo, a imagem de  
f(x)=|g(x)| = [0, 11] .

Tente  um procedimento análoga para a outra função.

Um abraço,
Frederico.
From: [EMAIL PROTECTED]
Reply-To: [EMAIL PROTECTED]
To: [EMAIL PROTECTED]
Subject: [obm-l] 2 questoes
Date: Sun, 22 Jun 2003 11:59:04 EDT
Ola pessoal,





Como resolver estes:





1) Prove que, dado um número racional a/b e umnúmero natural n maior ou

igual a 2, nem sempre raiz enésima de a/b eh racional.







2) Qual o conjunto imagem das seguintes funcoes:



a) ƒ(x)=|-8-3senx|



b) ƒ(x)=|-2+3cosx|




_
MSN Messenger: converse com os seus amigos online.  
http://messenger.msn.com.br

=
Instruções para entrar na lista, sair da lista e usar a lista em
http://www.mat.puc-rio.br/~nicolau/olimp/obm-l.html
=


Re: [obm-l] Livros

2003-06-23 Por tôpico Frederico Reis Marques de Brito
Esse livro tb encontra-se traduzido: AS PROVAS ESTÃO NO LIVRo, da editora 
Edgard Blucher.
Realmente muito bom...

Frederico.


From: Johann Peter Gustav Lejeune Dirichlet 
[EMAIL PROTECTED]
Reply-To: [EMAIL PROTECTED]
To: [EMAIL PROTECTED]
Subject: Re: [obm-l] Livros
Date: Mon, 23 Jun 2003 13:44:07 -0300 (ART)

Proofs from THE BOOK,Martin Aigner e Gunter Ziegler

Diego Navarro [EMAIL PROTECTED] wrote: Aproveito para incentivar os 
membros da lista a também fornecerem dicas de
 bons livros, sempre que possível.

Discrete thoughts, Kac/Rota/Schwartz. Tem uma cópia na biblioteca da PUC. 
Não tão
interessante, mas mais acessível, Descartes' Dream - não lembro mais o 
autor.
Davis/Hersh, acho.

E claro, o clássico, A mathematician's apology do Hardy. Volta e meia 
estou relendo um
dos três.

=
Instruções para entrar na lista, sair da lista e usar a lista em
http://www.mat.puc-rio.br/~nicolau/olimp/obm-l.html
=


-
Yahoo! Mail
Mais espaço, mais segurança e gratuito: caixa postal de 6MB, antivírus, 
proteção contra spam.
_
MSN Hotmail, o maior webmail do Brasil.  http://www.hotmail.com
=
Instruções para entrar na lista, sair da lista e usar a lista em
http://www.mat.puc-rio.br/~nicolau/olimp/obm-l.html
=


[obm-l] Re: [obm-l] Re: [obm-l] limites_funções_várias_variáveis

2003-06-22 Por tôpico Frederico Reis Marques de Brito
Bom eu não usei essa expressão, quem usou foi o autor do primeiro email ao 
qual respondi. É claro que limites laterais só existem em dimensão 1. Quanto 
ao erro, realmente há um grave erro no meu exemplo, que o professor Morgado 
felizmente percebeu. Não é possível fazer x0 e y0  tendendo a 0 e 0 , sob 
a curva que defini, posto que qdo x-0, x-10 = y0. Coisas que ocorrem qdo 
respondemos a uma pergunta sem refletir o necessário sobre a resposta.
Frederico.


From: Angelo Barone Netto [EMAIL PROTECTED]
Reply-To: [EMAIL PROTECTED]
To: [EMAIL PROTECTED]
Subject: [obm-l] Re: [obm-l] limites_funções_várias_variáveis
Date: Sat, 21 Jun 2003 18:56:45 -0300 (EST)
Caro Frederico.
Nao e usual falar de limites laterais no R^2 pois ai
nao ha dois lados; contudo se V. encontrar duas semiretas
opostas segundo as quais os limites sejam distintos
e correto concluir que o limite nao existe.
Tome mais cuidado  com os perentesis. Seu exemplo
parece conter um engano de digitaca, razao por que
vou dar outro exemplo:
f(x,y)=(x/|x|)+y, definida em R^2\{(0,0)}.
lim f quando (x,y) tende a (0,0) nao existe pois

se (x,0) tende a (0,0) com x0, f tende a -1,
se (x,0) tende a (0,0) com x0, f tende a 1 e -1/=1
(/+ significa e diferente de).
Angelo Barone{\ --\ }Netto   Universidade de Sao Paulo
Departamento de Matematica Aplicada  Instituto de Matematica e Estatistica
Rua do Matao, 1010   Butanta - Cidade Universitaria
Caixa Postal 66 281  phone +55-11-3091-6162/6224/6136
05311-970 - Sao Paulo - SP   fax +55-11-3091-6131
Agencia Cidade de Sao Paulo
.








=
Instruções para entrar na lista, sair da lista e usar a lista em
http://www.mat.puc-rio.br/~nicolau/olimp/obm-l.html
=
_
MSN Messenger: converse com os seus amigos online.  
http://messenger.msn.com.br

=
Instruções para entrar na lista, sair da lista e usar a lista em
http://www.mat.puc-rio.br/~nicolau/olimp/obm-l.html
=


[obm-l] Re: [obm-l] limites_funções_várias_variáveis

2003-06-21 Por tôpico Frederico Reis Marques de Brito
Note que se o limite existisse independeria da forma de aproximação da 
origem. Considere então  x, y0, tendendo a 0 e 0.

E Analogamente, tomar y = 0 , com relação à sua última pergunta, e em 
seguida fazer os limites laterais encerra a mesma idéia dada acima.está 
correta então.

Frederico.

From: Marcos Reynaldo [EMAIL PROTECTED]
Reply-To: [EMAIL PROTECTED]
To: [EMAIL PROTECTED]
Subject: Re: [obm-l] limites_funções_várias_variáveis
Date: Sat, 21 Jun 2003 02:20:21 -0300 (ART)
Frederico, obrigado por responder. Não entendi a
passagem abaixo onde você diz para fazer y=x/(x-1).
Para o caso de x e y estarem sem o módulo entendi, mas
x e y estão em módulo, como me livrar deles ? Outra
coisa, posso usar normalmente os limites laterais para
funções de duas variáveis, como no caso abaixo (no
final do e-mail) ?
obrigado

[]'s Marcos

 --- Frederico Reis Marques de Brito 
(b) Considerando o caminho x=0 (eixo oy ) , o limite
dá zero e tomando o caminho   y = x/(x-1), que
evidentemente passa pela origem, o limite vai dar
1.
Frederico.

From: Marcos Reynaldo [EMAIL PROTECTED]
Caros colegas, comecei a estudar um pouco de limites
de funções de várias variáveis, mas tem alguns que não
consegui entender. Se alguém puder me ajudar agradeço.
...
2) calcule, se existir:
b) lim(xy/(|x|+|y|) para (x,y)--(0,0)
Outra questão, em lim(x-y/(x^2+y^2) para (x,y)--(0,0)
posso aplicar limites laterais (aqueles que se
estuda em cálculo 1 ?
Tipo, considere (x,y) caminhando ao longo do eixo x
até (0,0). Então tenho pontos do tipo (x,0).
Simplificando vou ter lim 1/x se x tende a zero pela
esquerda dá menos infinito , se for pela direita mais
infinito, então o limite não existe.
A minha dúvida aqui é se posso usar os conceitos de
limites laterais do cálculo 1 para expressões de
várias variáveis.
[]'s Marcos
___
Yahoo! Mail
Mais espaço, mais segurança e gratuito: caixa postal de 6MB, antivírus, 
proteção contra spam.
http://br.mail.yahoo.com/
=
Instruções para entrar na lista, sair da lista e usar a lista em
http://www.mat.puc-rio.br/~nicolau/olimp/obm-l.html
=
_
MSN Messenger: converse com os seus amigos online.  
http://messenger.msn.com.br

=
Instruções para entrar na lista, sair da lista e usar a lista em
http://www.mat.puc-rio.br/~nicolau/olimp/obm-l.html
=


[obm-l] Re: [obm-l] limites_funções_várias_variáveis

2003-06-18 Por tôpico Frederico Reis Marques de Brito
1) Esse tirar coelho da cartola  só é possível para quem tem alguma 
maturidade matemática. Vc já tinha um caminho ao longo do qual o limite 
dava zero, precisava de outro onde o limite desse diferente de zero. Como há 
x^2 no numerador e x - 0, a idéia mais simples é fazer com que o 
denominador tb se torne um múltiplo de x^2, para simplificarmos e obttermos 
um limite não-nulo. Daí,  para que x +y=x^2, tomamos y = x^2-x, uma 
par[ábola que passa pela origem.

Quanto ao problema 2 ),
(a) note que a função seno é limitada, isto é,  -1=sen(w)=1. Como  x -0 , 
decorre que  x * sen(1/x^2+1/y^2) -0  também.

(b) Considerando o caminho x=0 (eixo oy ) , o limite dá zero e tomando o 
caminho   y = x/(x-1), que evidentemente passa pela origem, o limite vai dar 
1.

Frederico.

From: Marcos Reynaldo [EMAIL PROTECTED]
Reply-To: [EMAIL PROTECTED]
To: [EMAIL PROTECTED]
Subject: [obm-l] limites_funções_várias_variáveis
Date: Wed, 18 Jun 2003 05:52:15 -0300 (ART)
Caros colegas, comecei a estudar um pouco de limites
de funções de várias variáveis, mas tem alguns que não
consegui entender. Se alguém puder me ajudar agradeço.
1) mostre que lim(x^2/(x+y)) quando(x,y)--(0,0) não
existe.
Nesse aqui, vi uma solução em que um cidadão pegava
pontos percorrendo o eixo y, onde o limite dá zero.
Depois ele pega pontos da curva y=x^2-x, ai o limite
dá 1. Pela regra dos dois caminhos conclui-se que o
limite não existe. Não entendi de onde veio o tal do
y=x^2-x (parece uma coisa do tipo tirar coelho da
cartola).
2) calcule, se existir:
a) lim(x*sen(1/x^2+1/y^2) para(x,y)--(0,0)
b) lim(xy/(|x|+|y|) para (x,y)--(0,0)
Outra questão, em lim(x-y/(x^2+y^2) para (x,y)--(0,0)
posso aplicar limites laterais (aqueles que se estuda
em cálculo 1 ?
Tipo, considere (x,y) caminhando ao longo do eixo x
até (0,0). Então tenho pontos do tipo (x,0).
Simplificando vou ter lim 1/x se x tende a zero pela
esquerda dá menos infinito , se for pela direita mais
infinito, então o limite não existe.
A minha dúvida aqui é se posso usar os conceitos de
limites laterais do cálculo 1 para expressões de
várias variáveis.
[]'s Marcos

___
Yahoo! Mail
Mais espaço, mais segurança e gratuito: caixa postal de 6MB, antivírus, 
proteção contra spam.
http://br.mail.yahoo.com/
=
Instruções para entrar na lista, sair da lista e usar a lista em
http://www.mat.puc-rio.br/~nicolau/olimp/obm-l.html
=
_
MSN Messenger: converse com os seus amigos online.  
http://messenger.msn.com.br

=
Instruções para entrar na lista, sair da lista e usar a lista em
http://www.mat.puc-rio.br/~nicolau/olimp/obm-l.html
=


Re: [obm-l] [PROF] criterios de correcao

2003-06-09 Por tôpico Frederico Reis Marques de Brito
Se entendi bem o que vc fez, não.

Para provar uma igualdade  A = B, não podemos assumir que essa igualdade 
seja válida, manipular os dois membros da igualdade até constatar uma 
igualdade verdadeira. Por exemplo   2  = 3  , multiplicando por  0 :   
obtemos 0 = 0 , que é verdadeira. Ou ainda:-2 = 2 é falsa, mas elevando 
ao quadrado: 4 = 4.
Assim, a forma correta é , à partir de um dos lados da igualdade, devemos, 
por meio de implicações verdadeiras, concluir o outro lado. É claro que, tb 
podemos, em alguns casos, desenvolver os doias membros SEPARADAMENTE, isto 
é, sem que haja interação entre A e B, reduzindo os dois membros a uma mesma 
expressão C, teremos provado que:

A= C   e  B = C  = A = C. Entretanto, nas minha opinião, esse é um processo 
altamente deselegante e deve, o quanto possível, ser evitado. Não devemos 
nos esquecer que a Matemátixca além de uma ciência tem um que de arte, e por 
isso, beleza é essencial.
Frederico.

From: Ariel de Silvio [EMAIL PROTECTED]
Reply-To: [EMAIL PROTECTED]
To: [EMAIL PROTECTED]
Subject: [obm-l] [PROF] criterios de correcao
Date: Sun, 08 Jun 2003 23:04:43 -0300
Essa é uma pergunta pros professores, que corrigem provas...
sobre criterio de correçao...
especificamente em questoes PROVE QUE, DEMOSTRE QUE...
como responder
um exemplo:

Prove que
(1 + senx + i*cosx) / (1 - senx - i*cosx) = (tgx + secx)*i
para todo x real, xpi/2 + k*pi
bom, desenvolvi, e cheguei a tal equação
1 + senx + i*cosx = 1 + senx + i*cosx
é assim q se responde?? logicamente com todo o desenvolvimento ate ai...

[]s
Ariel
=
Instruções para entrar na lista, sair da lista e usar a lista em
http://www.mat.puc-rio.br/~nicolau/olimp/obm-l.html
=
_
MSN Hotmail, o maior webmail do Brasil.  http://www.hotmail.com
=
Instruções para entrar na lista, sair da lista e usar a lista em
http://www.mat.puc-rio.br/~nicolau/olimp/obm-l.html
=


Re: [obm-l] livro

2003-06-08 Por tôpico Frederico Reis Marques de Brito
Como todo livro de cálculo tem uma série de defeitos e algumas vantagens 
também. De forma geral indico que nunca se estude por um único livro, 
qualquer que seja a disciplina.  Nesse caso, indico que vc dê uma olhada no 
Leithold, por exemplo.

Frederico.

From: adr.scr.m [EMAIL PROTECTED]
Reply-To: [EMAIL PROTECTED]
To: [EMAIL PROTECTED]
Subject: [obm-l] livro
Date: Sun,  8 Jun 2003 09:43:08 -0300
ai toh no 1ºperíodo de engenharia e toh usando o livro
de cálculo:
Um Curso de Cálculo
Hamilton Luiz Guidorizzi
e gostaria de saber o q acham dele,pq soh  toh estudando
por ele.
obrigado.
__
Seleção de Softwares UOL.
10 softwares escolhidos pelo UOL para você e sua família.
http://www.uol.com.br/selecao
=
Instruções para entrar na lista, sair da lista e usar a lista em
http://www.mat.puc-rio.br/~nicolau/olimp/obm-l.html
=
_
MSN Messenger: converse com os seus amigos online.  
http://messenger.msn.com.br

=
Instruções para entrar na lista, sair da lista e usar a lista em
http://www.mat.puc-rio.br/~nicolau/olimp/obm-l.html
=


[obm-l] Re: [obm-l] Off-Topic: Recomendação de Filme

2003-06-06 Por tôpico Frederico Reis Marques de Brito
boa tarde caros colegas. Interesso-me por vídeos sobre Matemática, mas sua 
mensagem não deixa claro que onde se encontram esses vídeos, me interesso 
sobretudo pelo primeiro. Eles foram feitos por alguma universidade 
brasileira? Estão disponíveis na rede?

Desde já agradeço.

Frederico.


From: Victor Luiz [EMAIL PROTECTED]
Reply-To: [EMAIL PROTECTED]
To: [EMAIL PROTECTED]
Subject: [obm-l] Off-Topic: Recomendação de Filme
Date: Wed, 4 Jun 2003 23:09:56 -0300
Uma vez um colega e eu fizemos perguntas aqui sobre o hipercubo. Pra quem
gostou ou ficou curioso sobre o assunto tem o filme Cubo 2: Hipercubo 
(Tem
o primeiro também que fala sobre números primos e tal) que fala um pouco
sobre isso... Apesar de o filme dividir bruscamente opiniões sobre a
qualidade dele eu não deixo de recomendar só pela parte matemática da 
coisa,
por menor que ela seja...

Victor Luiz Salgado de Lima.


Spam sux. www.wecanstopspam.org
=
Instruções para entrar na lista, sair da lista e usar a lista em
http://www.mat.puc-rio.br/~nicolau/olimp/obm-l.html
=
_
MSN Hotmail, o maior webmail do Brasil.  http://www.hotmail.com
=
Instruções para entrar na lista, sair da lista e usar a lista em
http://www.mat.puc-rio.br/~nicolau/olimp/obm-l.html
=


Re: [obm-l] Livros

2003-06-06 Por tôpico Frederico Reis Marques de Brito
Cara Dri,
Os problemas de Matemática não são de outro planeta senão o nosso mesmo, 
aopenas, muitas vezes é preciso construir uma sólida estrutura para atacar 
ou mesmo entender os problemas mais difíceis, afinal se fossem triviais não 
despertariam interesse.

No seu caso, acho que uma boa pedida seriam os livros da coleção A 
MATEMÁTICA DO ENSINO MÉDIO, uma publicação da SBM em três volumes., quee 
trata de temas classicos, mas como uma abordagem mais formal e com uma 
profundidade adequada. Você pode comprá-los na filial da SBM de sua cidade, 
veja a lista no site www.sbm.org.br , todos têm preços acessíveis. Depois 
disso, há vários livros destinados as Olimpíadas, tais como: É DIVERTIDO 
RESOLVER PROBLEMAS, de nosso colega de lista Luís Lopes ou a coletânea de 
problemas das Olimpíadas do Cone SUl, vendida pela Papel Virtual, só para 
citar algumas. Também indico, para que você conheça um pouco das diversas 
áreas da Matemática o best-seller  O QUE É A MATEMÁTICA, de Richard 
Courant e Hebert Robbins, da Cultura Moderna.

Boa sorte e espero quecomo você os desafios da Matemática sejam um convite 
ao estudo dessa ciência e não um princípio de exclusão.

Frederico.


From: Dri - uol [EMAIL PROTECTED]
Reply-To: [EMAIL PROTECTED]
To: [EMAIL PROTECTED]
Subject: [obm-l] Livros
Date: Wed, 4 Jun 2003 12:16:27 +0900
Caros colegas,

Acho fascinante cada resolucao nova que aparece por aqui(mesmo nao
entendendo boa parte...), e cada vez mais me desperta a vontade de 
aprender,
aprender e aprender...
Sinto muito orgulho de poder fazer parte de uma lista como esta, tendo
grande personalidades ao meu redor...
Sou apenas uma estudante de cursinho... ainda estou tendo nocoes basicas...
porem, adoro desafios...
Assim sendo, peco a ajuda de vcs, quem puder me indicar alguns livros de
exercicios ou com metodos de raciocinios diferentes... qualquer coisa que
possa ajudar a desenvolver o basico que tenho e me ajudar a encarar
problemas de outros planetas...

Desde jah agradeco,

[]s
Dri
=
Instruções para entrar na lista, sair da lista e usar a lista em
http://www.mat.puc-rio.br/~nicolau/olimp/obm-l.html
=
_
MSN Hotmail, o maior webmail do Brasil.  http://www.hotmail.com
=
Instruções para entrar na lista, sair da lista e usar a lista em
http://www.mat.puc-rio.br/~nicolau/olimp/obm-l.html
=


[obm-l] Re: Off topic-indignação( Que maluquice!!! )

2003-06-04 Por tôpico Frederico Reis Marques de Brito
Não sei o que se passa com os membros dessa lista, melhor dizendo, com 
alguns deles. Tão absurda a mensagem indignada de um sujeito que se inscreve 
numa lista de Problemas das Olimpíadas de Matemática e quer que as mensagens 
tratem de temas cotidianos e simplórios, ainda, de forma didática, quanto 
a resposta de um outro que não soube medir as palavras. Causa-me profunda 
indignação as duas atitudes e, talvez ainda mais a primeira. Mais uma vez 
sou forçado a dizer que as pessoas andam perdendo a noção. Fico entristecido 
com a retirada de nosso caro colega, pois sei que será uma inestimável 
perda. Não farei o mesmo pois, apesar desses últimos incidentes, acho a 
lista muito proveitosa e espero que O Nicolau tome as providências 
necessárias para colocar ordem nessa lista.

Sem mais, encerro dizendo que o respeito à ciência, em especial nesse caso à 
Matemática, é essencial. Vamos pensar um pouco mais antes de escrever 
mensagens para essa lista.

Frederico.


From: Carlos César de Araújo [EMAIL PROTECTED]
Reply-To: [EMAIL PROTECTED]
To: [EMAIL PROTECTED]
Subject: [obm-l] Re: [obm-l] Re: [obm-l] Off topic-indignação
Date: Tue, 3 Jun 2003 01:49:50 -0300
Prezados colegas,

Após a leitura de respostas como essas, RETIRO-ME desta lista. Chamem-me
quando houver um pouco mais de moderação.
- Original Message -
From: + BRiSSiU + [EMAIL PROTECTED]
To: [EMAIL PROTECTED]
Sent: Tuesday, June 03, 2003 12:58 AM
Subject: [obm-l] Re: [obm-l] Off topic-indignação
MessagePorque você não vai se foder?

Vai filosofar em outro lugar, rapaz.

Se você não é capaz de entender a notação matemática aqui utilizada, 
aprenda
e volte depois.

Se não gosta dos enunciados diretos, monte um enredo para si próprio, mas
não nos encha o saco com suas abobrinhas.
+ BRiSSiU +
  - Original Message -
  From: J.Paulo roxer ´til the end
  To: [EMAIL PROTECTED]
  Sent: Monday, June 02, 2003 8:30 PM
  Subject: [obm-l] Off topic-indignação
  Eu queria q nesta lista,os assuntos fossem tratados de forma mais
objetiva.Não entendo absolutamente nada do que vcs dizem.Estudo,continuo
estudando,mas não entra essas coisas.
  Se fosse só eu,eu poderia fazer uma auto-denominação ao me chamar de
estúpido mesmo,mas não é.Tenho mandado algumas questões que vcs põem, para
amigos e eles não entendem.Aliás,é raro alguém na net gostar de tratar de
assuntos como matemática e filosofia.
  Em todos meus anos de vida escolar,meus colegas inegavelmente não sabiam
do que o professor falava.
  É sempre uma explicação mal feita,superficial,que não visa o
engrandecimento intelectual do aluno.
  Já sei que matemática é até importante,mas deve ser ensinada de forma 
mais
concisa,tendo um objetivo.
  Não adianta explicar ou fazer uma questão que não se sabe pra que 
serve.De
nada adianta dar explicações x e y sem saber a razão que leva a isso.
  É como dizer:Siga por esta rua,vá andando.

  Vou tomar como exemplo,a questão colocada por um integrante da
lista.Mostre que n! = (n/2+1)^(n-1), ocorrendo desigualdade estrita para
n=3.
  Qual o objetivo?Um software pode ser construído usando isso?Um carro,uma
casa,um tubo de ensaio pode ser feito usando essa questão?
  É preciso dizer:Nessa questão,usa-se tal fórmula.Nessa outra,usa-se essa
fórmula.Nenhum livro de matemática é coeso,nenhum relata as origens,o 
porquê
e o objetivo (exemplos usados no dia-a-dia)
  Nas minhas aulas de física,chega a me dar sono de tanto tédio.

  Sinto muito por estarem lendo este e-mail,mas não sei onde colocar 
minhas
dúvidas.Estou em fase de vestibular,vou passar mesmo que seja sem saber
mat. e fís. e penso em fazer uma monografia ou algo do tipo que denuncie a
extrema incapacidade dos professores de matemática e física de
ensinar.Nenhum é conciso,prático e possuidor de senso crítico.Apenas
aceitam,aprendem e não sabem transmitir o que aprenderam.
  Uma coisa é aprender,a outra,é a arte de ser humilde e transmitir da 
forma
mais simples possível.

  Se alguém gosta de chat e quiser conversar sobre a má  temática,pode 
me
adicionar no msn messenger. [EMAIL PROTECTED]
  Acho que vou sair desta lista,não dá pra entender nada mesmo.

  João Paulo
  
  Email.it, the professional e-mail, gratis per te: clicca qui
  Sponsor:
  Solo su Occhialeria.it una vastissima scelta a prezzi insuperabili!
  Per te le migliori marche e un incredibile assortimento.
  Clicca qui




=
Instruções para entrar na lista, sair da lista e usar a lista em
http://www.mat.puc-rio.br/~nicolau/olimp/obm-l.html
=
_
MSN Messenger: converse com os seus amigos online.  
http://messenger.msn.com.br

=
Instruções para entrar na lista, sair da lista e usar a lista em
http://www.mat.puc-rio.br/~nicolau/olimp/obm-l.html

Re: [obm-l] Problema de aneis de polinomios

2003-06-02 Por tôpico Frederico Reis Marques de Brito
Não quero fazer patrulha ideológica, mas a sucessão de emails sobre esse 
tema me causou profunda estranheza. Sou obrigado a dizer que além de não 
saber o que vem a ser um polinômio, o Maçaranduba tb não deve ter noção de 
quem é o professor Morgado. É evidente que todos somos passíveis de erros, 
mas para se afirmar que o MOrgado deu um contra-exemplo furado é 
necessário pensar um bocado e ter muita coragem( acho que eu não teria )  
sobre isto antes, sobretudo em se tratando de um assunto tão simples, como o 
que motivou todas essas mensagens.

Enfim, acho que alguns partici´pantes da lista deveriam sintonizar-se.

Frederico.


From: A. C. Morgado [EMAIL PROTECTED]
Reply-To: [EMAIL PROTECTED]
To: [EMAIL PROTECTED]
Subject: Re: [obm-l] Problema de aneis de polinomios
Date: Sat, 31 May 2003 20:27:03 -0300
Voce sabe o que eh um polinomio? Isso que voce esta indicando aih eu nao 
sei o que eh, mas polinomio nao eh. Imagine se o seu corpo for R: 
produtorio de (x-w), w percorrendo os reais.
Voce sabe o que eh um polinomio nulo? Voce ja se deu ao trabalho de olhar a 
resposta do Carlos Cesar a sua pergunta?
Esta eh minha ultima manifestaçao a respeito.

Carlos Maçaranduba wrote:

usando o seu argumento eu poderia dizer que no caso de
um corpo infinito , eu poderia construir um produtorio
de (x - w) infinitos para todo w que pertence ao
corpo.Isto é possivel pelo teorema das raizes de um
polinomio num corpo.Entao eu obteria um polinomio não
nulo de infinitos fatoresNão é um polinomio nulo
como (x - 1)(x - 0)mod2 do seu exemplo..
--- A. C. Morgado [EMAIL PROTECTED]
escreveu:  Um polinomio eh  nulo quando os seus
coeficientes

sao nulos, o que nao eh o caso do polinomio f(x) = x^2 + x no corpo dos
inteiros modulo 2; dois dos coeficientes desse polinomio sao iguais a
1.
Entao, eh falso que f = 0.
Mas f(0) = f(1) = 0, ou seja, f(w) = 0 para todo w
em Z2.
Leia a resposta que lhe foi mandada por Carlos
César de Araújo e convença-se de que isso eh um contra-exemplo sim..
Carlos Maçaranduba wrote:



Pode ser que seja problema de interpretação, mas eu
acho que isto não é contra-exemplo PORQUE:
-Pela hipotese, eu nao deveria considerar, mesmo


para


um corpo finito que f(w) = 0 ,para todo elemento w
pertencente ao corpo finito e  CONCLUIR QUE f =0 É
FALSO NESTE CASO
-UM CONTRA-EXEMPLO BOM NAO SERIA RESPEITANDO O QUE


EU


DISSE ACIMA



--- A. C. Morgado [EMAIL PROTECTED]
escreveu:  f(x) = x^2 + x  em Z(2) eh um
contraexemplo.



Carlos Maçaranduba wrote:





Seja k um corpo infinito.Se f pertence a k[x] é


tal


que f(w) = 0 para todo elemento w pertencente a


k,


então f = 0.Mostrar por exemplo que esta




propriedade é




falha se k é finito.





___






Yahoo! Mail
Mais espaço, mais segurança e gratuito: caixa



postal de 6MB, antivírus, proteção contra spam.




http://br.mail.yahoo.com/




=






Instruções para entrar na lista, sair da lista e




usar a lista em



http://www.mat.puc-rio.br/~nicolau/olimp/obm-l.html






=














=






Instruções para entrar na lista, sair da lista e
usar a lista em

http://www.mat.puc-rio.br/~nicolau/olimp/obm-l.html






=



___


Yahoo! Mail
Mais espaço, mais segurança e gratuito: caixa

postal de 6MB, antivírus, proteção contra spam.


http://br.mail.yahoo.com/


=


Instruções para entrar na lista, sair da lista e


usar a lista em


http://www.mat.puc-rio.br/~nicolau/olimp/obm-l.html


=









___
Yahoo! Mail
Mais espaço, mais segurança e gratuito: caixa postal de 6MB, antivírus, 
proteção contra spam.
http://br.mail.yahoo.com/
=
Instruções para entrar na lista, sair da lista e usar a lista em
http://www.mat.puc-rio.br/~nicolau/olimp/obm-l.html
=





_
MSN Messenger: converse com os seus amigos online.  
http://messenger.msn.com.br

=
Instruções para entrar na lista, sair da lista e usar a lista em
http://www.mat.puc-rio.br/~nicolau/olimp/obm-l.html
=


[obm-l] Re: [obm-l] Fractais no ensino médio

2003-06-02 Por tôpico Frederico Reis Marques de Brito
Concordo com você, meu caro conterrâneo, exceto no que tange a inclusão de 
temas pedagógicos na lista ( e também aproveito para lembrar-lhe que alguns 
grandes matemáticos detestavam ensinar...). esse exemplo que citou e um dos 
típicos modismos que assolam os incautos. Especialmente em nossa área em que 
muitos professores dos ensinos fundamental e médio  sentem-se semi-deuses, 
embora, na maioria dos casos, pouco saibam sobre Matemática de verdade, como 
diria Hardy. Acho que a postura correta seria recorrer a hist´poria da 
Matemática que possui capítulos interessantíssimos e de mais fácil 
compreensão. Os problemas antigos, a novela da eq de terceiro grau, são 
exemplos motivadores.
Frederico Reis.


From: Carlos César de Araújo [EMAIL PROTECTED]
Reply-To: [EMAIL PROTECTED]
To: [EMAIL PROTECTED]
Subject: [obm-l] Fractais no ensino médio
Date: Sat, 31 May 2003 15:41:29 -0300
Prezados colegas,

Desculpem-me por levar à vossa consideração uma questão inapropriada para a
lista, mas minha opinião é que os MATEMÁTICOS e USUÁRIOS da nossa ciência
também devem se pronunciar sobre temas pedagógicos. Como se sabe, grandes
matemáticos do passado foram hábeis educadores; além disso, progressos
puramente técnicos em matemática decorreram de tentativas de esclarecer
tópicos durante o ENSINO dos mesmos.
Minha questão (algo polêmica) é a seguinte. Tenho observado, 
particularmente
aqui em Belo Horizonte, uma certa onda em falar sobre fractais e teoria 
do
caos para estudantes comuns (sem nenhuma conotação pejorativa) do ensino
médio. Recentemente, tomei conhecimento de professores que solicitoram aos
seus alunos que fizessem uma pesquisa pela Internet -- como se se
ensinasse metodologia científica nos colégios ... -- e escrevessem um texto
sobre fractais e caos. Dos alunos que me consultaram, nenhum conseguiu nota
máxima no trabalho, pelo fato de o respectivo professor ter alegado que o
texto não estava suficientemente claro. Fiquei imediatamente indignado, 
dado
que, tanto quanto sei, quase nenhum professor dessas instituições sabe
sequer DEFINIR tecnicamente o que é um fractal. Posso felicitar os
professores em sua tentativa de tornar a matemática mais atraente pela
sugestão de tais pesquisas de temas modernosos e de grande apelo visual,
mas considero irresponsável e ingênua tal proposta no contexto dessas
instituições, nas quais o aluno não conta com incentivos à sua auto-estima 
e
nem com o guia seguro de professores qualificados. O que vocês acham? Os
nossos alunos estariam realmente preparados para fazer pesquisas e produzir
textos razoáveis sobre temas tão avançados? O ensino médio já não
comportaria temas clássicos igualmente excitante e instrutivos para estudo
na Internet? Afinal, por que não pedem aos garotos para pesquisarem sobre
física qântica e a teoria gravitacional de Einstein?

Atenciosamente,

Carlos César de Araújo
Matemática para Gregos  Troianos
www.gregosetroianos.mat.br
Belo Horizonte, MG
=
Instruções para entrar na lista, sair da lista e usar a lista em
http://www.mat.puc-rio.br/~nicolau/olimp/obm-l.html
=
_
MSN Messenger: converse com os seus amigos online.  
http://messenger.msn.com.br

=
Instruções para entrar na lista, sair da lista e usar a lista em
http://www.mat.puc-rio.br/~nicolau/olimp/obm-l.html
=


Re: [obm-l] Problema de aneis de polinomios

2003-06-02 Por tôpico Frederico Reis Marques de Brito
Uma observação: normalmente emails são mensagens informais, escritos com 
linguagem coloquial. Não devemos nos ater a este tipo de detalhes: um email 
não é uma demonstração matemática. Aproveito a oportunidade para , apesar do 
que acabei de escrever, externar minha preocupação com o uso indiscriminado 
de estrangeirismos. Esse sim me parece um ponto relevante.

Frederico.

From: Eduardo Casagrande Stabel [EMAIL PROTECTED]
Reply-To: [EMAIL PROTECTED]
To: [EMAIL PROTECTED]
Subject: Re: [obm-l] Problema de aneis de polinomios
Date: Sun, 1 Jun 2003 12:34:17 -0300
Eu gostaria que chegássemos a uma solução definitiva com relação ao 
problema
da linguagem que se usa na lista, mas parece que não é possível, pois 
sempre
novos membros entram e falam como lhe bem entende.

Aqueles que já fazem parte da lista e já conhecem essa discussão,
principalmente relacionada à linguagem do Dirichlet, devem tomar uma 
posição
mais neutra em relação à linguagem, ignorando-a. Assim como diz o Cláudio,
acho que devemos nos focar mais na parte matemática das mensagens e deixar 
a
linguagem para trás. Desde que a mensagem não ofenda à pessoas, deixe que
usem termos como furado para falar sobre argumentos matemáticos, eles não
tem sentimentos e não se sentirão feridos.

Cada um que entra traz uma bagagem de linguagem, mas nós não estamos
interessados nela, estamos interessados na bagagem matemática. Eu próprio
estou ignorando bem mais a linguagem.
Abraço,
Eduardo.
From: Frederico Reis Marques de Brito [EMAIL PROTECTED]
 Não quero fazer patrulha ideológica, mas a sucessão de emails sobre esse
 tema me causou profunda estranheza. Sou obrigado a dizer que além de não
 saber o que vem a ser um polinômio, o Maçaranduba tb não deve ter noção 
de
 quem é o professor Morgado. É evidente que todos somos passíveis de 
erros,
 mas para se afirmar que o MOrgado deu um contra-exemplo furado é
 necessário pensar um bocado e ter muita coragem( acho que eu não teria )
 sobre isto antes, sobretudo em se tratando de um assunto tão simples, 
como
o
 que motivou todas essas mensagens.

 Enfim, acho que alguns partici´pantes da lista deveriam sintonizar-se.

 Frederico.

=
Instruções para entrar na lista, sair da lista e usar a lista em
http://www.mat.puc-rio.br/~nicolau/olimp/obm-l.html
=
_
MSN Messenger: converse com os seus amigos online.  
http://messenger.msn.com.br

=
Instruções para entrar na lista, sair da lista e usar a lista em
http://www.mat.puc-rio.br/~nicolau/olimp/obm-l.html
=


[obm-l] Novamente o LIvro.

2003-02-24 Por tôpico Frederico Reis Marques de Brito


Pessoal, por algum acaso do destino, alguém conseguiu adquirir os livros de 
Geometria I e II e o Álgebra I do Morgado? A livraria que o Morgado deu  o 
nome está com o telefone desligado há tempos, mandei uma carta, não me 
responderam. Alguém sabe outro lugar onde posso conseguí-los?
Desculpe a insistência, mas gostaria muito de adquirí-los.
Obrigado,
Frederico ( BH - MG. )



_
MSN Messenger: converse com os seus amigos online.  
http://messenger.msn.com.br

=
Instruções para entrar na lista, sair da lista e usar a lista em
http://www.mat.puc-rio.br/~nicolau/olimp/obm-l.html
O administrador desta lista é [EMAIL PROTECTED]
=


Re: [obm-l] Livro Geometria

2003-02-05 Por tôpico Frederico Reis Marques de Brito


Sobre os livros Geometria I e II e Álgebra I , tentei em vão falar no fone 
indicado pelo Morgado ( até rimou... ). Mas o telefone está programado para 
não receber ligações. Gostaria de saber os tópicos abordados nesses livros, 
os preços e se há alggum site ou algum representante da editora em Belo 
Horizonte. Aproveito para esclarecer uma dúvida conceitual. O que vem a ser 
um quadrilátero cíclico?
Desde já deixo aqui meus agradecimentos.
Frederico.




From: A. C. Morgado [EMAIL PROTECTED]
Reply-To: [EMAIL PROTECTED]
To: [EMAIL PROTECTED]
Subject: Re: [obm-l] Livro Geometria
Date: Fri, 17 Jan 2003 15:05:29 -0200

Agradecemos as referencias elogiosas do Paulo.
Corrijo as declaraçoes do Wagner:
1) o Geometria 1 eh um bom livro. Claro que, por tratar de conceitos
mais basicos e se dirigir basicamente a um leitor que seja um bom aluno
de SETIMA SERIE, sempre os autores (e leitores mais adiantados) acharao
que se poderia dar um tratamento axiomaticamente mais rigoroso; mas o
objetivo era fazer um livro de Geometria para vestibulares dificeis e
concursos de admissao a escolas militares, extremamente procuradas na
epoca; ainda hoje eh um bom livro para IME, ITA, Colegio Naval, etc.
2) Geometria 1, Geometria 2 e Algebra 1 podem ser pedidos a
FCZ Livros
Rua Carneiro Ribeiro 22 loja A
21050-570  Maria da GraçaRio de JaneiroRJ
Telefax  (21) 2581-2873
Morgado

Paulo Santa Rita wrote:

 Ola Leonardo e demais
 colegas desta lista ... OBM-L,

 Foi esse livro que despertou meu interesse pela Matematica.

 Voce teve a mesma impressao que eu tive, quando o li pela primeira
 vez, apos adquiri-lo em um sebo ( sebo = livraria de livros usados ).
 Muitas vezes me perguntei o que o tornava tao interessante e diferente
 dos outros ...

 Me parece que e porque os autores ( Eduardo Wagner e Augusto Morgado )
 nao perdem tempo provando coisas simples e evidentes, partindo
 imediatamente a exposicao de fatoss espetaculares e inusitadas, que
 dificilmente imaginariamos que ocorrem.

 Quando um matematico explica um fenomeno inusitado ( por exemplo : o
 circulo de nove pontos ) ele aguca nossa inteligencia e nos vemos
 claramente que a Matemaica nao se resume a provas de fatos evidentes
 e, portanto, desmotivadoras ( por exemplo : prove que 1+1=2 ).

 Um Matematico adulto pode apreciar o formalismo, mas uma mente nova
 geralmente nao aprecia estas coisas ... Prove a uma crianca algo que
 ela duvida e nao suspeia e voce vai conquistar o interesse dela ... me
 parece que e esse simples detalhe que diferencia as grandes obras da
 mesmice e mediocridade que campeia na imensa maioria das obras
 didaticas da matematica ...

 As Olimpiadas de Matematica, que todos nos gostamos e admiramos e
 pelas quais fazemos verdadeiros sacrificios e uma continuacao de tudo
 isso ... Por que elas fazem sucesso ? Simplesmente porque as pessoas
 inteligentes odeiam coisas rotineiras e burocraticas, que sao os
 aspectos tipicos do ensino comum ...

 Neste sentido, o Livro a que voce se refere, o Geometria II dos Prof
 Wagner e Morgado, se nao foi o marco inicial e corajoso de uma
 revolucao pedagogica, foi, ao menos, o alvorecer de tudo isso. E nos
 somos felizardos por termos em nossa lista estes dois Prof's que
 iniciaram esta revolucao : Wagner e Morgado.

 Seria excelente que estes Profs dessem continuidade ao que comecaram
 com tanto brilho e eficiencia !

 Para que esta mensagem nao fique totalmente fora de nossa tradicao,
 aqui vai uma joia do Geometria II :

 1)Sejam a, b, c e d os lados de um quadrilatero ciclico.
 Mostre qua a area S desse quadrilatero pode ser expressa como :
 S=Raiz_Quadrada((p-a)(p-b)(p-c)(p-d)) onde p e o semi-perimetro.

 2)Se o quadrilatero e incritivel e circunscritivel, entao :
 S=Raiz_Quadrada(abcd)

 PROBLEMA : Se ABC e um triangulo e C o circulo inscrito nele, sejam
 C1, C2 e C3 as tres partes da area do triangulo que nao pertencem ao
 circulo. Calcule cada uma, separadamente, em funcao dos lados do
 triangulo.

 Um Abraco a Todos
 Paulo Santa Rita
 6,,170103




 From: Leonardo Borges Avelino [EMAIL PROTECTED]
 To: [EMAIL PROTECTED]
 Subject: [obm-l] Livro Geometria
 Date: Thu, Jan 16, 2003, 10:07 PM


 Caros amigos:

  Estava na casa de meu amigo e ele me mostrou um livro
 impressionante, que
 se chama: Geometria II dos prof.s Eduardo Wagner, Augusto Morgado e
 Miguel.
 Pergunta:

 Onde consigo este livro? e (desculpem-me se a pergunta for idiota) se
 existe o Geometria I?


 Valeu!!
 Leonardo Borges



 _
 MSN Hotmail, o maior webmail do Brasil. http://www.hotmail.com

 
=
 Instruções para entrar na lista, sair da lista e usar a lista em
 http://www.mat.puc-rio.br/~nicolau/olimp/obm-l.html
 O administrador desta lista é [EMAIL PROTECTED]
 
=






[obm-l] Re: [obm-l] Retorno: polinômios

2003-01-30 Por tôpico Frederico Reis Marques de Brito
Bem, não tive paciência de conferir as contas. Como boa parte dos 
matemáticos, tenho certa aversão a cálculos tediosos...
Uma solução simples é a seguinte. Como sabemos que P1 e P2 tem grau 2, os 
quocientes das divisões por (x-1)(x+2) e (x+1)(x+2) são polinômios 
constantes: q1 e q2 , nessa ordem.
Dai:  P1(x)= (x-1)(x+2)q1 + 3x+1   e   P2(x)=(x+1)(x+2)q2 +(2x-1) .
Usando que P1(0)=P2(0)=0 , encontramos:  q1=q2=1/2 ==
P_1(x)=(1/2)x^2 + (7/2) x   e   P_2(x)=(1/2)x^2 + (9/2)x .
Evidentemente, dividindo-se P_1 por P_2, o quociente é o polinômio constante 
 Q(x)=1 .
Frederico Reis.






From: [EMAIL PROTECTED]
Reply-To: [EMAIL PROTECTED]
To: [EMAIL PROTECTED]
Subject: [obm-l] Retorno: polinômios
Date: Thu, 30 Jan 2003 19:52:05 EST

Olá pessoal,

Ontem eu enviei esta questão:

UnB) P1(x) e P2(x) são polinômios do 2ºgrau que se anulam quando x=0. O 
resto
da divisão de P1(x) por (x-1)(x+2) é 3x +1 . O resto da divisão de P2(x) 
por
(x+1)(x+2) é 2x - 1. Então o quociente da divisão de P1(x) por P2(x) é :

resp: 1

Obs: Houve a seguinte resposta na lista:
Como zero é raiz de P1(x) e P2(x):
P1(x)= ax^2 + bx
P2(x)= cx^2 + dx
Usando a divisão de polinômios:
Sendo = o símbolo de idêntidade
ax^2 + bx = (x-1)(x+2)Q(x) + (3x+1)
Da definição de identidade:
para x=1, temos: a+ b = 4
para x= -2, temos: 4a -2b= -5
Resolvendo o sistema: a=2 e b=2
Portanto, P1(x)=2x^2 + 2x
Analogamente faça com o polinômio P2(x)
Depois divida um polinômio pelo outro.
P.S:O resto é trabalho algébrico

Minhas dúvidas: O sistema acima não dá como resultado a=2 e b=2. Outra 
dúvida
foi também que resolvendo todo questão eu cheguei a Q(x)=6 e R(x)= -2x 
[ambos
valores da divisão de p1(x) por p2(x)], mas como a questão pede somente
Q(x)=6, mas as alternativas são
a)1
b)0
c)x+1
d)n.d.a
E o gabarito diz que é 1.





_
MSN Hotmail, o maior webmail do Brasil.  http://www.hotmail.com

=
Instruções para entrar na lista, sair da lista e usar a lista em
http://www.mat.puc-rio.br/~nicolau/olimp/obm-l.html
O administrador desta lista é [EMAIL PROTECTED]
=



[obm-l] Re: [obm-l] EXATIDÃO MATEMÁTICA

2003-01-20 Por tôpico Frederico Reis Marques de Brito
Caro Thyago, em primeiro lugar esee problema com a definição de círculo 
não pões em xeque a exatidão da Matemática. Dwefinição é apenas um nome, não 
importa qual seja o nome dado, importa o objeto matemático. Nessa linha, há 
autores que fazem distinção entre círculo e circunferência, enquanto outros 
não. Quanto ao disco, não há dúvida, representa a região interna delimitada 
pela cirdcunferência, incluindo o bordo ( a própria circunferência )  ou 
não, conforme seja fechado ou aberto. Não creio que circunferência refira-se 
apenas ao comprimento. E finalmente, creio que questões como essa devem , a 
fim de elucidar o enunciado e evitar dubiedades, apresentar a definição, 
isto é, no caso presente deveriam explicitar a que conjunto se referem. Aí, 
não faz muita diferença se chamaram de  círculo ou circunferência. OBS: é 
mais comum que se pensa, autores diferentes usarem definições diferentes, 
isto não causa confusão, desde que no contexto, seja explicitada a definição 
seguida .






From: Thyago Alexandre Kufner [EMAIL PROTECTED]
Reply-To: [EMAIL PROTECTED]
To: [EMAIL PROTECTED]
Subject: [obm-l] EXATIDÃO MATEMÁTICA
Date: Mon, 20 Jan 2003 19:39:43 -0200

Olá usuários da lista.

No último vestibular da UFRGS, ocorreu-me uma centelha de desconfiança
quanto à exatidão matemática.

A questão era o seguinte:

Na figura abaixo, A e B são vértices do quadrado inscrito no círculo

FIGURA: Um círculo e um quadrado inscrito com os dois vértices de baixo
marcados com A e B

Se um ponto E do círculo, diferente de todos os vértices do quadrado, é
tomado ao acaso, a probabilidade de que A, B e E sejam vértices de um
tri6angulo obtusângulo é
(A) 1/4
(B) 1/3
(C) 1/2
(D) 2/3
(E) 3/4

Quando fiz a cadeira de GEOMETRIA e também quando estudei no ensino médio,
aprendi que circunferência seria apenas a linha e círculo seria a porção
limitada pela circunferência. E também diríamos circunferência quando
quiséssemos nos referir ao comprimento desta linha.

Bom, a discussão é a seguinte: utilizando estas definições que aprendi, a
resposta para a questão seria 1-3/(2pi), pois o ponto E poderia estar
dentro, no círculo. Mas a solução dada pelo gabarito é letra E, que se
faz valer, de acordo com o aprendizado anterior, somente se tivesse escrito
Se um ponto E da CIRCUNFERÊNCIA.

Discutindo com alguns colegas, obtive como resposta mais satisfatória o
seguinte:

Hoje em dia tenho visto designar a região do plano por DISCO e a linha que
limita esta região por círculo. A palavra circunferência é usada para
designar o complrimento do círculo (linha).

Não sou contra as modificações nas nomenclaturas. O que chamo a atenção é,
como pode a mais exata das ciências trocar uma nomenclatura (círculo) que
antes definia uma coisa (porção do plano) para outra (que se fosse trocado
por um aluno anteriormente, seria motivo de erro)?
Se esta modificação fosse para uma palavra nova, seria melhor. Pois ao se
deparar com um novo termo, iríamos pesquisar para ver o que significa tal
termo. Mas, se o termo já existe, e designava algo diferente, acaba não nos
trazendo a dúvida, mas sim gerando erros de comunicação naquela que deveria
ser a comunicação mais exata existente (a matemática).

Se eu estivesse fazendo vestibular, e esta questão fosse dissertativa, com
certeza iria responder 1-3/(2pi), e, provavelmente, iria errar.

Gostaria de obter respostas das mais variadas pessoas desta lista, desde os
alunos atuais contando como foi seu aprendizado, até os mais graduados
contando como lidam com esta situação. Por favor, quando responderem,
indiquem qual o nível de relacionamente com a matemática que vocês têm :-)

Atenciosamente
Prof. Thyago
WebMaster cursinho.hpg.com.br

=
Instruções para entrar na lista, sair da lista e usar a lista em
http://www.mat.puc-rio.br/~nicolau/olimp/obm-l.html
O administrador desta lista é [EMAIL PROTECTED]
=



_
MSN Messenger: converse com os seus amigos online. 
http://messenger.msn.com.br

=
Instruções para entrar na lista, sair da lista e usar a lista em
http://www.mat.puc-rio.br/~nicolau/olimp/obm-l.html
O administrador desta lista é [EMAIL PROTECTED]
=


Re: [obm-l] Livros

2002-11-09 Por tôpico Frederico Reis Marques de Brito
Essa é uma tarefa difícil: Encontrar livros de Matemática bons e coerentes 
para o ensino médio. Boa parte deles, senão todos, é um amontoado de 
receitas de bolo, sem nenhuma preocupação científica. Nesse contexto 
encontra-se a coleção do professor Iezzi, a meu ver, inútil. OS livros da 
SBM pertencentes a coleção do professor suprem, em algumas áreas essa 
deficiência. Destaco em particular os três volumes da série Matemática do 
Ensino Médio, do prof Elon e colaboradores. Há alguns outros livros que não 
são da SBM. Citarei alguns:

[1] Introdução à Análise Combinatória - José Plínio O. Santos , Margarida P. 
Mello , Idani T. C. Murari  - 3a ed - Editora UNICAMP.  ( pode ser adquirido 
em qualquer Livraria UNiversitária, por exemplo na da UFPe, ou direto no 
site: www.editora.unicamp.br ( em torno de R$30,00).

[2] Introdução à Matemática - Licio Hernanes Bezerra , Paulo Henrique V. DE 
Barros, Carlos Tomei , Celso Wilmer - ed. da UFSC - Trata de métodos 
dedutivos e da construção dos conjuntos  N , Z , R  , de forma formal.  Pode 
ser adquirido na livraria da UFPe. ( em torno de R$20,00 )

[3] Números: Uma INtrodução à Matemática - César Polcino Milies , Sõnia 
Pitta Coelho -  edUSP.  UMa ótima introdução à teoria dos nos inteiros.  
Pode ser adquirido no site www.usp.br/edusp   ( cerca de R$20,00 ) .

Qual








From: bruno lima [EMAIL PROTECTED]
Reply-To: [EMAIL PROTECTED]
To: [EMAIL PROTECTED]
Subject: Re: [obm-l] Livros
Date: Fri, 8 Nov 2002 11:44:32 -0300 (ART)


Se esta for uma coleção de 10 volumes do Gelson Iezzi, acho que está muito 
bom. Mas se por um acaso tiver tempo e disposição pode estudar umas coisas 
mais aprofundadas, por exemplo, se gostar de :
1-Geometria, procure um do Coxeter (é em  ingles);
2-Álgebra, procure Int. à Teoria dos Números de José Plínio Santos, é da 
coleção matemática universi'tária custa uns 30 pilas , olhe www.impa.br em 
biografia;
3-Combinatória,procure Análise Combinatória , nao lembro o autor, da 
coleçao Professor de Matemática, é um vermelho tambem pode ser achado em 
www.impa.br deve custar uns 25.
4-Se gostar de outras coisas me manda um e-mail
 Renato Lira [EMAIL PROTECTED] wrote:Olá, eu sou estudante de 
Recife-PE e estou na 1ª série do Ensino Médio, eu estudo através da colecao 
Fundamentos de Matemática Elementar, creio que muitos desta lista a 
conheca bem como seus defeitos. Alguém poderia me apontar quais os assuntos 
em que tal coleção nao apresenta um bom grau de aprofundamento nos 
assuntos(tomando como referencia exames do IME e ITA) e, ao apontar, se 
possível indicar livros ou locais onde posso adquirir um material de 
qualidade com exercícios com um bom grau de aprofundamento. 
Grato pela atencao, Renato Lira


-
Yahoo! GeoCities
Tudo para criar o seu site: ferramentas fáceis de usar, espaço de sobra e 
acessórios.


_
MSN Hotmail, o maior webmail do Brasil. http://www.hotmail.com

=
Instruções para entrar na lista, sair da lista e usar a lista em
http://www.mat.puc-rio.br/~nicolau/olimp/obm-l.html
O administrador desta lista é [EMAIL PROTECTED]
=



Re: [obm-l] infinito

2002-06-29 Por tôpico Frederico Reis Marques de Brito

Imagino que ele tenha se referido a compactificação de IR. O conjunto dos 
números reais pode ser compactificado com a adjunção de infinito. É  como se 
o infinito fosse considerado como um número. Imagine o conjunto dos 
números reais como uma reta. Agora imagine um círculo.  Tire um ponto do 
círculo, o que sobra pode ser desenrolado   numa reta.  Este ponto que 
extraimos é o infinito . A construção, do ponto de vista intuitivo, é o 
inverso desta. Isto é, tomamos a reta e acrescentamos oum ponto fora dela, o 
infinito, o resultado é uma circunferência. Daí,  + e - infinito, as pontas 
da reta  são unidas e, portanto, coincidem na compactificação.
Espero ter ajudado.
Fred.


From: adr.scr.m [EMAIL PROTECTED]
Reply-To: [EMAIL PROTECTED]
To: [EMAIL PROTECTED]
Subject: [obm-l] infinito
Date: Sat, 29 Jun 2002 15:23:30 -0200

Outro dia meu professor (de FISICA),fez uma
representacao dos numeros de 0 ate + ou -
infinito e eu nao entendi muito bem.Ele os
colocou num circulo,e disse que nao existem
os numeros + ou - infinito,e` somente um
numero,e que o sinal dependeria por que lado
voce chegaria ao infinito,igual a ideia do
+ou- 0.Queria saber se esta certa e porque?E
porque nao seria uma reta?
Obrigado.
Adriano.


__
AcessoBOL, só R$ 9,90! O menor preço do mercado!
Assine já! http://www.bol.com.br/acessobol


=
Instruções para entrar na lista, sair da lista e usar a lista em
http://www.mat.puc-rio.br/~nicolau/olimp/obm-l.html
O administrador desta lista é [EMAIL PROTECTED]
=





=
Instruções para entrar na lista, sair da lista e usar a lista em
http://www.mat.puc-rio.br/~nicolau/olimp/obm-l.html
O administrador desta lista é [EMAIL PROTECTED]
=



  1   2   >